SlideShare a Scribd company logo
A 25 year old female, known to be infected with HIV and a CD4 count 90 presented with
complaints of altered sensorium following a partial seizure. Gadolinium enhanced MRI scan
showed multiple ring enhancing lesions in both hemispheres, thalamus, pons, basal ganglia
and cerebellum. The most probable diagnosis in this scenario is?
A: Mycobacterium tuberculosis
B:
Multiple brain
metastases
C: Cryptococcal meningitis
D: Pneumocystis infiltration of brain
Correct Ans:A
Explanation
HIV patients with low CD 4 counts if presents with multiple
ring enhancing lesions is suggestive of Tuberculoma, Primary
CNS lymphoma and Toxoplasmosis. In this scenario only TB
is the given choice and hence the answer.
Sample Previous Year Question on HIV based on previous Year Questions of
NEET PG, USMLE,PLAB,FMGE (MCI Screening). Please visit
www.medicoapps.org for more such Quizzes
A preterm baby developed bullous lesion on the skin soon after birth. The X- ray shows
periostitis. What should be the next investigation?
A: VDRL for mother & baby
B:
ELISA for
HIV
C: PCR for TB
D: Hepatitis surface antigen for mother
Correct Ans:A
Explanation
The baby described in the question stem is most likely suffering from the early
manifestations of congenital syphilis. As per the choices given VDRL is the test done to
diagnose syphilis.
Ref: Harrison’s Principles of Internal Medicine, 16th Edition, Page 981; Skin Diseases &
Sexually Transmitted Infections, Uday Khopkar – 6th Edition, Page 257; Nelson 17th
,COGDT 10th Edition, Page 667;CPDT 18th Edition, Page 56.
Sample Previous Year Question on HIV based on previous Year Questions of
NEET PG, USMLE,PLAB,FMGE (MCI Screening). Please visit
www.medicoapps.org for more such Quizzes
Thalidomide can be used in all of the following conditions, EXCEPT:
A: Behcet's syndrome
B: HIV associated peripheral neuropathy
C: HIV associated mouth ulcers
D: Erythema nodosum leprosum
Correct Ans:B
Explanation
Thalidomide has been used for many years in the treatment of some manifestations of
leprosy and has been used for erythema nodosum leprosum; it is also useful in
management of the skin manifestations of lupus erythematosus.
The most important toxicity is teratogenesis. Because of this effect, thalidomide
prescription and use are closely regulated by the manufacturer. Other adverse effects of
thalidomide include peripheral neuropathy, constipation, rash, fatigue, hypothyroidism, and
increased risk of deep vein thrombosis.
In more serious cases of Behcet syndrome, thalidomide (100 mg/d) is effective.
Ref: Lake D.F., Briggs A.D., Akporiaye E.T. (2012). Chapter 55. Immunopharmacology. In
B.G. Katzung, S.B. Masters, A.J. Trevor (Eds), Basic & Clinical Pharmacology, 12e.
Sample Previous Year Question on HIV based on previous Year Questions of
NEET PG, USMLE,PLAB,FMGE (MCI Screening). Please visit
www.medicoapps.org for more such Quizzes
All are drugs used against HIV, EXCEPT:
A: Elvucitabine
B:
Bevirima
t
C: Maraviroc
D: Tegaserod
Correct Ans:D
Explanation
New therapies are continually being sought for HIV that exploit new viral targets, have
activity against resistant viral strains, have a lower incidence of adverse effects, and offer
convenient dosing.
New agents of existing classes that are currently in advanced stages of clinical development
include the NRTI agent elvucitabine, the NNRTI agents TMC-278 and IDX899, the PI agent
bracanavir, entry inhibitors such as the CCR5 receptor antagonists vicriviroc and PRO 140,
and integrase inhibitors such as elvitegravir. In addition, new drug classes such as
maturation inhibitors (bevirimat) and the CD4 receptor inhibitor TNX-355 are under
investigation.
Ref: Katzung 11th edition Chapter 49.
Sample Previous Year Question on HIV based on previous Year Questions of
NEET PG, USMLE,PLAB,FMGE (MCI Screening). Please visit
www.medicoapps.org for more such Quizzes
Maraviroc is used in treatment of:
A: CMV
B:
HS
V
C: HIV
D: Measles
Correct Ans:C
Explanation
Maraviroc binds specifically and selectively to CCR5, one of two coreceptors necessary for
entrance of HIV into CD4+ cells, thus blocking entry of CCR5-tropic HIV into these cells.
Ref: Katzung 11th edition Chapter 49.
Sample Previous Year Question on HIV based on previous Year Questions of
NEET PG, USMLE,PLAB,FMGE (MCI Screening). Please visit
www.medicoapps.org for more such Quizzes
Lamivudine acts against which of the following viruses?
A: HIV
B: HBV
C: Both of the above
D: None of the above
Correct Ans:C
Explanation
Lamivudine inhibits HBV DNA polymerase and HIV reverse transcriptase by competing with
deoxycytidine triphosphate for incorporation into the viral DNA, resulting in chain
termination. Lamivudine achieves 3–4 log decreases in viral replication in most patients and
suppression of HBV DNA to undetectable levels in about 44% of patients.
Ref: Katzung 11th edition Chapter 49.
Sample Previous Year Question on HIV based on previous Year Questions of
NEET PG, USMLE,PLAB,FMGE (MCI Screening). Please visit
www.medicoapps.org for more such Quizzes
A 15 yr female with HIV infection on anti retroviral therapy presents with nausea vomiting
and flank pain. Urine microscopy showed rectangular plate and needle shaped crystals.
Serum creatine was raised. What will be the likely cause?
A:
Indinavi
r
B: Ritonavir
C: Nevirapine
D: Zidovudine
Correct Ans:A
Explanation
Crystal- induced nephropathy: Indinavir has been commonly associated with crystal-
induced nephropathy. Crystallization of the HIV protease inhibitor indinavir within the renal
tubules is associated with a range of urinary tract manifestations, including nephrolithiasis,
asymptomatic crystalluria, crystalluria with dysuria (or renal colic), and crystal
nephropathy, is influenced by alkaline urine. Other drus that may cause crystal nephropathy
are acyclovir, atazanavir and sulfazadine methotrexate
Ref: CURRENT Diagnosis & Treatment Nephrology & Hypertension - Page 125.
Sample Previous Year Question on HIV based on previous Year Questions of
NEET PG, USMLE,PLAB,FMGE (MCI Screening). Please visit
www.medicoapps.org for more such Quizzes
Metformin is used as a antihyperglycemic agent in DM, but there are some other uses for
this drug. All the following are such uses of the drug, except:
A: PCOD
B: Non-alcoholic fatty liver disease
C: Alcoholic fatty liver disease
D: Metabolic abnormalities associated with HIV disease
Correct Ans:C
Explanation
Other uses of metformin:
• PCOD
• Impaired glucose tolerance
• Obesity
• Metabolic abnormalities associated with HIV disease
• Non alcoholic fatty liver disease
Ref: Harrison, 18th Edition, Page 2995
Sample Previous Year Question on HIV based on previous Year Questions of
NEET PG, USMLE,PLAB,FMGE (MCI Screening). Please visit
www.medicoapps.org for more such Quizzes
Thalidomide is used in all, EXCEPT:
A: Erythema nodosum leprosum
B:
HIV
neuropathy
C: HIV associated ulcer
D: Multiple myeloma
Correct Ans:B
Explanation
Thalidomide is indicated for the treatment of patients with erythema nodosum leprosum
and multiple myeloma. In addition, it has orphan drug status for mycobacterial infections,
Crohn's disease, HIV-associated wasting, Kaposi sarcoma, lupus, myelofibrosis, brain
malignancies, leprosy, graft-versus-host disease, and aphthous ulcers.
Also Know:
Thalidomide should never be taken by women who are pregnant or who could become
pregnant while taking the drug.It is best known for the severe, life-threatening birth
defects it caused when administered to pregnant women.
Sample Previous Year Question on HIV based on previous Year Questions of
NEET PG, USMLE,PLAB,FMGE (MCI Screening). Please visit
www.medicoapps.org for more such Quizzes
The important coreceptors for HIV to bind with CD4 receptors are:
A: CCR4 and CXCR3
B:
CCR5 and
CXCR4
C: CCR4 and CXCR5
D: CCR4 and CXCR2
Correct Ans:B
Explanation
The primary cellular receptor for HIV is CD4 (Helper subset of T cells). HIV has two major
coreceptors: CCR5 and CXCR4 (for fusion and entry)
ALSO KNOW:
Kaposi's sarcoma cannot be explained completely by the immunodeficiency caused by HIV
infection.
Ref: Harrison, Edition-18, Page-1519.
Sample Previous Year Question on HIV based on previous Year Questions of
NEET PG, USMLE,PLAB,FMGE (MCI Screening). Please visit
www.medicoapps.org for more such Quizzes
Neurologic abnormalities have been noted in about one-third of patients with AIDS. Which
of the following is NOT seen in HIV involvement of CNS?
A: Perivascular giant cell
B: Vacuolar degeneration of post column
C: Microglial nodule formation
D: Inclusion bodies
Correct Ans:D
Explanation
The main cell types that are infected in the brain in HIV infection are the perivascular
macrophages and the microglial cells. A diffuse and multifocal rarefaction of the cerebral
white matter accompanied by scanty perivascular infiltrates of lymphocytes and clusters of
a few foamy macrophages, microglial nodules, and multinucleated giant cells. Vacuolar
degeneration of posterior column is associated with the AIDS dementia complex.
Ref: Ropper A.H., Samuels M.A. (2009). Chapter 33. Viral Infections of the Nervous System,
Chronic Meningitis, and Prion Diseases. In A.H. Ropper, M.A. Samuels (Eds), Adams and
Victor's Principles of Neurology, 9e.
Sample Previous Year Question on HIV based on previous Year Questions of
NEET PG, USMLE,PLAB,FMGE (MCI Screening). Please visit
www.medicoapps.org for more such Quizzes
Which of the following disease cause modification of finger prints?
A: Leprosy
B:
HI
V
C: Hypertension
D: Acromegaly
Correct Ans:A
Explanation
Leprosy can modify finger prints. Electrical injury and radiation can also modify finger
prints.
In criminal cases impression of all 10 fingers are taken but in civil cases left thumb
impression is taken in case of males and right thumb impression in case of females.
Dactylography or finger print system consist of taking the impression of the pulp of the
fingers and thumb with printers ink on an unglazed white paper. The fingerprints of even
identical twins are different. The chances of two persons having identical finger prints is
about one in thirty times the population of the world.
Ref: Concise Textbook Of Forensic Medicine & Toxicology By Sharma, page 19.
Sample Previous Year Question on HIV based on previous Year Questions of
NEET PG, USMLE,PLAB,FMGE (MCI Screening). Please visit
www.medicoapps.org for more such Quizzes
Identify the CORRECT description of Molotov cocktail:
A: Petrol bomb
B: Mixture of alcohol and Cocaine
C: Combination of drugs used for HIV treatment
D: Combination of drugs used by Cocaine addicts
Correct Ans:A
Explanation
A Molotov cocktail is also known as a petrol bomb or alcohol bomb. It is a simple
type of improvised incendiary device. The simplest form consists of a stoppered
bottle filled with a combustible liquid, such as gasoline or high-proof alcohol,
with a fuel-soaked rag stuffed in the neck of the bottle.
Sample Previous Year Question on HIV based on previous Year Questions of
NEET PG, USMLE,PLAB,FMGE (MCI Screening). Please visit
www.medicoapps.org for more such Quizzes
Sentinel laboratories are mainly involved in:
A: External Quality Assessment of laboratories
B:
Diagnosis of HIV
infection
C: Control of Bioterrorism
D: Diagnosis of Tuberculosis
Correct Ans:C
Explanation
Because communication between clinical and public health laboratories is so
critical in the post-2001 era, Centre for Disease Control (CDC) created an
organizational structure whereby isolates and lines of communication flow freely
among numbers. The Laboratory Response Network (LRN) was originally
designed as a four-tier system. Clinical laboratories (Level A) were at the bottom
of a triangle, sending specimens to a public health laboratory (level B) or Level C
(state public health laboratory) for confirmation. Isolates were ultimately sent to
CDC or United States Army Medical Research Institute for Infectious Diseases
(USMRIID)-Level D, for archiving ang sophisticated molecular testing. This
original system has now been changed to a three-tier system in which Level A
laboratories are now called Sentinel laboratories; Level B & C laboratories are
called Reference laboratories & Level D are called National laboratories.
The main role of Sentinel Microbiology laboratories is to raise suspicion when a
targeted agent is suspected in a human specimen. Detection of a possible
bioterrorism event will depend on:
• A laboratory having an active microbial surveillance and
monitoring program
• Vigilant technologists looking for a disease that does not occur
naturally in a particular geographic region (eg: Plague in New
York city); is transmitted by an aerosol route of infection; is a
single case of disease caused by an unusual agent (eg:
Burkholderia mallei usually only seen in the far east); Good
communication with infection control practitioners, infectious
disease Physicians and local or regional public health laboratories.
Ref: Bailey & Scott’s Diagnostic Microbiology; 12th edition; Page: 954-955
Sample Previous Year Question on HIV based on previous Year Questions of
NEET PG, USMLE,PLAB,FMGE (MCI Screening). Please visit
www.medicoapps.org for more such Quizzes
One of the most common causes of diarrhea in HIV infected patients is
cryptosporidiosis. Which of the following is the method used for diagnosis of
cryptosporidiosis in fecal matter?
A: Ziehl-Neelsen stain of the stool
B:
Gram stain of the
stool
C: Normal saline suspension of stool
D: Iodine suspension of stool
Correct Ans:A
Explanation
Cryptosporidiosis is a protozoan which is most commonly involved in the
pathogenesis of diarrhea in HIV infected patients. The cysts for cryptosporidiosis
are best demonstrated by Ziehl-Neelsen stain of fecal mater.
Other important clinical laboratory tests for stool are summarized as follows.
Analysis of Stool:
1. Physical examination for color, frank blood, undigested food, worms etc.
2. Chemical examination of Reducing substance for lactose intolerance, of Fecal
fat for malabsorption syndrome, of Protein content for Protein losing
enteropathy. Occult blood test.
3. Direct light microscopy for demonstration of RBC, WBC,Crystals and Ova of
different types of helminths. Demonstration of protozoan like E. histolytica and
Giardia.
4. Culture of stool for Salmonella, Shigella, Campylobacter, etc.
5. Assay for demonstration toxins in stool in cases of patients suffering from
Pseudomembranous colitis (toxin of Cl. difficile).
6. ELISA- This test it is able to find viral antigens (Rotavirus) as a causative
agent for infantile diarrhea. 7. Electron microscopy for demonstration of virus
(Rotavirus).
8. Gram/Z-N stain. Z-N staining of stool smear is highly diagnostic for
demonstration of Cryptosporidiosis (protozoa) responsible for diarrhea in an
immunocompromised host (AIDS).
Ref: Brooks G.F. (2013). Chapter 46. Medical Parasitology. In G.F. Brooks
(Ed),Jawetz, Melnick, & Adelberg's Medical Microbiology, 26e.
Sample Previous Year Question on HIV based on previous Year Questions of
NEET PG, USMLE,PLAB,FMGE (MCI Screening). Please visit
www.medicoapps.org for more such Quizzes
Which of the following causes highest risk of nosocomial infection to a patient?
A: Patient admitted for elective surgery
B: HIV patient coming in follow up OPD
C: Patient undergoing endoscopy
D: Patient admitted for normal delivery
Correct Ans:A
Explanation
The fact that at least 25–50% of nosocomial infections are due to the combined
effect of the patient's own flora and invasive devices highlights the importance of
improvements in the use and design of such devices. Urinary tract infections
(UTIs) account for ~34% of nosocomial infections. Pneumonia accounts for
~13% of nosocomial infections. Wound infections account for ~17% of
nosocomial infections
UTI is the most common cause of nosocomial infection. Other common cause is
pneumonia and surgical procedures. Contaminated surgical equipments and
from health care workers can be the cause of infection in surgical wound.
Also Know:
The most common pathogens in postoperative wound infections are S. aureus,
coagulase-negative staphylococci, and enteric and anaerobic bacteria. In rapidly
progressing postoperative infections, which manifest within 24–48 h of a surgical
procedure, the level of suspicion regarding group A streptococcal or clostridial
infection should be high.
Ref: Weinstein R.A. (2012). Chapter 131. Health Care–Associated Infections. In
D.L. Longo, A.S. Fauci, D.L. Kasper, S.L. Hauser, J.L. Jameson, J. Loscalzo
(Eds), Harrison's Principles of Internal Medicine, 18e.
Sample Previous Year Question on HIV based on previous Year Questions of
NEET PG, USMLE,PLAB,FMGE (MCI Screening). Please visit
www.medicoapps.org for more such Quizzes
FALSE statement ragarding HIV-associated TB is:
A:
Extrapulmonary TB is common among HIV-infected
patients
B: The diagnosis of TB in HIV-infected patients may be difficult
C:
Immune reconstitution inflammatory syndrome (IRIS) is more common among patients with
advanced immunosuppression and extra pulmonary TB
D:
Patients with both HIV infection and TB are more infectious than persons without HIV co-
infection
Correct Ans:D
Explanation
The most infectious patients have cavitary pulmonary disease. Patients with
sputum smear–negative and those with culture-negative pulmonary TB and
extrapulmonary TB are essentially noninfectious. Because persons with both HIV
infection and TB are less likely to have cavitations, and they are less likely to be
smear positive they may be less infectious than persons without HIV co-infection.
Ref: Harrisons principles of internal medicine, 18th edition, Page: 1342.
Sample Previous Year Question on HIV based on previous Year Questions of
NEET PG, USMLE,PLAB,FMGE (MCI Screening). Please visit
www.medicoapps.org for more such Quizzes
Regarding HIV infection, not true is:
A: p24 is used for early diagnosis
B: Lysis of infected CD4 cells is seen
C: Dendritic cells do not support replication
D: Macrophage is a reservoir for the virus
Correct Ans:C
Explanation
Follicular dendritic cells from tonsils, can be infected with HIV even without the
involvement of CD4.
Ref: Text Book of Microbilogy By Ananthanarayan, 6th Edition, Page 546
Sample Previous Year Question on HIV based on previous Year Questions of
NEET PG, USMLE,PLAB,FMGE (MCI Screening). Please visit
www.medicoapps.org for more such Quizzes
'H5 Nl' may be best described as a:
A: Bird flu virus
B: Vaccine for HIV
C: Agent for Japanese encephalitis
D: New strain of Plasmodium falciparum
Correct Ans:A
Explanation
H5N1 is the Avian Flue a type of Influenza A virus, also called the Bird Flu
Virus.
Ref: Molecular Medicine: An Introductory Text By Trent, 3rd Edition, Pages
209-10; Clinical Infectious Disease By Schlossberg, 2008, Page 1290
Sample Previous Year Question on HIV based on previous Year Questions of
NEET PG, USMLE,PLAB,FMGE (MCI Screening). Please visit
www.medicoapps.org for more such Quizzes
The genome of HIV virus contains which of the following?
A: Single stranded DNA
B:
Single stranded
RNA
C: Double stranded DNA
D: Double stranded RNA
Correct Ans:B
Explanation
HIV is an enveloped icosahedral sphere. It has 2 identical, non complementary
strands of RNA and 3 enzymes (reverse transcriptase, integrase and protease)
packed in a cone shaped protein core. This core is surrounded by a protein coat
called capsid. This capsid along with the enclosed nucleic acid is called
nucleocapsid. Capsid acts as a protective shell around the nucleic acid core and
helps to introduce the viral genome into host cell by adsorbing readily to host cell
surfaces.
Most RNA viruses are single stranded except reoviridae which is double
stranded. Most DNA virus are double stranded except poxviridae which is single
stranded.
Ref: HIV and AIDS: Basic Elements and Priorities By S. Karthikeyan, Pages 41-
3
Sample Previous Year Question on HIV based on previous Year Questions of
NEET PG, USMLE,PLAB,FMGE (MCI Screening). Please visit
www.medicoapps.org for more such Quizzes
Regarding HIV which of the following statement is not TRUE?
A: It is a DNA retrovirus
B: Contains Reverse Transcriptase
C: May infect host CD4 cells other than T-lymphocytes
D: Causes a reduction in host CD4 cells at late stage of disease
Correct Ans:A
Explanation
The genome of HIV is diploid, composed of 2 identical single stranded positive
sense RNA copies. In association with viral RNA is the reverse transcriptase
enzyme which is the characteristic feature of retroviruses.
Pathogenesis:
When virus infects a cell RNA, viral RNA is transcribed by the enzyme, first to
single stranded DNA and then to double stranded DNA which is integrated into
the host cell chromosome. The provirus initiate viral replication by directing
synthesis of viral RNA and other components.
When naked virus buds outs through host cell surface membrane, it acquires a
lipoprotein envelope which consist of lipid derived from the host cell membrane
and glycoprotein which are virus coded. The spikes which constitute the major
surface component of the virus, binds to CD4 receptors on susceptible host cells.
The transmembrane pedicle cause cell fusion.
Ref: Ananthanarayan and Panicker’s Textbook of Microbiology, 8th Edition,
Page 395
Sample Previous Year Question on HIV based on previous Year Questions of
NEET PG, USMLE,PLAB,FMGE (MCI Screening). Please visit
www.medicoapps.org for more such Quizzes
CMV retinitis in HIV occurs when the CD4 counts fall below which of the
following levels?
A: 50
B: 100
C: 200
D: 150
Correct Ans:A
Explanation
CMV Retinitis is the most common cause of loss of vision in AIDS patients.
Majority of cases of CMV retinitis occur in patients with a CD4+ T cell count
<50/micro L. Therefore, patients at high risk of CMV retinitis (CD4+ T cell
count <100/micro L) should undergo an ophthalmologic examination every 3–6
months.
CMV retinitis usually presents as a painless, progressive loss of vision. Patients
may also complain of blurred vision, floaters and scintillations. It is usually
bilateral. The characteristic retinal appearance is presence of perivascular
hemorrhage and exudate. Treatment consists of oral valacyclovir, ganciclovir, IV
ganciclovir, or IV foscarnet, with cidofovir as an alternative.
Ref: Textbook of Microbiology By Ananthanarayan and Panicker, 8th Edition,
Page 573 ; Harrison’s Principles of Internal Medicine, 18th Edition, Chapter 189
Sample Previous Year Question on HIV based on previous Year Questions of
NEET PG, USMLE,PLAB,FMGE (MCI Screening). Please visit
www.medicoapps.org for more such Quizzes
Most common genital lesion in HIV patient is:
A: Chlamydia
B:
Herpe
s
C: Syphilis
D: Candida
Correct Ans:B
Explanation
Herpes simplex virus is the is the most common opportunistic infection causing
genital lesions in HIV patients. Herpes simplex virus type I and Type II cause
recurrent oral, labial, genital and perianal herpes simplex lesions in HIV infected
individuals. Herpes simplex virus type II (HSV2) is associated with genital
lesions.
Ref: Diagnosis and Management of HIV/AIDS : A Clinician's Perspective By
Usha K. Baveja, B. B. Rewari, Page 93; Centers for Disease Control and
Prevention. 2006 Sexually Transmitted Diseases Treatment Guidelines. MMWR
Recomm Rep 2006; 55(RR-11), Pages 1 - 94.
Sample Previous Year Question on HIV based on previous Year Questions of
NEET PG, USMLE,PLAB,FMGE (MCI Screening). Please visit
www.medicoapps.org for more such Quizzes
Which of the following has the highest chance of transmission of HIV?
A: Anal intercourse
B: Transfusion of blood products
C: Kidney transplant
D: Transplacental transmission
Correct Ans:B
Explanation
The approximate chance of infection/exposure is given below of various scenario
a) Sexual intercourse i.e oral, anal or vaginal : 0.1 – 1.0 %
b) Blood and blood product transfusion : >90%
c) Tissue and organ donation : 5 to 90 %
d) Sharing needles by drug addicts : 0.5 – 1.0 %
e) Mother to baby : 30 %
Ref: Textbook of microbiology By Ananthanarayanan, 6th Edition, Page 551
Sample Previous Year Question on HIV based on previous Year Questions of
NEET PG, USMLE,PLAB,FMGE (MCI Screening). Please visit
www.medicoapps.org for more such Quizzes
A 28-year-old HIV-positive male complains of pain on swallowing. Physical
examination is remarkable for white plaque-like material on his tongue and
buccal mucosa, which is scraped and sent to the laboratory. Based on these
findings, and on the laboratory results, the man is diagnosed with acquired
immunodeficiency syndrome (AIDS). With which of the following agents is the
man most likely infected?
A: Candida albicans
B:
Cytomegaloviru
s
C: Herpes simplex I
D: Human herpesvirus 8
Correct Ans:A
Explanation
Candida albicans produces oral thrush, an AIDS-defining lesion, which is
common in acute HIV disease, and becomes increasing common as the CD4 + cell
count falls. The lesions are usually painless. Diagnosis is by demonstration of
pseudohyphae using a wet smear with confirmation by culture.
Although cytomegalovirus is associated with numerous clinical scenarios in the
AIDS population, including odynophagia (painful swallowing), it would not
produce white plaques on the oral mucosa.
Herpes Simplex Iproduces vesicular lesions occurring in clusters in the oral
cavity. There is an increased risk of herpes infections in the AIDS group, but the
lesions do not resemble those described in the question.
Human herpesvirus 8 is the causative agent of Kaposi's sarcoma, a malignancy
arising from endothelial cells that appears as hemorrhagic nodules in different
organ systems. It is the most common cancer in the HIV infected population.
Sample Previous Year Question on HIV based on previous Year Questions of
NEET PG, USMLE,PLAB,FMGE (MCI Screening). Please visit
www.medicoapps.org for more such Quizzes
Which of the following is the MOST common cause of sporadic viral encephalitis
in adults?
A: Herpes virus
B: Enteroviruses
C: Ebstein-Barr virus
D: HIV
Correct Ans:A
Explanation
The most commonly identified viruses causing sporadic cases of acute
encephalitis in adults are herpesviruses (HSV, VZV, EBV).
Note Frequently Asked: Enteroviruses (coxsackieviruses, echoviruses, and
human enteroviruses 68–71) are the most commonly identified viruses causing
sporadic cases of acute meningitis.
Ref: Roos K.L., Tyler K.L. (2012). Chapter 381. Meningitis, Encephalitis, Brain
Abscess, and Empyema. In D.L. Longo, A.S. Fauci, D.L. Kasper, S.L. Hauser,
J.L. Jameson, J. Loscalzo (Eds), Harrison's Principles of Internal Medicine, 18e
Sample Previous Year Question on HIV based on previous Year Questions of
NEET PG, USMLE,PLAB,FMGE (MCI Screening). Please visit
www.medicoapps.org for more such Quizzes
HIV virus has a particularly high affinity for which of the following?
A: CD4 T Lymphocytes and B cells
B: CD4 T Lymphocytes and monocytes
C: CD8 T Lymphocytes and B cells
D: CD8 T Lymphocytes and CD4 T Lymphocytes
Correct Ans:B
Explanation
HIV has a particular tendency to bind to CD4 T lymphocytes (helper T
lymphocytes) and monocytes and then become internalized. Most other cells,
include CD8 T lymphocytes (suppressor T cells) and B cells, are not particularly
susceptible to the HIV virus.
HIV preferentially infects and kills helper (CD4) T lymphocytes, resulting in the
loss of cell-mediated immunity and a high probability that the host will
developopportunistic infections. Other cells (e.g., macrophages and monocytes)
that have CD4 proteins on their surfaces can be infected also.
Also know:
The genome of HIV consists of two identical molecules of single-stranded,
positive-polarity RNA and is said to be diploid.
In addition to the three typical retroviral genes gag, pol, and env, which encode
the structural proteins, the genome RNA has six regulatory genes. Two of these
regulatory genes, tat and rev, are required for replication, and the other
four, nef, vif, vpr, and vpu, are not required for replication and are termed
“accessory" genes.
Ref: Levinson W. (2012). Chapter 45. Human Immunodeficiency Virus. In W.
Levinson (Ed), Review of Medical Microbiology & Immunology, 12e.
Sample Previous Year Question on HIV based on previous Year Questions of
NEET PG, USMLE,PLAB,FMGE (MCI Screening). Please visit
www.medicoapps.org for more such Quizzes
What is the chance of HIV infection after needle prick injury?
A: 1/100
B:
1/30
0
C: 1/10000
D: 1 in 1 Lakh
Correct Ans:B
Explanation
Needle prick injury: HIV transmission following skin puncture from a
contaminated needle is 0.3 % and a mucous membrane exposure is 0.09%. The
seroconversion rate for HIV is 1:300, or 0.3% from a hollow-bore needle stick
injury. Antiretroviral drugs as postexposure prophylaxis decreases the risk of
infection.
Must know:
In Hepatitis B, the seroconversion ranges from 23 to 62%, with higher rates
associated with patients who are HBeAg positive, as compared to HBeAG
negative. The seroconversion rate for Hepatitis C is 1.8%.For Hepatitis C no
prophylactic treatment is available. HBV post exposure prophlylaxis with
hepatitis B immunoglobulin and initiation of Hep B vaccine is 90%effective.
Ref: Harrison’s Principles of internal medicine, 16th
edition, page 1081.
Sample Previous Year Question on HIV based on previous Year Questions of
NEET PG, USMLE,PLAB,FMGE (MCI Screening). Please visit
www.medicoapps.org for more such Quizzes
A 24-year-old man is infected with HIV during a sexual encounter with a
prostitute. He does not notice any symptoms in the period immediately after the
encounter. During the asymptomatic latent phase of his infection, the virus is
actively proliferating, and can be found in association with which of the
following?
A: B lymphocytes
B: Follicular dendritic cells in lymph nodes
C: Ganglion cells
D: Oligodendrocytes
Correct Ans:B
Explanation
Follicular dendritic cells in the germinal centers of lymph nodes are important
reservoirs of HIV. Although some follicular dendritic cells are infected with HIV,
most viral particles are found on the surface of their dendritic processes.
Follicular dendritic cells have receptors to the Fc portion of immunoglobulins
that serve to trap HIV virions coated with anti-HIV antibodies. These coated
HIV particles retain the ability to infect CD4+ T cells as they traverse the
dendritic cells.
B lymphocytes have a surface marker (CD21 protein-a complement receptor) to
which an Epstein-Barr envelope glycoprotein can bind. The virus associates with
the host cell genome, producing a latent infection. These B cells undergo
polyclonal activation and proliferation.
Ganglion cells, particularly the satellite cells around the ganglion cells in the
dorsal root ganglia, can be infected by varicella-zoster. Herpes type I and II
infect neurons that innervate skin and mucous membranes.
Oligodendrocytes are directly infected by two viruses, JC virus (a polyomavirus)
and measles virus. JC virus causes progressive multifocal leukoencephalopathy
(PML), and measles virus produces a latent syndrome called subacute sclerosing
panencephalitis (SSPE).
Ref: Levinson W. (2012). Chapter 58. Cellular Basis of the Immune Response. In
W. Levinson (Ed), Review of Medical Microbiology & Immunology, 12e.
Sample Previous Year Question on HIV based on previous Year Questions of
NEET PG, USMLE,PLAB,FMGE (MCI Screening). Please visit
www.medicoapps.org for more such Quizzes
During pre-operative investigations, a man found to have positive serological
results for HIV. Which of the following is highly specific test for HIV antibodies
in this patient?
A: ELISA
B: Southern blot
C: Northern blot
D:
Western
blot
Correct Ans:D
Explanation
The Western blot test is used as a measure of specific HIV-1 antibodies to confirm a
positive ELISA result. The criterion for a positive test is any two bands
corresponding to p24, gp41, and gp120/160. The absence of bands is a negative
result, whereas the presence of bands that do not meet the criterion for a positive
test is an indeterminate result. False-positive and false-negative results are
relatively uncommon.
HIV-2-infected patients may give an unusual pattern on an HIV-1 Western blot;
therefore, if HIV-2 is suspected, a separate HIV-2 Western blot is required.
Ref: Brooks G.F., Carroll K.C., Butel J.S., Morse S.A., Mietzner T.A. (2013).
Chapter 47. Principles of Diagnostic Medical Microbiology. In G.F. Brooks, K.C.
Carroll, J.S. Butel, S.A. Morse, T.A. Mietzner (Eds), Jawetz, Melnick, &
Adelberg's Medical Microbiology, 26e.
Sample Previous Year Question on HIV based on previous Year Questions of
NEET PG, USMLE,PLAB,FMGE (MCI Screening). Please visit
www.medicoapps.org for more such Quizzes
A 28-year-old HIV-positive male complains of pain on swallowing. Physical
examination is remarkable for white plaque-like material on his tongue and
buccal mucosa, which is scraped and sent to the laboratory. Based on these
findings, and on the laboratory results, the man is diagnosed with acquired
immunodeficiency syndrome (AIDS). With which of the following agents is the
man most likely infected?
A: Candida albicans
B:
Cytomegaloviru
s
C: Herpes simplex I
D: Human herpesvirus 8
Correct Ans:A
Explanation
Candida albicans produces oral thrush, an AIDS-defining lesion, which is
common in acute HIV disease, and becomes increasing common as the CD4 + cell
count falls. The lesions are usually painless. Diagnosis is by demonstration of
pseudohyphae using a wet smear with confirmation by culture.
Although cytomegalovirus is associated with numerous clinical scenarios in the
AIDS population, including odynophagia (painful swallowing), it would not
produce white plaques on the oral mucosa.
Herpes Simplex Iproduces vesicular lesions occurring in clusters in the oral
cavity. There is an increased risk of herpes infections in the AIDS group, but the
lesions do not resemble those described in the question.
Human herpesvirus 8 is the causative agent of Kaposi's sarcoma, a malignancy
arising from endothelial cells that appears as hemorrhagic nodules in different
organ systems. It is the most common cancer in the HIV infected population.
Sample Previous Year Question on HIV based on previous Year Questions of
NEET PG, USMLE,PLAB,FMGE (MCI Screening). Please visit
www.medicoapps.org for more such Quizzes
With needle stick injury, there is risk of transmitting all of the following
infections, EXCEPT:
A: HIV
B: HBV
C: HCV
D: HDV
Correct Ans:D
Explanation
Most common infections spread through needle stick injury are HIV, HBV and
HCV. HDV infection occur only in the presence of HBV. It is transmitted
through parenteral exposure or intimate contact. Vertical HDV transmission is
rare.
• HBV is the most infectious of the 3 viruses.
• There is about 2% risk of acquiring HCV infection after needle
stick injury. The is no vaccine against HCV and no treatment can
prevent infection after exposure to the virus.
• The risk of acquiring HIV is less than 0.5% after a single needle
stick injury involving a patient known to have HIV/AIDS in a
healthcare setting.
• The risk of HIV transmission from an infected patients blood or
other body fluid splashed into a caregivers eyes or mouth is much
lower than for needle stick injuries.
Ref: Levin M.J., Weinberg A. (2012). Chapter 40. Infections: Viral & Rickettsial.
In W.W. Hay, Jr., M.J. Levin, R.R. Deterding, J.J. Ross, J.M. Sondheimer
(Eds),CURRENT Diagnosis & Treatment: Pediatrics, 21e, Sokol R.J., Narkewicz
M.R. (2012). Chapter 22. Liver & Pancreas. In W.W. Hay, Jr., M.J. Levin, R.R.
Deterding, J.J. Ross, J.M. Sondheimer (Eds),CURRENT Diagnosis & Treatment:
Pediatrics, 21e.
Sample Previous Year Question on HIV based on previous Year Questions of
NEET PG, USMLE,PLAB,FMGE (MCI Screening). Please visit
www.medicoapps.org for more such Quizzes
What are the chances of contracting HIV after needle prick injury?
A: 1/100
B:
1/30
0
C: 1/10000
D: I in 1 Lac
Correct Ans:B
Explanation
The average risk of HIV infection following needle prick injury is 1/300. After
mucocutaneous exposure risk is approximately 0.09%. Post exposure
prophylaxis should be initiated within 1-2 hrs.
According CDC guidelines:
• A basic regimen consists of two-drug therapy, often consisting of
azidothymidine and lamivudine.
• In an expanded regimen: indinavir or nelfinavir is added to it.
• HBV is the most infectious of the 3 viruses.
• There is about 2% risk of acquiring HCV infection after needle
stick injury. The is no vaccine against HCV and no treatment can
prevent infection after exposure to the virus.
• The risk of HIV transmission from an infected patients blood or
other body fluid splashed into a caregivers eyes or mouth is much
lower than for needle stick injuries.
Ref: Rothman R.E., Marco C.A., Yang S. (2011). Chapter 149. Human
Immunodeficiency Virus Infection and Acquired Immunodeficiency Syndrome.
In R.K. Cydulka, G.D. Meckler (Eds), Tintinalli's Emergency Medicine: A
Comprehensive Study Guide, 7e.
Sample Previous Year Question on HIV based on previous Year Questions of
NEET PG, USMLE,PLAB,FMGE (MCI Screening). Please visit
www.medicoapps.org for more such Quizzes
Primary Receptor for Human Immunodeficiency Virus (HIV) is:
A: CD4
B: CD
8
C: CD3
D: CD56
Correct Ans:A
Explanation
HIV targets cells that express CD4, and can infect macrophages, dendritic cells
(both groups express CD4 at low levels) and CD4+ T cells.
Ref: Andrew E. Williams, “Immunology of Urogenital Tract and Conjunctiva”,
In the book, “Immunology: Mucosal and Body Surface Defences”, USA, Wiley
Blackwell Publications, Chapter 9.1; Harrison’s Principles of Internal Medicine,
17th Edition, Pages 1139, 1157; Textbook of Microbiology By Ananthnaraynan,
7th Edition, Page 586
Sample Previous Year Question on HIV based on previous Year Questions of
NEET PG, USMLE,PLAB,FMGE (MCI Screening). Please visit
www.medicoapps.org for more such Quizzes
Most common genital lesion in HIV patient is:
A: Chlamydia
B:
Herpe
s
C: Syphilis
D: Candida
Correct Ans:B
Explanation
Herpes simplex virus is the is the most common opportunistic infection causing
genital lesions in HIV patients. Herpes simplex virus type I and Type II cause
recurrent oral, labial, genital and perianal herpes simplex lesions in HIV infected
individuals. Herpes simplex virus type II (HSV2) is associated with genital
lesions.
Ref: Diagnosis and Management of HIV/AIDS : A Clinician's Perspective By
Usha K. Baveja, B. B. Rewari, Page 93; Centers for Disease Control and
Prevention. 2006 Sexually Transmitted Diseases Treatment Guidelines. MMWR
Recomm Rep 2006; 55(RR-11), Pages 1 - 94.
Sample Previous Year Question on HIV based on previous Year Questions of
NEET PG, USMLE,PLAB,FMGE (MCI Screening). Please visit
www.medicoapps.org for more such Quizzes
A 38-year-old man has been HIV-positive since the age of 26 but has refused
antiretroviral prophylaxis. His CD4 cell count, however, is still within normal
limits, and the virus is undetectable in the blood. Genetic investigations clarify
this apparent puzzle, demonstrating that this subject is homozygous for a
mutation affecting a chemokine receptor necessary for HIV entry into the cell.
Which of the following receptors is most likely involved?
A: CCR5
B:
CD
4
C: CD8
D: Gp120
Correct Ans:A
Explanation
HIV infection does not always progress to overt AIDS. In a few fortunate
individuals (called non-progressors), the infection remains asymptomatic, with a
stable CD4 cell count and low or undetectable viremia. It is not entirely clear
whether these patients will eventually develop AIDS. Some are resistant to HIV
infection because they carry two defective copies of the gene encoding the CCR5
coreceptor. There are 15 known coreceptors, of which CCR5 and CXCR4 are the
best characterized. CCR5 is a receptor for β-chemokines and serves as a
coreceptor for HIV internalization. Approximately one percent of Caucasians in
the U.S. is homozygous for such a protective CCR5 variant, and 20% are
heterozygotes.
HIV infection follows a slower course in heterozygotes. Homozygotes for this
protective CCR5 variant are not present among Black and Asian populations.
Recently, a variant haplotype of CXCR1 (a leukocyte receptor for fractalkine)
has been linked to rapid progression of HIV infection in homozygous individuals.
Thus, investigations on HIV coreceptors are beginning to explain the variability
in progression and severity of HIV infection among individuals.
CD4 (2nd Choice) is an important receptor of helper T lymphocytes,
macrophages, and Langerhans cells. It acts as the primary receptor for HIV
binding. Binding to CD4, however, is not sufficient for the entry of HIV into the
cells.
The viral envelope gp120 protein (4th Choice) first binds CD4. This results in a
conformational change in gp120 that allows this protein to bind coreceptors.
Such coreceptors are present on lymphocytes and macrophages and normally
serve as binding sites for chemokines. Once gp120 binds to CD4 and a
coreceptor, a second viral protein, gp41, facilitates fusion of the viral envelope
with the plasma membrane, allowing "injection" of the viral core into the cell.
Neither CD8 (5th Choice) nor the receptor for TNF-α plays a role in HIV binding
to cells.
Sample Previous Year Question on HIV based on previous Year Questions of
NEET PG, USMLE,PLAB,FMGE (MCI Screening). Please visit
www.medicoapps.org for more such Quizzes
A 24-year-old sex worker from Mumbai develops chronic abdominal pain, low-
grade fever, diarrhea, and malabsorption. He saw later found to be positive for
HIV. Oocysts are demonstrated in the stool. Which of the following organisms is
most likely to be the cause of the patient's diarrhea?
A: Diphyllobothrium latum
B:
Entamoeba
histolytica
C: Giardia lamblia
D: Isospora belli
Correct Ans:D
Explanation
All of the organisms listed are protozoa. There are two intestinal protozoa
specifically associated with AIDS that can cause transient diarrhea in
immunocompetent individuals but can cause debilitating, and potentially life-
threatening chronic diarrhea in AIDS patients. These organisms are Isospora
belli, treated with trimethoprim-sulfamethoxazole (or other folate antagonists)
and Cryptosporidium parvum.
Diphyllobothrium latum is the fish tapeworm and occasionally causes diarrhea.
Entamoeba histolytica and Giardia lamblia are both causes of diarrhea, but they
are not specifically associated with AIDS.
Sample Previous Year Question on HIV based on previous Year Questions of
NEET PG, USMLE,PLAB,FMGE (MCI Screening). Please visit
www.medicoapps.org for more such Quizzes
HIV patient is admitted with malabsorption, fever, chronic diarrhoea and acid
fast positive organism. What is the likely causative agent?
A: Giardia
B: Microsporidia
C: Isospora
D: E. histolytica
Correct Ans:C
Explanation
The coccidian parasite Isospora belli causes human intestinal disease. Infection is
acquired by the consumption of oocysts, after which the parasite invades
intestinal epithelial cells and undergoes both sexual and asexual cycles of
development. Oocysts excreted in stool are not immediately infectious but must
undergo further maturation.
In patients who have AIDS, infections often are not self-limited but rather resemble
cryptosporidiosis, with chronic, profuse watery diarrhea. Eosinophilia, which is not
found in other enteric protozoan infections, may be detectable.
The diagnosis is usually made by detection of the large (25-m) oocysts in stool by
modified acid-fast staining. Oocyst excretion may be low-level and intermittent;
if repeated stool examinations are unrevealing, sampling of duodenal contents by
aspiration or small-bowel biopsy (often with electron-microscopic examination)
may be necessary.
Ref: Weller P.F. (2012). Chapter 215. Protozoal Intestinal Infections and
Trichomoniasis. In D.L. Longo, A.S. Fauci, D.L. Kasper, S.L. Hauser, J.L.
Jameson, J. Loscalzo (Eds), Harrison's Principles of Internal Medicine, 18e
Sample Previous Year Question on HIV based on previous Year Questions of
NEET PG, USMLE,PLAB,FMGE (MCI Screening). Please visit
www.medicoapps.org for more such Quizzes
A 31 year old HIV-positive man develops a severe pneumonia. Lower respiratory
tract secretions obtained by fiberoptic bronchoscopy with bronchoalveolar
lavage and stained with methenamine silver stain demonstrate cup-shaped cysts
with sharply outlined walls. Which of the following organisms is the most likely
pathogen in this case?
A: Candida albicans
B:
Giardia
lamblia
C: Haemophilus influenzae
D: Pneumocystis carinii
Correct Ans:D
Explanation
The organism described is Pneumocystis carinii, which is an opportunistic
parasite that appears to be more closely related to fungi than to protozoa. Its cyst
form, when stained with silver stains, has the distinctive appearance described in
the question stem, and is typically found in frothy material that occupies the
lumen of alveoli. The trophozoites are smaller and much harder to recognize.
Bronchoalveolar lavage is considered much more reliable than induced sputum
as a diagnostic specimen.
Pneumocystis pneumonia is a common infection among AIDS patients, and is
very uncommon in other clinical settings. Formerly, many AIDS patients died
with Pneumocystis pneumonia, but the combination of early drug treatment
(with trimethoprim/sulfamethoxazole or pentamidine) and prophylaxis (usually
with trimethoprim/sulfamethoxazole) has decreased the number of fatal
infections. In severe cases, Pneumocystis infection can sometimes be
demonstrated in extrapulmonary sites.
Candida albicans(choice A) can infect the lung and stain with methenamine
silver, but the description of the lavage material would probably include the
terms fungal hyphae and yeast forms.
Giardia lamblia (choice B) causes diarrhea, rather than pneumonia.
Haemophilus influenzae(choice C) and Streptococcus pneumoniae(choice E) are
bacteria and would not stain with silver stains.
Ref: Levinson W. (2012). Chapter 52. Blood & Tissue Protozoa. In W. Levinson
(Ed),Review of Medical Microbiology & Immunology, 12e.
Sample Previous Year Question on HIV based on previous Year Questions of
NEET PG, USMLE,PLAB,FMGE (MCI Screening). Please visit
www.medicoapps.org for more such Quizzes
A terminally ill HIV infected patient develops focal neurologic signs, dementia,
and coma. Amoebic parasites are demonstrated in CSF. Which of the following
organisms is most likely to be the causative agent?
A: Acanthamoeba sp
B: Entamoeba histolytica
C: Giardia lamblia
D: Naegleria fowleri
Correct Ans:A
Explanation
Two types of free-living amoeba can infect the brain and meninges: Naegleria
fowleri and Acanthamoeba species. The former affects healthy adolescent or
adult divers, while the latter causes infection in patients with
immunosuppression because of diabetes, alcoholism, cancer, or HIV infection.
The brain infection characteristically has a prominent perivascular character,
which causes a multifocal hemorrhagic necrotizing meningoencephalitis.
Skin ulcers, nasal infection, or pneumonia may also be present. It is thought that
the organisms may release a toxin causing host tissue necrosis. Systemic
antifungal agents (e.g., amphotericin) have some activity against this organism,
but most cases are fatal.
Entamoeba histolytica causes amoebic dysentery and liver abscess. Giardia
lamblia is a flagellate, rather than an amoeba, and causes diarrhea.
Naegleria fowleri is an amoebic cause of meningoencephalitis in previously
healthy swimmers and divers.
Ref: Ray C.G., Ryan K.J. (2010). Chapter 51. Rhizopods. In C.G. Ray, K.J. Ryan
(Eds), Sherris Medical Microbiology, 5e.
Sample Previous Year Question on HIV based on previous Year Questions of
NEET PG, USMLE,PLAB,FMGE (MCI Screening). Please visit
www.medicoapps.org for more such Quizzes
All are caused by RNA viruses, EXCEPT:
A: HIV
B: Dengue
C: Herpangina
D: Fifth disease
Correct Ans:D
Explanation
Fifth disease also known as erythema infectiosum is caused by erythovirus
(Parvovirs) B19. Parvoviruses are linear, non segmented single stranded DNA
virus.
The various manifestations of human parvovirus B19 infection is;
• Erythema infectiosum (fifth disease)
• Polyarthritis
• Aplastic crisis
• Chronic anemia
• Congenital infection (anemia or hydrops fetalis)
Ref: Oski's Pediatrics: Principles And Practice, edited by Julia A. MacMillan,
Ralph David Feigin, 4 th Edition, Page 1230
Sample Previous Year Question on HIV based on previous Year Questions of
NEET PG, USMLE,PLAB,FMGE (MCI Screening). Please visit
www.medicoapps.org for more such Quizzes
A 24-year-old man presents with fever, rash, a mild headache, and a sore throat.
He denies HIV risk factors, although he is sexually active. On examination, his
temperature is 100.8°F and his pulse is 90/min. There is a diffuse, erythematous,
maculopapular rash over most of his body. Generalized adenopathy is
appreciated, and photophobia is noted when funduscopic examination is
attempted. If this man is not treated, which of the following changes in his
serologic status will most likely occur?
A:
The FTA-ABS titer would
fall
B: The FTA-ABS titer would rise
C: The VDRL titer would fall
D: The VDRL titer would rise
Correct Ans:C
Explanation
This is a case of syphilis, which is diagnosed serologically, using either
treponemal and nontreponemal tests. The FTA-ABS (fluorescent treponemal
antibody-absorbed test) is the most widely used of the specific tests and depends
on fluorescent labeling of the organisms with anti-treponemal antibody. The
specific tests tend to rise early in titer and stay elevated throughout the lifetime
of the host, or at least until well after drug therapy has been completed. The
VDRL (Venereal Disease Research Laboratory) is a nontreponemal test that
detects antibodies that cross-react with mammalian cardiolipin called reaginic
antibodies. This test will become positive after the specific test, and its titer will
fall late in infection, with or without drug therapy.
The FTA-ABS titer would fall is incorrect because this titer will remain high
throughout the life of the host, falling off only very slowly after drug cure.
The FTA-ABS titer would rise is incorrect because the specific antibody test will
reach a high positive titer and remain at that level in untreated late stage
syphilis.
The VDRL titer would rise is incorrect because reaginic antibody levels will fall
in late stage syphilis, with or without drug treatment.
Ref: Lukehart S.A. (2012). Chapter 169. Syphilis. In D.L. Longo, A.S. Fauci, D.L.
Kasper, S.L. Hauser, J.L. Jameson, J. Loscalzo (Eds), Harrison's Principles of
Internal Medicine, 18e
Sample Previous Year Question on HIV based on previous Year Questions of
NEET PG, USMLE,PLAB,FMGE (MCI Screening). Please visit
www.medicoapps.org for more such Quizzes
A man presents to a dermatologist because of a severe mucocutaneous rash that
involves most of his body,including his palms and soles. Questioning reveals that
he is a merchant marine who several months previously had an encounter with a
prostitute in Southeast Asia. Which of the following is the most likely causative
agent of this rash?
A: Herpes simplex I
B: Herpes simplex II
C: HIV
D: Treponema pallidum
Correct Ans:D
Explanation
The rash described is that of secondary syphilis, caused by Treponema
pallidum. Involvement of palms and soles by a rash is unusual, and secondary
syphilis should come to mind. Not all patients with secondary syphilis have a
severe form of the rash, and consequently some cases are missed. Primary
syphilis takes the form of a painless, button-like mass called chancres. Tertiary
syphilis, which is now rare, has a propensity for involving the aorta and central
nervous system and can also cause "gummas" (granulomatous-like lesions) in
many sites, notably including liver and bone.
Herpes simplex I usually causes perioral
Herpes simplex II usually causes genital vesicular lesions.
HIV does not itself cause a rash, although coinfection with other organisms can
result in a rash.
Ref: Ray C.G., Ryan K.J. (2010). Chapter 37. Spirochetes. In C.G. Ray, K.J.
Ryan (Eds), Sherris Medical Microbiology, 5e.
Sample Previous Year Question on HIV based on previous Year Questions of
NEET PG, USMLE,PLAB,FMGE (MCI Screening). Please visit
www.medicoapps.org for more such Quizzes
A 23-year-old, ill-appearing woman comes to the emergency room with a fever.
She notes the recent development of nausea, diarrhea, and a rash. Her last
menstrual period began 3 days ago.Physical examination is remarkable for blood
pressure of 90/45 mm Hg and heart rate of 120 beats per minute. A diffuse
erythematous rash with areas of desquamation over the hands and feet is noted.
Infection with which of the following agents is the most likely cause of these signs
and symptoms?
A: Clostridium perfringens
B: HIV-1
C: Shigella dysenteriae
D: Staphylococcus aureus
Correct Ans:D
Explanation
This patient has toxic shock syndrome (TSS), a multisystem syndrome caused by
a toxin (TSST-1) formed by certain strains of S. aureus. TSS usually affects
several organ systems (gastrointestinal, renal, hepatic, hematopoietic,
musculoskeletal, pulmonary) and can result in death. TSS has historically been
associated with the use of tampons in young women, but can also occur in other
patient populations.
Fever, hypotension, diarrhea, and diffuse rash with desquamation of the hands
and feet are common symptoms. Management of shock, renal failure, and adult
respiratory distress syndrome (ARDS) are a priority if these
conditions are present, in addition to appropriate antibacterial treatment.
C. perfringens causes gas gangrene, with necrosis of soft tissues, usually after a
traumatic wound. It is also a cause of food poisoning.
Although HIV-1 can cause many diverse findings and should never be
immediately ruled out, the findings in this patient are most specific for TSS.
Shigella dysenteriae is a cause of dysentery characterized by fever, abdominal
cramps, and bloody diarrhea.
Ref: Ray C.G., Ryan K.J. (2010). Chapter 24. Staphylococci. In C.G. Ray, K.J.
Ryan (Eds), Sherris Medical Microbiology, 5e.
Sample Previous Year Question on HIV based on previous Year Questions of
NEET PG, USMLE,PLAB,FMGE (MCI Screening). Please visit
www.medicoapps.org for more such Quizzes
A 33 year old HIV-positive male complains of headache and blurred vision.
Physical exam reveals papilledema and ataxia. Head CT is normal but CSF
obtained by lumbar puncture reveals encapsulated organisms observable with
India ink. Which of the following is true concerning this organism?
A: It can also be identified with methenamine silver stain
B:
It consists of branching septate
hyphae
C: It exists as a mycelial form at room temperature and as yeast at 37° C
D: It is an encapsulated non dimorphic yeast found worldwide
Correct Ans:D
Explanation
This patient has cryptococcal meningitis, as evidenced by the "encapsulated
organisms observable with India ink" in the CSF (a classic clue). Cryptococcus is
a non dimorphic yeast, meaning that it exists only in the yeast form. It is
encapsulated (that's why the India ink stain works so well) and it reproduces by
budding. It is found worldwide in bird droppings (think pigeons). It can also
cause transient pulmonary illness in otherwise healthy individuals.
The methenamine silver stain is used primarily to demonstrate Pneumocystis
carinii in tissues.
Branching septate hyphae are characteristic of Aspergillus fumigatus, among
other fungi.
Mycoses that exist in mycelial and yeast forms (dimorphism; diphasic;) are
Histoplasma capsulatum, Coccidioides immitis, Blastomyces dermatidis, and
Sporothrix schenckii.
Ref: Ray C.G., Ryan K.J. (2010). Chapter 46. Cryptococcus, Histoplasma,
Coccidioides, and Other Systemic Fungal Pathogens. In C.G. Ray, K.J. Ryan
(Eds), Sherris Medical Microbiology, 5e.
Sample Previous Year Question on HIV based on previous Year Questions of
NEET PG, USMLE,PLAB,FMGE (MCI Screening). Please visit
www.medicoapps.org for more such Quizzes
All are indicators of goal 6 of Millennium Development Goal, EXCEPT:
A: HIV prevalence among women aged 15-49 years
B:
Number of children orphaned by
HIV/AIDS
C: Prevalence and death rates of tuberculosis
D: Prevalence and death rates of malaria
Correct Ans:A
Explanation
Goal: 6. Combat HIV/AIDS, malaria and other diseases
18. HIV prevalence among young people aged 15 to 24 years
19. Condom use rate of the contraceptive prevalence rate
20. Number of children orphaned by HIV/AIDS
21. Prevalence and death rates associated with malaria
22. Proportion of population in malaria-risk areas using effective malaria
prevention and treatment measures
23. Prevalence and death rates associated with tuberculosis
24. Proportion of tuberculosis cases detected and cured under Directly Observed
Treatment, Short-course (DOTS)
Ref: Park’s textbook of Preventive and Social Medicine, 21st edition, page-27.
Sample Previous Year Question on HIV based on previous Year Questions of
NEET PG, USMLE,PLAB,FMGE (MCI Screening). Please visit
www.medicoapps.org for more such Quizzes
Targeted intervention programmes for HIV are done among the following group
of people, EXCEPT:
A: Commercial sex worker
B:
Migrant
laborers
C: Street children
D: Industrial worker
Correct Ans:D
Explanation
The basic purpose of the targeted interventions for HIV is to reduce the rate of
transmission of HIV among the most vulnerable population such as,
• Sex workers
• Intravenous drug users
• Homosexual men
• Truckers
• Migrant laborers
• Street children
The main activities under these intervention projects are behaviour change,
communication, treatment for STDs and creating an enabling environment that
will facilitate behaviour change.
Ref: Park’s Textbook of Preventive and Social Medicine, 19th edition, Page 359.
Sample Previous Year Question on HIV based on previous Year Questions of
NEET PG, USMLE,PLAB,FMGE (MCI Screening). Please visit
www.medicoapps.org for more such Quizzes
As per WHO 2010 guidelines, when should an HIV infected adult be started on
anti retroviral therapy?
A: CD4 count less than 600 irrespective of staging
B:
CD4 count less than 350 irrespective of
staging
C: CD4 count less than 500 irrespective of staging
D: CD4 count less than 450 irrespective of staging
Correct Ans:B
Explanation
Target population 2010 ART guideline 2006 ART guideline
HIV+ asymptomatic
ARV-naive
individuals
CD4 ≤350 cells/mm3 CD4 ≤200 cells/mm3
HIV+ symptomatic
ARV-naive
individuals
WHO clinical stage 2 if CD4 ≤350
cells/mm3
OR
WHO clinical stage 3 or 4 irrespective
WHO stage 2 or 3 and CD4
≤200 cells/mm3
WHO stage 3 if CD4
not available
of CD4 cell count WHO stage 4 irrespective of
CD4 cell count
Consider treatment for WHO
clinical stage 3 and CD4 cell
count between 200 and 350
cells/mm3
HIV+ pregnant
women
CD4 ≤350 cells/mm3 irrespective of
clinical symptoms OR
WHO clinical stage 3 or 4 irrespective
of CD4 cell count
WHO stage 1 or 2 and CD4
≤200 cells/mm3
WHO stage 3 and CD4 ≤350
cells/ mm3
WHO stage 4 irrespective of
CD4 count
Ref: Epidemiology of communicable diseases
Park’s textbook of Preventive and Social Medicine, 21stedition, page-327.
Sample Previous Year Question on HIV based on previous Year Questions of
NEET PG, USMLE,PLAB,FMGE (MCI Screening). Please visit
www.medicoapps.org for more such Quizzes
You are asked to help consult on a study which was recently completed in coastal
African country. In this study, a total of 274 soldiers stationed in three different
camps were examined and tested for the presence of bacterial sexually
transmitted diseases (STD) and human immunodeficiency virus (HIV) positivity.
In addition to the information from the clinical exam and laboratory specimens,
interviews were conducted to gather information on age, years of military
service, ethnicity, and region of origin. The data are now to be analyzed to learn
which factors were associated with the presence of STDs, including HIV. What is
the most accurate description of this study design?
A: A case-control study
B:
A cohort
study
C: A clinical trial
D: A cross-sectional study
Correct Ans:D
Explanation
In a cross-sectional study, a variety of independent variables can be studied for
their association with the outcome variables. In this example, the independent
variables would include age, years of military service, ethnicity, and region of
origin. The outcome variables include the presence of bacterial sexually
transmitted diseases (STDs) and HIV positivity. This study does not meet the
criteria for being a case-control, cohort, or case-cohort study, nor is it a clinical
trial. The cross-sectional study is limited in its power, and valid inferences may
be especially difficult to make from cross-sectional studies, even when
statistically significant associations are observed.
Sample Previous Year Question on HIV based on previous Year Questions of
NEET PG, USMLE,PLAB,FMGE (MCI Screening). Please visit
www.medicoapps.org for more such Quizzes
HIV sentinel surveillance is used to calculate:
A: Detection of high risk group
B: Prevalence
C: Monitoring disease trend
D: Detection of incidence
Correct Ans:C
Explanation
HIV sentinel surveillance used to calculate Trend finding
HIV sentinel surveillance (HSS) in India
• HIV sentinel surveillance (HSS) in India, since its inception in
1998, has evolved into a credible and robust system for HIV
epidemic monitoring and acclaimed as one of the best in the world.
• Sentinel surveillance provides essential information to understand
the trends and dynamics of HIV epidemic among different risk
groups in the country
• It aids in refinement of strategies and prioritization of focus for
prevention, care and treatment intervention under the National
AIDS Control Programme (NACP)
• HIV estimates of prevalence, incidence and mortality developed
based on findings from HIV Sentinel Surveillance enable the
programme in assessing the impacts the impacts at a macro level.
Ref: Park 22nd ed/ 400.
Sample Previous Year Question on HIV based on previous Year Questions of
NEET PG, USMLE,PLAB,FMGE (MCI Screening). Please visit
www.medicoapps.org for more such Quizzes
KAP studies in India were first used to study:
A: HIV
B: Malaria
C: Family planning
D: Cancer cervix
Correct Ans:C
Explanation
KAP studies were used to do family planning studies. It gives a description as to
what people think and know about fertility and family planning. It also measures
as to how effective is family planning programme.
HIV, malaria, and cervical cancer were also studied using KAP. But family
planning were the earliest studied by applying the principles of KAP.
Ref: Studies in Family Planning - India By Kamala Gopal Rao, Pages 840-42
Sample Previous Year Question on HIV based on previous Year Questions of
NEET PG, USMLE,PLAB,FMGE (MCI Screening). Please visit
www.medicoapps.org for more such Quizzes
All of the following are zoonosis, except:
A: Plague
B: Japanese Encephalitis (JE)
C: HIV
D: Tuberculosis (TB)
Correct Ans:C
Explanation
Zoonosis is defined by the Joint FAO/WHO Expert Committee as 'those diseases
and infections which are naturally transmitted between vertebrate animals and
man’. There are more than 150 diseases and infections having reservoir as
animals and is communicable to man. These zoonotic diseases may be caused by
viruses, bacteriae, rickettisiae, protozoa, helminths, anthropods, fungi or insects.
HIV is not a zoonotic disease while the other three infections are zoonotic
diseases. The human immunodeficiency virus has only human as reservoirs.
Ref: Park Textbook of Preventive and Social Medicine, 19th Edition, Pages 642,
688
Sample Previous Year Question on HIV based on previous Year Questions of
NEET PG, USMLE,PLAB,FMGE (MCI Screening). Please visit
www.medicoapps.org for more such Quizzes
Which of the following is the number of national helpline for HIV/AIDS?
A: 1081
B:
109
1
C: 1097
D: 1100
Correct Ans:C
Explanation
A toll free national AIDS telephone help line has been set up to provide access to
information and counselling.
Ref: Park 21st edition, page 399.
Sample Previous Year Question on HIV based on previous Year Questions of
NEET PG, USMLE,PLAB,FMGE (MCI Screening). Please visit
www.medicoapps.org for more such Quizzes
A new drug with in vitro activity against HIV is tested on a population of patients with
Western-blot confirmed HIV infections, in Mumbai. Out of the 200 individuals in the patient
population, 100 individuals are chosen by a lot to receive the drug. The drug, which is
tasteless, is administered in a cup of orange juice; the other patients receive pure orange
juice. Neither the nurses, doctors, nor patients know which patients receive the drug. At the
end of the study period, the number of CD4+ T cells is determined for all of the subjects.
What is the research method used in this study?
A: Case-control study
B:
Case
report
C: Cohort study
D: Double-blind randomized clinical trial
Correct Ans:D
Explanation
This study conducted in Mumbai is an example of a placebo-controlled, double-blind
randomized clinical trial. In this type of study, subjects with a particular disorder are
randomized (the lottery) to receive either the treatment in question or a placebo. The
information about whether the patient receives the treatment or the placebo is not known
to either the subject or the investigator (double-blind). This type of study may be used to
infer causality, i.e., if the patients taking the medication have more CD4+ T cells than the
other group, it is due to the drug in question.
A case-control study (Choice : Case-control study) is a retrospective study that pairs known
cases of a disease with matched controls. This type of study gives information about the
importance of risk factors. The lack of a prospective design makes inferences about
causality difficult.
A case report (Choice : Case report) is a published report of a single incidence of a disease
in a subject. This is usually done for rare diseases, or for rare associations of a disease with
a particular risk factor.
A cohort study (Choice : Cohort study) is an observational (non-interventional) study that
tracks subjects prospectively. This type of study gives a measure of incidence of a disease.
Sample Previous Year Question on HIV based on previous Year Questions of
NEET PG, USMLE,PLAB,FMGE (MCI Screening). Please visit
www.medicoapps.org for more such Quizzes
In a city with a population of 1,000,000, 10,000 individuals have HIV disease. There are
1000 new cases of HIV disease and 200 deaths each year from the disease. There are 2500
deaths per year from all causes. Assuming no net emigration from or immigration to the
city, the incidence of HIV disease in this city is given by which of the following?
A: 200/1,000,000
B:
800/1,000,00
0
C: 1000/1,000,000
D: 2500/1,000,000
Correct Ans:C
Explanation
The incidence of a disease is given by the number of new cases in a given period divided by
the total population. In this case, this is equal to 1000/1,000,000.
The disease-specific mortality rate is the number of deaths per year from a specific disease
divided by the population; in this case, 200/1,000,000.
The rate of increase of a disease is given by the number of new cases per year, minus the
number of deaths (or cures) per year, divided by the total population. Since there is yet no
cure for HIV disease, the number of cures is 0.
In this case, the rate of increase is (1000-200)/1,000,000 = 800/1,000,000. The crude
mortality rate is given by the number of deaths from all causes, divided by the population;
in this case, 2500/1,000,000.
Sample Previous Year Question on HIV based on previous Year Questions of
NEET PG, USMLE,PLAB,FMGE (MCI Screening). Please visit
www.medicoapps.org for more such Quizzes
All of the following are zoonosis, except:
A: Plague
B: Japanese Encephalitis (JE)
C: HIV
D: Tuberculosis (TB)
Correct Ans:C
Explanation
Zoonosis is defined by the Joint FAO/WHO Expert Committee as 'those diseases and
infections which are naturally transmitted between vertebrate animals and man’. There are
more than 150 diseases and infections having reservoir as animals and is communicable to
man. These zoonotic diseases may be caused by viruses, bacteriae, rickettisiae, protozoa,
helminths, anthropods, fungi or insects.
HIV is not a zoonotic disease while the other three infections are zoonotic diseases. The
human immunodeficiency virus has only human as reservoirs.
Ref: Park Textbook of Preventive and Social Medicine, 19th Edition, Pages 642, 688
Sample Previous Year Question on HIV based on previous Year Questions of
NEET PG, USMLE,PLAB,FMGE (MCI Screening). Please visit
www.medicoapps.org for more such Quizzes
Which of the following exposure carries the maximum risk of transmission of HIV?
A: Needle Prick
B: Trans-Placental
C: Blood Transfusion
D: Sexual Intercourse
Correct Ans:C
Explanation
It is estimated that 90% of individuals who were exposed to HIV - contaminated products
became infected. So, blood transfusion carries maximum risk of HIV transmission. 75-90%
of individuals who received contaminated factor VIII acquire HIV.
Ref: Harrison’s Principles of Internal Medicine, 17th Edition, Page 1144; Sexual and
Reproductive Health: A Public Health Perspective By Paul Van Look, Pages 117-9
Sample Previous Year Question on HIV based on previous Year Questions of
NEET PG, USMLE,PLAB,FMGE (MCI Screening). Please visit
www.medicoapps.org for more such Quizzes
One of your staff nurse had a deep prick from the needle used to inject an HIV positive
individual. What is the treatment regime that should be started in her/him?
Note that drug resistance was suspected in the HIV patient.
A: 2 NRTIs for 3 months
B: 1 NRTI and 1 NtRTI for 28 days
C: 2 NRTIs + 1 protease inhibitor for 28 days
D: 2 protease inhibitor + 2 NtRTIs for 3 months
Correct Ans:C
Explanation
In this scenario ARVs should be initiated as soon as possible and no later than 72 hours. It
should include 2 NRTI drugs. If drug resistance is suspected a protease inhibitor should
also be added. The duration of treatment is 28 days. If the blood sample drawn from the
exposed person is HIV negative at the time of exposure, the test should be repeated at 3
and 6 months.
Ref: Park, Edition 21, Page - 328
Sample Previous Year Question on HIV based on previous Year Questions of
NEET PG, USMLE,PLAB,FMGE (MCI Screening). Please visit
www.medicoapps.org for more such Quizzes
WHO stage IV HIV includes all, EXCEPT:
A: Toxoplasmosis
B: Pneumocystis carinii
C: HIV wasting syndrome
D: Oral thrush
Correct Ans:D
Explanation
According to WHO clinical staging system for HIV infection, oral thrush is seen in stage III
disease.
• Stage IV includes,
• HIV wasting syndrome
• Pneumocystis carinii pneumonia
• Toxoplasmosis of the brain
• Cryptosporidiosis
• Cryptococcosis
• Cytomegalovirus disease
• Herpes virus infection
• Progressive multifocal leukoencephalopathy
• Disseminated fungal infections
• Candidiasis of the esophagus, trachea, bronchi, or lungs
• Atypical mycobacteriosis
• Non-typhoid salmonella septicemia
• Extrapulmonary TB
• Lymphoma
• Kaposi sarcoma
• HIV encephalopathy
Ref: Park’s Textbook of Preventive and Social Medicine, 19th edition, Page 292.
Sample Previous Year Question on HIV based on previous Year Questions of
NEET PG, USMLE,PLAB,FMGE (MCI Screening). Please visit
www.medicoapps.org for more such Quizzes
According to CDC recommendations, HIV screening of pregnant women is:
A: Opt in testing
B: Opt out testing
C: Compulsory
D: Symptomatic
Correct Ans:B
Explanation
The Center for Disease Control and Prevention, the American College of Obstetricians and
Gynecologists, the American Academy of Pediatrics, and the United States Preventive
Services Task Force recommend prenatal screening using an opt-out approach. This means
that the woman is notified that HIV testing is included in a comprehensive set of antenatal
tests, but that testing may be declined. Women are given information regarding HIV but are
not required to sign a specific consent. Through the use of such opt-out strategies, HIV
testing rates have increased. Each provider should be aware of specific state laws
concerning screening.
Ref: Cunningham F.G., Leveno K.J., Bloom S.L., Hauth J.C., Rouse D.J., Spong C.Y. (2010).
Chapter 59. Sexually Transmitted Diseases. In F.G. Cunningham, K.J. Leveno, S.L. Bloom,
J.C. Hauth, D.J. Rouse, C.Y. Spong (Eds), Williams Obstetrics, 23e.
Sample Previous Year Question on HIV based on previous Year Questions of
NEET PG, USMLE,PLAB,FMGE (MCI Screening). Please visit
www.medicoapps.org for more such Quizzes
Cotrimoxazole therapy is to be given in HIV infected patients irrespective of presence of
symptoms if CD4 count is less than:
A: 100
B:
15
0
C: 200
D: 350
Correct Ans:C
Explanation
Routine prophylaxis with co-trimoxazole (CPT) is provided under the national programme.
Under the national programme, CPT may be initiated in the following scenarios:
If CD4 is not available (or result pending): WHO clinical stage 3 and 4
If CD4 is available:
• HIV infected adults with CD4 <200 cells/mm
• or CD4 <350 cells/mm if if patient is symptomatic
• or WHO clinical stage 3 or 4 irrespective of CD4.
Ref: http://naco.gov.in/upload/Policies%20&%20Guidelines/1.%20Antiretroviral%20The
rapy%20Guidelines%20for%20HIV-
Infected%20Adults%20and%20Adolescents%20Including%20Post-exposure.pdf
Sample Previous Year Question on HIV based on previous Year Questions of
NEET PG, USMLE,PLAB,FMGE (MCI Screening). Please visit
www.medicoapps.org for more such Quizzes
Anti retroviral therapy is to be given in HIV infected patients irrespective of presence of
symptoms if CD4 count is less than:
A: 100
B:
15
0
C: 200
D: 350
Correct Ans:C
Explanation
• Offer ART to symptomatic patients if the CD4 count is 200–350 cells/mm3
• Consider ART for asymptomatic patients with CD4 count between 200-350
cells/mm3
and monitor closely for new symptoms.
• If the CD4 count is 200–250 cells/mm3, physicians can consider repeating
the CD4 test in 4 weeks in asymptomatic patients. This is to rule out the
possibility of a 20% margin of error in laboratory results.
• Patients should start ART before the CD4 count drops below 200 cells/mm3
Ref: http://naco.gov.in/upload/Policies%20&%20Guidelines/1.%20Antiretroviral%20The
rapy%20Guidelines%20for%20HIV-
infected%20Adults%20and%20Adolescents%20Including%20Post-exposure.pdf
Sample Previous Year Question on HIV based on previous Year Questions of
NEET PG, USMLE,PLAB,FMGE (MCI Screening). Please visit
www.medicoapps.org for more such Quizzes
Anti retroviral therapy is to be given in HIV infected patients irrespective of
presence of symptoms if CD4 count is less than:
A: 100
B:
15
0
C: 200
D: 350
Correct Ans:C
Explanation
• Offer ART to symptomatic patients if the CD4 count is 200–350 cells/mm3
• Consider ART for asymptomatic patients with CD4 count between 200-350
cells/mm3
and monitor closely for new symptoms.
• If the CD4 count is 200–250 cells/mm3, physicians can consider repeating
the CD4 test in 4 weeks in asymptomatic patients. This is to rule out the
possibility of a 20% margin of error in laboratory results.
• Patients should start ART before the CD4 count drops below 200 cells/mm3
Ref: http://naco.gov.in/upload/Policies%20&%20Guidelines/1.%20Antiretroviral%20The
rapy%20Guidelines%20for%20HIV-
infected%20Adults%20and%20Adolescents%20Including%20Post-exposure.pdf
Sample Previous Year Question on HIV based on previous Year Questions of
NEET PG, USMLE,PLAB,FMGE (MCI Screening). Please visit
www.medicoapps.org for more such Quizzes
You are asked to help consult on a study which was recently completed in coastal African
country. In this study, a total of 274 soldiers stationed in three different camps were
examined and tested for the presence of bacterial sexually transmitted diseases (STD) and
human immunodeficiency virus (HIV) positivity. In addition to the information from the
clinical exam and laboratory specimens, interviews were conducted to gather information
on age, years of military service, ethnicity, and region of origin. The data are now to be
analyzed to learn which factors were associated with the presence of STDs, including HIV.
What is the most accurate description of this study design?
A: A case-control study
B:
A cohort
study
C: A clinical trial
D: A cross-sectional study
Correct Ans:D
Explanation
In a cross-sectional study, a variety of independent variables can be studied for their
association with the outcome variables. In this example, the independent variables would
include age, years of military service, ethnicity, and region of origin. The outcome variables
include the presence of bacterial sexually transmitted diseases (STDs) and HIV positivity.
This study does not meet the criteria for being a case-control, cohort, or case-cohort study,
nor is it a clinical trial. The cross-sectional study is limited in its power, and valid inferences
may be especially difficult to make from cross-sectional studies, even when statistically
significant associations are observed.
Sample Previous Year Question on HIV based on previous Year Questions of
NEET PG, USMLE,PLAB,FMGE (MCI Screening). Please visit
www.medicoapps.org for more such Quizzes
A HIV infected patient develops multibacillary leprosy. What is the recommended
treatment?
A:
No anti leprosy drug should be
given
B: Rifampicin, dapsone and clofazimine
C: Rifampicin and dapsone
D: Rifampicin only
Correct Ans:B
Explanation
Multi drug therapy for leprosy is not contraindicated in HIV infection. WHO recommends the
following drugs for treatment of multibacillary leprosy
• Rifampicin: 600 mg once monthly
• Dapsone: 100 mg daily
• Clofazimine: 300 mg once monthly supervised; and 50 mg daily, self
administered.
Ref: Park 21st edition, page 297.
Sample Previous Year Question on HIV based on previous Year Questions of
NEET PG, USMLE,PLAB,FMGE (MCI Screening). Please visit
www.medicoapps.org for more such Quizzes
Tonsillar carcinoma is associated with infection of:
A:
HI
V
B: HPV
C: HSV
D: None of the above
Correct Ans:B
Explanation
HPV is an epitheliotropic virus that has been detected to varying degrees within samples of
oral cavity squamous cell carcinoma. Infection alone is not considered sufficient for
malignant conversion; however, results of multiple studies suggest a role of HPV in a subset
of head and neck squamous cell carcinoma. Approximately 40% of tonsillar carcinomas
demonstrate evidence of HPV types 16 and 18.
Ref: Schwartz’s principle of surgery 9th edition, chapter 18.
Sample Previous Year Question on HIV based on previous Year Questions of
NEET PG, USMLE,PLAB,FMGE (MCI Screening). Please visit
www.medicoapps.org for more such Quizzes
Which of the following is an absolute contraindication for renal transplant?
A: Untreated malignancy
B:
Age >65
years
C: HIV infection
D: Chronic hepatitis B or C
Correct Ans:A
Explanation
Absolute contraindications to kidney transplantation
Untreated malignancy
Active infection
Untreated HIV infection or AIDS
Any condition where life expectancy is under two years
Relative contraindications to kidney transplantation
Comorbid condition
Age >65 years
Untreated coronary artery disease
Obesity
HIV infection
Previous malignancy
Chronic hepatitis B or C
Ref: CLINICAL REVIEW, Renal transplantation, BMJ 2011;343:d7300 ; The Renal
Association. Assessment of the potential kidney transplant recipient. 2011.
Sample Previous Year Question on HIV based on previous Year Questions of
NEET PG, USMLE,PLAB,FMGE (MCI Screening). Please visit
www.medicoapps.org for more such Quizzes
Salmonella osteomyelitis is common in which of the following?
A: Sickle cell disease
B: HIV
C: IV drug abusers
D: Pregnancy
Correct Ans:A
Explanation
Increased incidence of salmonella (may spread from gallbladder infection) is seen in sickle
cell anemia, but staph aureus is still the most common. Staph aureus is the most common
cause of osteomyelits in all children, including those with sickle cell disease. Salmonella
osteomyelitis occurs most commonly in children with sickle cell disease, but is still less
common than Staph aureus in these patients. Osteomyelitis most often affects diaphysis.
Sample Previous Year Question on HIV based on previous Year Questions of
NEET PG, USMLE,PLAB,FMGE (MCI Screening). Please visit
www.medicoapps.org for more such Quizzes
Which of the following congenital infections is associated with minimal teratogenic risk to
the fetus:
A: HIV
B: Rubella
C: Varicella
D: CMV
Correct Ans:A
Explanation
HIV infection in pregnant women has no proven teratogenic effects on the fetus. While the
other three are teratogenic infections.
Ref: Dutta textbook of Obstetrics 6th edition, page 299
Sample Previous Year Question on HIV based on previous Year Questions of
NEET PG, USMLE,PLAB,FMGE (MCI Screening). Please visit
www.medicoapps.org for more such Quizzes
All of the following interventions are recommended to prevent mother to child transmission
of HIV, except:
A: Avoid Ergometrine in third stage of labour
B: Highly Active Antiretroviral Therapy (HAART)
C: Elective Caesarean section
D: Intrapartum Zidovudine
Correct Ans:A
Explanation
Ergometrine is give the third stage of labour to reduce post partum haemorrhage, it has no
role in the prevention of HIV transmission.
Ref: Williams Obstetrics, 23rd Edition, Page 1251,1252, 1253; Textbook of Obstetrics By DC
Dutta, 6th Edition, Page 301; Disease Control Priorities in Developing Countries By Dean T.
Jamison, World Bank, Disease Control Priorities Project, Page 346
Sample Previous Year Question on HIV based on previous Year Questions of
NEET PG, USMLE,PLAB,FMGE (MCI Screening). Please visit
www.medicoapps.org for more such Quizzes
A patient presented with intense vulvar pruritus, a white curd like, cheesy vaginal
discharge and vulvar erythema. Candidiasis was diagnosed. Which among the following is
NOT a risk factor for vaginal candidiasis?
A: Diabetes mellitus
B:
Hypertensio
n
C: Human Immunodeficiency Virus (HIV) infection
D: Pregnancy
Correct Ans:B
Explanation
Risk factors of candida infection include:
• Diabetes mellitus
• Human immunodeficiency virus [HIV]
• Obesity
• Pregnancy
• Medication (antibiotics, corticosteroids, oral contraceptives)
• Chronic debilitation
Ref: Mazdisnian F. (2007). Chapter 37. Benign Disorders of the Vulva & Vagina. In A.H.
DeCherney, L. Nathan (Eds), CURRENT Diagnosis & Treatment Obstetrics & Gynecology,
10e.
Sample Previous Year Question on HIV based on previous Year Questions of
NEET PG, USMLE,PLAB,FMGE (MCI Screening). Please visit
www.medicoapps.org for more such Quizzes
A 24 year old female with multiple sex partners presented with vaginal discharge which is
thin and white with fishy odor. Whiff test is positive which indicates the infection with
which of the following organism?
A: TORCH
B:
HI
V
C: Bacterial vaginosis
D: Birth anoxia
Correct Ans:C
Explanation
Whiff test is positive for bacterial vaginosis. It involves the production of a fishy odor when
mixing vaginal fluid with 10% potassium hydroxide.
Amstel criteria for diagnosing BV:
Three of the following four must be present.
• Thin, homogenous vaginal discharge
• A positive whiff test
• Vaginal fluid pH greater than 4.5
• Presence of clue cells (epithelial cells covered with bacteria) on
microscopic examination
Ref: Rosen T. (2012). Chapter 205. Gonorrhea, Mycoplasma, and Vaginosis. In L.A.
Goldsmith, S.I. Katz, B.A. Gilchrest, A.S. Paller, D.J. Leffell, N.A. Dallas (Eds), Fitzpatrick's
Dermatology in General Medicine, 8e.
Sample Previous Year Question on HIV based on previous Year Questions of
NEET PG, USMLE,PLAB,FMGE (MCI Screening). Please visit
www.medicoapps.org for more such Quizzes
Virus associated with cancer cervix is:
A: HPV
B:
HI
V
C: EBV
D: HTLV
Correct Ans:A
Explanation
HPV has a established strong association in the preponderance of cervical
cancer. Persistent HPV infection along with metaplasia resulting from any other
etiology results in various degrees of dysplasia. HPV infection is diagnosed by
demonstrating koilocytes(large cells with vacuolated cytoplasm and large bland
nucleus) and dyskaryocytes (small clusters of cells with orangophilic cytoplasm
and dark small nuclei) in cytology; it can also be confirmed with PCR - DNA.
Ref: Recent Advances in Adolescent Health By Roza Olyai, Page 32 ; Cancer
Associated Viruses By Erle Robertson, Page 509 ; Human Oncogenic Viruses By
Jing-Hsiung James Ou, T. S. Benedict Yen, Pages 5-11
Sample Previous Year Question on HIV based on previous Year Questions of
NEET PG, USMLE,PLAB,FMGE (MCI Screening). Please visit
www.medicoapps.org for more such Quizzes
Patient diagnosed as squamous cell intraepithelial lesion. Which of the following
has the highest risk for progression to carcinoma?
A:
Low grade squamous intraepithelial
neoplasia
B: High grade squamous intraepithelial neoplasia
C: Squamous intraepithelial neoplasia associated with HPV 16
D: Squamous intraepithelial neoplasia associated with HIV
Correct Ans:B
Explanation
Cervical intraepithelial neoplasia (CIN grades I–III) and squamous
intraepithelial lesions (SIL) are dysplastic changes in the cervical epithelium that
are a precursor to malignancy (i.e., squamous cell carcinoma of the cervix) and
are the result of infection with HPV.
High-grade SIL and CIN II–III lesions are caused by HPV types 16, 18, 31, 33, 35,
39, 45, and 52, and have a high risk for progression to an invasive squamous cell
carcinoma. Low-grade SIL and CIN I lesions are caused by HPV types 6, 11, 42,
and 44, and have a low risk for progression to squamous cell carcinoma.
Ref: Kemp W.L., Burns D.K., Brown T.G. (2008). Chapter 17. Pathology of the
Male and Female Reproductive Tract and Breast. In W.L. Kemp, D.K. Burns,
T.G. Brown (Eds), Pathology: The Big Picture
Sample Previous Year Question on HIV based on previous Year Questions of
NEET PG, USMLE,PLAB,FMGE (MCI Screening). Please visit
www.medicoapps.org for more such Quizzes
All the following mechanisms occur in a neonate for heat production, EXCEPT:
A: Shivering
B: Breakdown of brown fat with adrenaline secretion
C: Universal flexion like a fetus
D: Cutaneous vasoconstriction
Correct Ans:A
Explanation
Non shivering thermogenesis or breaking of brown fat is the main method of heat
production in the newborn. Shivering which is a major mechanism of heat production in
adults is rarely seen in newborn. In newborns, brown fat is located in the upper back and
neck, mediastinum and around the kidneys. They contain high store of triglycerides which
are broken down during metabolic process to produce glycerol and fatty acids which in turn
releases heat.
Mechanisms of heat production in newborn are:
• Newborn’s mechanism of heat production is primarily through increased
metabolic process. Cold stress increases the metabolism and thus requiring
an increase in oxygen and calorie consumption.
• Vasoconstriction of blood vessels.
• Increase in metabolic rate and muscular activity.
• Non shivering thermogenesis
Ref: Universal Newborn Hearing Screening edited by Lynn G. Spivak page 121, Examination
of the Newborn: An Evidence Based Guide By Anne Lomax page 56.
Sample Previous Year Question on HIV based on previous Year Questions of
NEET PG, USMLE,PLAB,FMGE (MCI Screening). Please visit
www.medicoapps.org for more such Quizzes
A preterm baby developed bullous lesion on the skin soon after birth. The X- ray shows
periostitis. What should be the next investigation?
A: VDRL for mother & baby
B:
ELISA for
HIV
C: PCR for TB
D: Hepatitis surface antigen for mother
Correct Ans:A
Explanation
The baby described in the question stem is most likely suffering from the early
manifestations of congenital syphilis. As per the choices given VDRL is the test done to
diagnose syphilis.
Ref: Harrison’s Principles of Internal Medicine, 16th Edition, Page 981; Skin Diseases &
Sexually Transmitted Infections, Uday Khopkar – 6th Edition, Page 257; Nelson 17th
,COGDT 10th Edition, Page 667;CPDT 18th Edition, Page 56.
Sample Previous Year Question on HIV based on previous Year Questions of
NEET PG, USMLE,PLAB,FMGE (MCI Screening). Please visit
www.medicoapps.org for more such Quizzes
Which response is not seen in newborns as a measure of thermogenesis?
A: Shivering
B: Breakdown of brown fat
C: Universal flexion
D: Cutaneous vasoconstrictions
Correct Ans:A
Explanation
Newborns capacity to generate heat by shivering is limited. Primary mechanism of heat
production in the neonate is through metabolism of brown fat or through non shivering
thermogenesis.
Major location of brown fat storage in neonates are upper back and neck, mediastinum and
around the kidneys. Brown fat contains hish stores of triglycerides which is broken down
into glycerol and fatty acids. Oxidation of fatty acids result in the generation of heat.
Sample Previous Year Question on HIV based on previous Year Questions of
NEET PG, USMLE,PLAB,FMGE (MCI Screening). Please visit
www.medicoapps.org for more such Quizzes
A preterm baby developed bullous lesion on the skin soon after birth. The X- ray shows
periostitis. What should be the next investigation?
A: VDRL for mother & baby
B:
ELISA for
HIV
C: PCR for TB
D: Hepatitis surface antigen for mother
Correct Ans:A
Explanation
The baby described in the question stem is most likely suffering from the early
manifestations of congenital syphilis. As per the choices given VDRL is the test done to
diagnose syphilis.
Ref: Harrison’s Principles of Internal Medicine, 16th Edition, Page 981; Skin Diseases &
Sexually Transmitted Infections, Uday Khopkar – 6th Edition, Page 257; Nelson 17th
,COGDT 10th Edition, Page 667;CPDT 18th Edition, Page 56.
Sample Previous Year Question on HIV based on previous Year Questions of
NEET PG, USMLE,PLAB,FMGE (MCI Screening). Please visit
www.medicoapps.org for more such Quizzes
Which of the following investigation is the BEST method of diagnosing HIV in childhood?
A: CD4 cell counts
B:
P24
antigen
C: ELISA
D: Anti HIV antibody
Correct Ans:B
Explanation
The two tests used for diagnosing HIV infection in in children
less than 18 months are detection of HIV nucleic acid (DNA
or RNA) AND detection of HIV p24 antigen. Definitive
diagnosis requires presence of reactive tests on two separate
determinations using one of the above tests after 4 or 6
weeks and confirmed by the second test. Diagnostic testing
should be done 6 weeks or longer after complete cessation of
breast feeding.
Laboratory criteria for diagnosing HIV infection in children
more than 18 months:
• Serologic testing
• Detection of HIV nucleic acid(DNA or RNA)
• Detection of HIV p24 antigen
Ref: Advances in Pediatrics by Dutta By Jaypee Brothers, page 256.
Sample Previous Year Question on HIV based on previous Year Questions of
NEET PG, USMLE,PLAB,FMGE (MCI Screening). Please visit
www.medicoapps.org for more such Quizzes
Which of the following investigation is the BEST method of diagnosis of HIV in childhood?
A: CD4 cell counts
B:
P24
antigen
C: ELISA
D: Anti HIV antibody
Correct Ans:B
Explanation
The two tests used for diagnosing HIV infection in in children less than 18 months are
detection of HIV nucleic acid (DNA or RNA) AND detection of HIV p24 antigen.Definitive
diagnosis requires presence of reactive tests on two separate determinations using one of
the above tests after 4 or 6 weeks and confirmed by the second test. Diagnostic testing
should be done 6 weeks or longer after complete cessation of breast feeding.
Infants born to HIV infected mothers will have HIV antibody until 6-18 months regardless
of infection status due to transplacental passage of maternal antibody. The preferred test
for infant diagnosis is detection of HIV DNA or RNA in blood, collectively referred to as
nucleic acid amplification testing (NAT).
Laboratory criteria for diagnosing HIV infection in children more than 18 months:
• Serologic testing
• Detection of HIV nucleic acid(DNA or RNA)
• Detection of HIV p24 antigen
Ref: Advances in Pediatrics by Dutta By Jaypee Brothers, page 256. McFarland E.J. (2012).
Chapter 41. Human Immunodeficiency Virus Infection. In W.W. Hay, Jr., M.J. Levin, R.R.
Deterding, J.J. Ross, J.M. Sondheimer (Eds), CURRENT Diagnosis & Treatment: Pediatrics,
21e.
Sample Previous Year Question on HIV based on previous Year Questions of
NEET PG, USMLE,PLAB,FMGE (MCI Screening). Please visit
www.medicoapps.org for more such Quizzes
A patient with HIV is on therapy with protease inhibitors. He presents with limitation of
abduction and internal rotation of the hip. The most probable diagnosis is:
A: Tuberculosis of hip joint
B: Avascular necrosis (AVN) of femoral head
C: Secondary osteoarthritis of hip
D: Septic arthritis of hip
Correct Ans:B
Explanation
Limitation of abduction and internal rotation of hip is often seen in avascular necrosis of
femoral head or Legg-Calve-Perthes disease. Hypertriglyceridemia is a well known risk
factor of avascular necrosis of femoral head (AVN). It has been suggested that treatment
with protease inhibitors increase triglyceride levels, thereby increasing risk of avascular
necrosis of femoral head.
Ref: Current Diagnosis and Treatment of Pain (CDTP), 2006, Page 305; Antiretrovial
Therapy By Clercq, 2001, Page 167; Apley’s System of Orthopaedics and Fractures By Louis
Solomon, 9th Edition, Chapter 19, The Hip, Pages 511-15
Sample Previous Year Question on HIV based on previous Year Questions of
NEET PG, USMLE,PLAB,FMGE (MCI Screening). Please visit
www.medicoapps.org for more such Quizzes
Which one of the following viruses is least likely to cross placenta?
A: Rubella
B: Herpes Simplex Virus
C: HIV
D: HBV
Correct Ans:D
Explanation
Considering all the above given options, the hepatitis B virus is least likely to cross the
placenta due to its larger molecular size (virion of 42nm in diameter) which gives it less
access through the placenta. It is not teratogenic.
Ref: Dutta Textbook of obstetrics 6th edition, Pages 292, 299-300; Medical disorders in
pregnancy: an update By Hiralal Konar, Pralhad Kushtagi, Pages 67-74.
Sample Previous Year Question on HIV based on previous Year Questions of
NEET PG, USMLE,PLAB,FMGE (MCI Screening). Please visit
www.medicoapps.org for more such Quizzes
All of the following interventions are recommended to prevent mother to child transmission
of HIV, except:
A: Avoid Ergometrine in third stage of labour
B: Highly Active Antiretroviral Therapy (HAART)
C: Elective Caesarean section
D: Intrapartum Zidovudine
Correct Ans:A
Explanation
Ergometrine is give the third stage of labour to reduce post partum haemorrhage, it has no
role in the prevention of HIV transmission.
Ref: Williams Obstetrics, 23rd Edition, Page 1251,1252, 1253; Textbook of Obstetrics By DC
Dutta, 6th Edition, Page 301; Disease Control Priorities in Developing Countries By Dean T.
Jamison, World Bank, Disease Control Priorities Project, Page 346
Sample Previous Year Question on HIV based on previous Year Questions of
NEET PG, USMLE,PLAB,FMGE (MCI Screening). Please visit
www.medicoapps.org for more such Quizzes
All of the following interventions are recommended to prevent mother to child transmission
of HIV, except:
A: Avoid breast feeding
B: Highly Active Antiretroviral Therapy (HAART)
C: Vaginal delivery
D: Intrapartum Zidovudine
Correct Ans:C
Explanation
For HIV-infected mothers, a scheduled cesarean delivery at 38 weeks can reduce the risk of mother -
to - child transmission of the virus. If, before her scheduled cesarean delivery, rupture of
membranes or she goes into labor, a cesarean delivery may not reduce the risk of mother to-child
transmission of HIV.
Ref: Recommendations for Use of Antiretroviral Drugs in Pregnant HIV-1-Infected
Women By Michael A. Stoto, Donna A. Almario, Institute of Medicine (U.S.). Committee on
Perinatal Transmission of HIV, National Research Council (U.S.). Board on Children, Youth,
and Families, Pages 358-360.
Sample Previous Year Question on HIV based on previous Year Questions of
NEET PG, USMLE,PLAB,FMGE (MCI Screening). Please visit
www.medicoapps.org for more such Quizzes
Non Immune Hydrops Fetalis is caused by:
A: CMV
B: Parvovirus
C: HSV
D:
HI
V
Correct Ans:B
Explanation
Non immune hydrops can be caused by a variety of causes. Among the infectious cause,
parvovirus can lead to non immune hydrops.
Hydrops fetalis refers to excessive accumulation of fluid in at least two fetal body cavities.
It can be of two types, immune and non immune. Immune type result from hemolysis of
fetal RBC by maternal IgG antibodies to fetal antigen. Non immune type, is the result of any
non antibody mediated process that lead to hydrops.
Ref: Fetal Medicine : Basic Science and Clinical Practice, 2nd Edition, Page 522; Ian Donald's
Practical Obstetric Problems, 6th Edition, Page 218; William's Obstetrics, 23rd Edition, Page
627; Moffet's Pediatric Infectious Diseases : A Problem Oriented Approach, 4th Edition,
Pages 377, 643.
Sample Previous Year Question on HIV based on previous Year Questions of
NEET PG, USMLE,PLAB,FMGE (MCI Screening). Please visit
www.medicoapps.org for more such Quizzes
Premature baby of 34 weeks was delivered. Baby had bullous lesion on the body. X ray
shows periostitis. What is the next investigation?
A: VDRL for mother & baby
B:
ELISA for
HIV
C: PCR for T.B
D: Hepatitis surface antigen for mother
Correct Ans:A
Explanation
This child is suffering from congenital syphilis. Here, this mother and infant require
quantitative nontreponemal (VDRL, RPR) syphilis testing.
The earliest manifestations of congenital syphilis (appearing 2–6 weeks after birth) include
rhinitis, or “snuffles” (23%); mucocutaneous lesions (35–41%); bone changes (61%),
including osteochondritis, osteitis, and periostitis detectable by x-ray examination of long
bones; hepatosplenomegaly (50%); lymphadenopathy (32%); anemia (34%); jaundice
(30%); thrombocytopenia; and leukocytosis.
Ref: Ogle J.W., Anderson M.S. (2012). Chapter 42. Infections: Bacterial & Spirochetal.In
Hay W.W., Jr, Levin M.J., Deterding R.R., Abzug M.J., Sondheimer J.M. (Eds),CURRENT
Diagnosis & Treatment: Pediatrics, 21e.
Sample Previous Year Question on HIV based on previous Year Questions of
NEET PG, USMLE,PLAB,FMGE (MCI Screening). Please visit
www.medicoapps.org for more such Quizzes
All of the following interventions are recommended to prevent mother to child transmission
of HIV, except:
A: Avoid Ergometrine in third stage of labour
B: Highly Active Antiretroviral Therapy (HAART)
C: Elective Caesarean section
D: Intrapartum Zidovudine
Correct Ans:A
Explanation
Ergometrine is give the third stage of labour to reduce post partum haemorrhage, it has no
role in the prevention of HIV transmission.
Hiv aids sample questions based on neet pg , usmle, plab and fmge pattern (mci screening) final
Hiv aids sample questions based on neet pg , usmle, plab and fmge pattern (mci screening) final
Hiv aids sample questions based on neet pg , usmle, plab and fmge pattern (mci screening) final
Hiv aids sample questions based on neet pg , usmle, plab and fmge pattern (mci screening) final
Hiv aids sample questions based on neet pg , usmle, plab and fmge pattern (mci screening) final
Hiv aids sample questions based on neet pg , usmle, plab and fmge pattern (mci screening) final
Hiv aids sample questions based on neet pg , usmle, plab and fmge pattern (mci screening) final
Hiv aids sample questions based on neet pg , usmle, plab and fmge pattern (mci screening) final

More Related Content

What's hot

Rheumatology MCQs Practice questions with explanation
Rheumatology MCQs Practice questions with explanationRheumatology MCQs Practice questions with explanation
Rheumatology MCQs Practice questions with explanation
Dr. Almas A
 
Pathology of Hepatitis - Quiz
Pathology of Hepatitis - QuizPathology of Hepatitis - Quiz
Pathology of Hepatitis - Quiz
Shashidhar Venkatesh Murthy
 
Infections in immunocompromised patients
Infections in immunocompromised patientsInfections in immunocompromised patients
Infections in immunocompromised patientsجهاد الخريصي
 
OSCE
OSCEOSCE
MCQs.docx
MCQs.docxMCQs.docx
MCQs.docx
IkramMuhammad10
 
Internal Medicine Board Review
Internal Medicine  Board ReviewInternal Medicine  Board Review
Internal Medicine Board Reviewjcm MD
 
Medicine ospe
Medicine ospeMedicine ospe
Medicine ospe
Dr. Sajid Ali Talpur
 
Dermatology MCQ and AAFP.pptx
Dermatology MCQ and AAFP.pptxDermatology MCQ and AAFP.pptx
Dermatology MCQ and AAFP.pptx
Abdulaziz Bagasi
 
Internal Medicine Board Review - Dermatology Flashcards - by Knowmedge
Internal Medicine Board Review - Dermatology Flashcards - by KnowmedgeInternal Medicine Board Review - Dermatology Flashcards - by Knowmedge
Internal Medicine Board Review - Dermatology Flashcards - by Knowmedge
Knowmedge
 
Git mcqs dr.ahmed_mowafy
Git mcqs dr.ahmed_mowafyGit mcqs dr.ahmed_mowafy
Git mcqs dr.ahmed_mowafy
czer Shmary
 
Pediatric Arab Board MCQ Review - Emergency Medicine
Pediatric Arab Board MCQ Review - Emergency Medicine Pediatric Arab Board MCQ Review - Emergency Medicine
Pediatric Arab Board MCQ Review - Emergency Medicine
Fatima Farid
 
Laboratory diagnosis of primary immunodeficiencies
Laboratory diagnosis of primary immunodeficienciesLaboratory diagnosis of primary immunodeficiencies
Laboratory diagnosis of primary immunodeficiencies
Chulalongkorn Allergy and Clinical Immunology Research Group
 
Approach to a child with suspected immunodefeciency
Approach to a child with suspected immunodefeciencyApproach to a child with suspected immunodefeciency
Approach to a child with suspected immunodefeciency
Nitin Pawar
 
500 single best answers in medicine
500 single best answers in medicine500 single best answers in medicine
500 single best answers in medicine
hamadadodo
 
Hyper-IgE syndrome
Hyper-IgE syndromeHyper-IgE syndrome
Microbiology MCQs
Microbiology MCQsMicrobiology MCQs
Microbiology MCQs
Pankaj Jha
 
Viral exanthems-module
Viral exanthems-moduleViral exanthems-module
Viral exanthems-module
pedgishih
 
Angioedema
AngioedemaAngioedema
Angioedema
Reem Alyahya
 
Diamond blackfan a nemia final
Diamond blackfan a nemia finalDiamond blackfan a nemia final
Diamond blackfan a nemia final
Dr. Liza Bulsara
 

What's hot (20)

Rheumatology MCQs Practice questions with explanation
Rheumatology MCQs Practice questions with explanationRheumatology MCQs Practice questions with explanation
Rheumatology MCQs Practice questions with explanation
 
Pathology of Hepatitis - Quiz
Pathology of Hepatitis - QuizPathology of Hepatitis - Quiz
Pathology of Hepatitis - Quiz
 
Infections in immunocompromised patients
Infections in immunocompromised patientsInfections in immunocompromised patients
Infections in immunocompromised patients
 
OSCE
OSCEOSCE
OSCE
 
MCQs.docx
MCQs.docxMCQs.docx
MCQs.docx
 
Internal Medicine Board Review
Internal Medicine  Board ReviewInternal Medicine  Board Review
Internal Medicine Board Review
 
Medicine ospe
Medicine ospeMedicine ospe
Medicine ospe
 
Dermatology MCQ and AAFP.pptx
Dermatology MCQ and AAFP.pptxDermatology MCQ and AAFP.pptx
Dermatology MCQ and AAFP.pptx
 
Internal Medicine Board Review - Dermatology Flashcards - by Knowmedge
Internal Medicine Board Review - Dermatology Flashcards - by KnowmedgeInternal Medicine Board Review - Dermatology Flashcards - by Knowmedge
Internal Medicine Board Review - Dermatology Flashcards - by Knowmedge
 
Git mcqs dr.ahmed_mowafy
Git mcqs dr.ahmed_mowafyGit mcqs dr.ahmed_mowafy
Git mcqs dr.ahmed_mowafy
 
Pediatric Arab Board MCQ Review - Emergency Medicine
Pediatric Arab Board MCQ Review - Emergency Medicine Pediatric Arab Board MCQ Review - Emergency Medicine
Pediatric Arab Board MCQ Review - Emergency Medicine
 
Laboratory diagnosis of primary immunodeficiencies
Laboratory diagnosis of primary immunodeficienciesLaboratory diagnosis of primary immunodeficiencies
Laboratory diagnosis of primary immunodeficiencies
 
Approach to a child with suspected immunodefeciency
Approach to a child with suspected immunodefeciencyApproach to a child with suspected immunodefeciency
Approach to a child with suspected immunodefeciency
 
Epstein-Barr Virus
Epstein-Barr Virus�Epstein-Barr Virus�
Epstein-Barr Virus
 
500 single best answers in medicine
500 single best answers in medicine500 single best answers in medicine
500 single best answers in medicine
 
Hyper-IgE syndrome
Hyper-IgE syndromeHyper-IgE syndrome
Hyper-IgE syndrome
 
Microbiology MCQs
Microbiology MCQsMicrobiology MCQs
Microbiology MCQs
 
Viral exanthems-module
Viral exanthems-moduleViral exanthems-module
Viral exanthems-module
 
Angioedema
AngioedemaAngioedema
Angioedema
 
Diamond blackfan a nemia final
Diamond blackfan a nemia finalDiamond blackfan a nemia final
Diamond blackfan a nemia final
 

Viewers also liked

HIV/AIDS Short Quiz
HIV/AIDS Short QuizHIV/AIDS Short Quiz
HIV/AIDS Short Quizsophiepen
 
Hiv aids sample questions based on neet pg , usmle, plab and fmge pattern (mc...
Hiv aids sample questions based on neet pg , usmle, plab and fmge pattern (mc...Hiv aids sample questions based on neet pg , usmle, plab and fmge pattern (mc...
Hiv aids sample questions based on neet pg , usmle, plab and fmge pattern (mc...Medico Apps
 
Sample Questionnaire
Sample QuestionnaireSample Questionnaire
Sample Questionnaire
Deighton Gooden
 
Level of awareness of HIV/AIDS
Level of awareness of HIV/AIDSLevel of awareness of HIV/AIDS
Aids awareness day
Aids awareness dayAids awareness day
Aids awareness day
VINOD KUMAR
 
Medical question on hiv aids early stage
Medical question on hiv aids early stageMedical question on hiv aids early stage
Medical question on hiv aids early stageOnline doctor
 
Hiv test
Hiv testHiv test
Hiv test
ainakadir
 
Usmle mindmaps
Usmle mindmapsUsmle mindmaps
Usmle mindmaps
medicmaps
 
Hiv aids sample questions based on neet pg , usmle, plab and fmge pattern (mc...
Hiv aids sample questions based on neet pg , usmle, plab and fmge pattern (mc...Hiv aids sample questions based on neet pg , usmle, plab and fmge pattern (mc...
Hiv aids sample questions based on neet pg , usmle, plab and fmge pattern (mc...Medico Apps
 
usmle step 1 content
usmle step 1 contentusmle step 1 content
Pharmacokinetics (updated 2011) - drdhriiti
Pharmacokinetics (updated 2011)  - drdhriitiPharmacokinetics (updated 2011)  - drdhriiti
Pharmacokinetics (updated 2011) - drdhriiti
http://neigrihms.gov.in/
 
Unmit hiv aids unit knowledge and awareness study oct 2010-final
Unmit hiv aids unit knowledge and awareness study oct 2010-finalUnmit hiv aids unit knowledge and awareness study oct 2010-final
Unmit hiv aids unit knowledge and awareness study oct 2010-final
Sudha Rathor
 
Pharmacodynamics
PharmacodynamicsPharmacodynamics
Pharmacodynamics
Khalid
 
Caso Clínico: Sífilis
Caso Clínico: SífilisCaso Clínico: Sífilis
Caso Clínico: Sífilis
UACH, Valdivia
 
Class factors modifying drug action
Class factors modifying drug actionClass factors modifying drug action
Class factors modifying drug action
Raghu Prasada
 
High yield enzymes for USMLE Step 1
High yield enzymes for USMLE Step 1High yield enzymes for USMLE Step 1
High yield enzymes for USMLE Step 1
Osman Altohamy
 
Presenting problems in HIV infection
Presenting problems in HIV infectionPresenting problems in HIV infection
Presenting problems in HIV infection
Dr Santosh Kumaraswamy
 
Inhalational anaesthetic agents
Inhalational anaesthetic agentsInhalational anaesthetic agents
Inhalational anaesthetic agentsgaganbrar18
 

Viewers also liked (20)

HIV/AIDS Short Quiz
HIV/AIDS Short QuizHIV/AIDS Short Quiz
HIV/AIDS Short Quiz
 
Hiv aids sample questions based on neet pg , usmle, plab and fmge pattern (mc...
Hiv aids sample questions based on neet pg , usmle, plab and fmge pattern (mc...Hiv aids sample questions based on neet pg , usmle, plab and fmge pattern (mc...
Hiv aids sample questions based on neet pg , usmle, plab and fmge pattern (mc...
 
Sample Questionnaire
Sample QuestionnaireSample Questionnaire
Sample Questionnaire
 
Level of awareness of HIV/AIDS
Level of awareness of HIV/AIDSLevel of awareness of HIV/AIDS
Level of awareness of HIV/AIDS
 
Aids awareness day
Aids awareness dayAids awareness day
Aids awareness day
 
Factors affecting drug development nd pharmacokinetics
Factors affecting drug development nd pharmacokineticsFactors affecting drug development nd pharmacokinetics
Factors affecting drug development nd pharmacokinetics
 
Medical question on hiv aids early stage
Medical question on hiv aids early stageMedical question on hiv aids early stage
Medical question on hiv aids early stage
 
AIDS QUIZ
AIDS QUIZAIDS QUIZ
AIDS QUIZ
 
Hiv test
Hiv testHiv test
Hiv test
 
Usmle mindmaps
Usmle mindmapsUsmle mindmaps
Usmle mindmaps
 
Hiv aids sample questions based on neet pg , usmle, plab and fmge pattern (mc...
Hiv aids sample questions based on neet pg , usmle, plab and fmge pattern (mc...Hiv aids sample questions based on neet pg , usmle, plab and fmge pattern (mc...
Hiv aids sample questions based on neet pg , usmle, plab and fmge pattern (mc...
 
usmle step 1 content
usmle step 1 contentusmle step 1 content
usmle step 1 content
 
Pharmacokinetics (updated 2011) - drdhriiti
Pharmacokinetics (updated 2011)  - drdhriitiPharmacokinetics (updated 2011)  - drdhriiti
Pharmacokinetics (updated 2011) - drdhriiti
 
Unmit hiv aids unit knowledge and awareness study oct 2010-final
Unmit hiv aids unit knowledge and awareness study oct 2010-finalUnmit hiv aids unit knowledge and awareness study oct 2010-final
Unmit hiv aids unit knowledge and awareness study oct 2010-final
 
Pharmacodynamics
PharmacodynamicsPharmacodynamics
Pharmacodynamics
 
Caso Clínico: Sífilis
Caso Clínico: SífilisCaso Clínico: Sífilis
Caso Clínico: Sífilis
 
Class factors modifying drug action
Class factors modifying drug actionClass factors modifying drug action
Class factors modifying drug action
 
High yield enzymes for USMLE Step 1
High yield enzymes for USMLE Step 1High yield enzymes for USMLE Step 1
High yield enzymes for USMLE Step 1
 
Presenting problems in HIV infection
Presenting problems in HIV infectionPresenting problems in HIV infection
Presenting problems in HIV infection
 
Inhalational anaesthetic agents
Inhalational anaesthetic agentsInhalational anaesthetic agents
Inhalational anaesthetic agents
 

Similar to Hiv aids sample questions based on neet pg , usmle, plab and fmge pattern (mci screening) final

Hiv aids sample questions based on neet pg , usmle, plab and fmge pattern (mc...
Hiv aids sample questions based on neet pg , usmle, plab and fmge pattern (mc...Hiv aids sample questions based on neet pg , usmle, plab and fmge pattern (mc...
Hiv aids sample questions based on neet pg , usmle, plab and fmge pattern (mc...Medico Apps
 
Stem cell transplantation for primary immunodeficiency diseases
Stem cell transplantation for primary immunodeficiency diseasesStem cell transplantation for primary immunodeficiency diseases
Stem cell transplantation for primary immunodeficiency diseases
Amir Abbas Hedayati Asl
 
CISPLATIN AND VITAMIN D.............pptx
CISPLATIN AND VITAMIN D.............pptxCISPLATIN AND VITAMIN D.............pptx
CISPLATIN AND VITAMIN D.............pptx
AlyaaKaram1
 
Previous year question on cataract based on neet pg, usmle, plab and fmge or ...
Previous year question on cataract based on neet pg, usmle, plab and fmge or ...Previous year question on cataract based on neet pg, usmle, plab and fmge or ...
Previous year question on cataract based on neet pg, usmle, plab and fmge or ...
Medico Apps
 
Contemporary Management of HIV.How Common Comorbidities Affect ART Management...
Contemporary Management of HIV.How Common Comorbidities Affect ART Management...Contemporary Management of HIV.How Common Comorbidities Affect ART Management...
Contemporary Management of HIV.How Common Comorbidities Affect ART Management...
hivlifeinfo
 
Previous year question on poisoninig part i and ii based on neet pg, usmle, p...
Previous year question on poisoninig part i and ii based on neet pg, usmle, p...Previous year question on poisoninig part i and ii based on neet pg, usmle, p...
Previous year question on poisoninig part i and ii based on neet pg, usmle, p...
Abhishek Gupta
 
Previous year question on staining based on neet pg, usmle, plab and fmge or ...
Previous year question on staining based on neet pg, usmle, plab and fmge or ...Previous year question on staining based on neet pg, usmle, plab and fmge or ...
Previous year question on staining based on neet pg, usmle, plab and fmge or ...
Abhishek Gupta
 
HCM - Egreso - Diarrea en Paciente con VIH
HCM - Egreso - Diarrea en Paciente con VIHHCM - Egreso - Diarrea en Paciente con VIH
HCM - Egreso - Diarrea en Paciente con VIH
guest40ed2d
 
Vitamind presentation advanced
Vitamind presentation  advancedVitamind presentation  advanced
Vitamind presentation advanced
BALASUBRAMANIAM IYER
 
REVIEW ON HIV ON THE CURRENT BASIS DATA .pptx
REVIEW ON HIV ON THE CURRENT BASIS DATA  .pptxREVIEW ON HIV ON THE CURRENT BASIS DATA  .pptx
REVIEW ON HIV ON THE CURRENT BASIS DATA .pptx
GCHANDRASAIKUMAR
 
HIV.ppt
HIV.pptHIV.ppt
Eds 16 10
Eds 16 10Eds 16 10
Eds 16 10
maha fahmy
 
Pathology of JC Virus
Pathology of JC VirusPathology of JC Virus
Pathology of JC Virus
Yonah Ziemba
 
Understanding Hiv Diagnostics And Lab Tests
Understanding Hiv Diagnostics And Lab TestsUnderstanding Hiv Diagnostics And Lab Tests
Understanding Hiv Diagnostics And Lab Tests
arthur_smith
 
Wiskott Aldrich Syndrome Final Powerpoint
Wiskott Aldrich Syndrome Final PowerpointWiskott Aldrich Syndrome Final Powerpoint
Wiskott Aldrich Syndrome Final Powerpoint
Jefferson School of Education
 
126371918 case-study-hiv-with-diarrhea
126371918 case-study-hiv-with-diarrhea126371918 case-study-hiv-with-diarrhea
126371918 case-study-hiv-with-diarrhea
homeworkping8
 
COMMON VARIABLE IMMUNODEFICIENCY (CVID)
COMMON VARIABLE IMMUNODEFICIENCY (CVID)COMMON VARIABLE IMMUNODEFICIENCY (CVID)
COMMON VARIABLE IMMUNODEFICIENCY (CVID)
Abdullatif Al-Rashed
 
Neurocognitive Changes in Newly Diagnosed Patient with Low CD4: Implications ...
Neurocognitive Changes in Newly Diagnosed Patient with Low CD4: Implications ...Neurocognitive Changes in Newly Diagnosed Patient with Low CD4: Implications ...
Neurocognitive Changes in Newly Diagnosed Patient with Low CD4: Implications ...
UC San Diego AntiViral Research Center
 
Previous year question on carcinma urinary bladder based on neet pg, usmle, p...
Previous year question on carcinma urinary bladder based on neet pg, usmle, p...Previous year question on carcinma urinary bladder based on neet pg, usmle, p...
Previous year question on carcinma urinary bladder based on neet pg, usmle, p...
Abhishek Gupta
 

Similar to Hiv aids sample questions based on neet pg , usmle, plab and fmge pattern (mci screening) final (20)

Hiv aids sample questions based on neet pg , usmle, plab and fmge pattern (mc...
Hiv aids sample questions based on neet pg , usmle, plab and fmge pattern (mc...Hiv aids sample questions based on neet pg , usmle, plab and fmge pattern (mc...
Hiv aids sample questions based on neet pg , usmle, plab and fmge pattern (mc...
 
Mmv sample-mc qs (1)
Mmv sample-mc qs (1)Mmv sample-mc qs (1)
Mmv sample-mc qs (1)
 
Stem cell transplantation for primary immunodeficiency diseases
Stem cell transplantation for primary immunodeficiency diseasesStem cell transplantation for primary immunodeficiency diseases
Stem cell transplantation for primary immunodeficiency diseases
 
CISPLATIN AND VITAMIN D.............pptx
CISPLATIN AND VITAMIN D.............pptxCISPLATIN AND VITAMIN D.............pptx
CISPLATIN AND VITAMIN D.............pptx
 
Previous year question on cataract based on neet pg, usmle, plab and fmge or ...
Previous year question on cataract based on neet pg, usmle, plab and fmge or ...Previous year question on cataract based on neet pg, usmle, plab and fmge or ...
Previous year question on cataract based on neet pg, usmle, plab and fmge or ...
 
Contemporary Management of HIV.How Common Comorbidities Affect ART Management...
Contemporary Management of HIV.How Common Comorbidities Affect ART Management...Contemporary Management of HIV.How Common Comorbidities Affect ART Management...
Contemporary Management of HIV.How Common Comorbidities Affect ART Management...
 
Previous year question on poisoninig part i and ii based on neet pg, usmle, p...
Previous year question on poisoninig part i and ii based on neet pg, usmle, p...Previous year question on poisoninig part i and ii based on neet pg, usmle, p...
Previous year question on poisoninig part i and ii based on neet pg, usmle, p...
 
Previous year question on staining based on neet pg, usmle, plab and fmge or ...
Previous year question on staining based on neet pg, usmle, plab and fmge or ...Previous year question on staining based on neet pg, usmle, plab and fmge or ...
Previous year question on staining based on neet pg, usmle, plab and fmge or ...
 
HCM - Egreso - Diarrea en Paciente con VIH
HCM - Egreso - Diarrea en Paciente con VIHHCM - Egreso - Diarrea en Paciente con VIH
HCM - Egreso - Diarrea en Paciente con VIH
 
Vitamind presentation advanced
Vitamind presentation  advancedVitamind presentation  advanced
Vitamind presentation advanced
 
REVIEW ON HIV ON THE CURRENT BASIS DATA .pptx
REVIEW ON HIV ON THE CURRENT BASIS DATA  .pptxREVIEW ON HIV ON THE CURRENT BASIS DATA  .pptx
REVIEW ON HIV ON THE CURRENT BASIS DATA .pptx
 
HIV.ppt
HIV.pptHIV.ppt
HIV.ppt
 
Eds 16 10
Eds 16 10Eds 16 10
Eds 16 10
 
Pathology of JC Virus
Pathology of JC VirusPathology of JC Virus
Pathology of JC Virus
 
Understanding Hiv Diagnostics And Lab Tests
Understanding Hiv Diagnostics And Lab TestsUnderstanding Hiv Diagnostics And Lab Tests
Understanding Hiv Diagnostics And Lab Tests
 
Wiskott Aldrich Syndrome Final Powerpoint
Wiskott Aldrich Syndrome Final PowerpointWiskott Aldrich Syndrome Final Powerpoint
Wiskott Aldrich Syndrome Final Powerpoint
 
126371918 case-study-hiv-with-diarrhea
126371918 case-study-hiv-with-diarrhea126371918 case-study-hiv-with-diarrhea
126371918 case-study-hiv-with-diarrhea
 
COMMON VARIABLE IMMUNODEFICIENCY (CVID)
COMMON VARIABLE IMMUNODEFICIENCY (CVID)COMMON VARIABLE IMMUNODEFICIENCY (CVID)
COMMON VARIABLE IMMUNODEFICIENCY (CVID)
 
Neurocognitive Changes in Newly Diagnosed Patient with Low CD4: Implications ...
Neurocognitive Changes in Newly Diagnosed Patient with Low CD4: Implications ...Neurocognitive Changes in Newly Diagnosed Patient with Low CD4: Implications ...
Neurocognitive Changes in Newly Diagnosed Patient with Low CD4: Implications ...
 
Previous year question on carcinma urinary bladder based on neet pg, usmle, p...
Previous year question on carcinma urinary bladder based on neet pg, usmle, p...Previous year question on carcinma urinary bladder based on neet pg, usmle, p...
Previous year question on carcinma urinary bladder based on neet pg, usmle, p...
 

More from Medico Apps

Previous year question on hepatomegaly based on neet pg, usmle, plab and fmge...
Previous year question on hepatomegaly based on neet pg, usmle, plab and fmge...Previous year question on hepatomegaly based on neet pg, usmle, plab and fmge...
Previous year question on hepatomegaly based on neet pg, usmle, plab and fmge...
Medico Apps
 
Previous year question on rabies based on neet pg, usmle, plab and fmge or mc...
Previous year question on rabies based on neet pg, usmle, plab and fmge or mc...Previous year question on rabies based on neet pg, usmle, plab and fmge or mc...
Previous year question on rabies based on neet pg, usmle, plab and fmge or mc...
Medico Apps
 
Previous year question on medulloblastoma based on neet pg, usmle, plab and f...
Previous year question on medulloblastoma based on neet pg, usmle, plab and f...Previous year question on medulloblastoma based on neet pg, usmle, plab and f...
Previous year question on medulloblastoma based on neet pg, usmle, plab and f...
Medico Apps
 
Previous year question on pharyngeal arches embryology based on neet pg, usml...
Previous year question on pharyngeal arches embryology based on neet pg, usml...Previous year question on pharyngeal arches embryology based on neet pg, usml...
Previous year question on pharyngeal arches embryology based on neet pg, usml...
Medico Apps
 
Previous year question on pemphigus vulgaris based on neet pg, usmle, plab an...
Previous year question on pemphigus vulgaris based on neet pg, usmle, plab an...Previous year question on pemphigus vulgaris based on neet pg, usmle, plab an...
Previous year question on pemphigus vulgaris based on neet pg, usmle, plab an...
Medico Apps
 
Previous year question on leptospirosis based on neet pg, usmle, plab and fmg...
Previous year question on leptospirosis based on neet pg, usmle, plab and fmg...Previous year question on leptospirosis based on neet pg, usmle, plab and fmg...
Previous year question on leptospirosis based on neet pg, usmle, plab and fmg...
Medico Apps
 
Previous year question on glycolysis based on neet pg, usmle, plab and fmge o...
Previous year question on glycolysis based on neet pg, usmle, plab and fmge o...Previous year question on glycolysis based on neet pg, usmle, plab and fmge o...
Previous year question on glycolysis based on neet pg, usmle, plab and fmge o...
Medico Apps
 
Previous year question on glycolysis based on neet pg, usmle, plab and fmge o...
Previous year question on glycolysis based on neet pg, usmle, plab and fmge o...Previous year question on glycolysis based on neet pg, usmle, plab and fmge o...
Previous year question on glycolysis based on neet pg, usmle, plab and fmge o...
Medico Apps
 
Previous year question on bone cyst based on neet pg, usmle, plab and fmge or...
Previous year question on bone cyst based on neet pg, usmle, plab and fmge or...Previous year question on bone cyst based on neet pg, usmle, plab and fmge or...
Previous year question on bone cyst based on neet pg, usmle, plab and fmge or...
Medico Apps
 
Previous year question on ketamine based on neet pg, usmle, plab and fmge or ...
Previous year question on ketamine based on neet pg, usmle, plab and fmge or ...Previous year question on ketamine based on neet pg, usmle, plab and fmge or ...
Previous year question on ketamine based on neet pg, usmle, plab and fmge or ...
Medico Apps
 
Ketamine (anaesthesia )sample questions based on neet pg , usmle, plab and fm...
Ketamine (anaesthesia )sample questions based on neet pg , usmle, plab and fm...Ketamine (anaesthesia )sample questions based on neet pg , usmle, plab and fm...
Ketamine (anaesthesia )sample questions based on neet pg , usmle, plab and fm...Medico Apps
 
Radiology most important signs sample questions based on neet pg , usmle, pla...
Radiology most important signs sample questions based on neet pg , usmle, pla...Radiology most important signs sample questions based on neet pg , usmle, pla...
Radiology most important signs sample questions based on neet pg , usmle, pla...
Medico Apps
 
Hiv aids sample questions based on neet pg , usmle, plab and fmge pattern (mc...
Hiv aids sample questions based on neet pg , usmle, plab and fmge pattern (mc...Hiv aids sample questions based on neet pg , usmle, plab and fmge pattern (mc...
Hiv aids sample questions based on neet pg , usmle, plab and fmge pattern (mc...Medico Apps
 
Meconium stained baby sample questions based on neet pg , usmle, plab and fmg...
Meconium stained baby sample questions based on neet pg , usmle, plab and fmg...Meconium stained baby sample questions based on neet pg , usmle, plab and fmg...
Meconium stained baby sample questions based on neet pg , usmle, plab and fmg...
Medico Apps
 
Pancreatitis sample questions based on neet pg , usmle, plab and fmge pattern...
Pancreatitis sample questions based on neet pg , usmle, plab and fmge pattern...Pancreatitis sample questions based on neet pg , usmle, plab and fmge pattern...
Pancreatitis sample questions based on neet pg , usmle, plab and fmge pattern...
Medico Apps
 
Alzheimers disease sample questions based on neet pg , usmle, plab and fmge p...
Alzheimers disease sample questions based on neet pg , usmle, plab and fmge p...Alzheimers disease sample questions based on neet pg , usmle, plab and fmge p...
Alzheimers disease sample questions based on neet pg , usmle, plab and fmge p...
Medico Apps
 
Cystic fibrosis sample questions based on neet pg , usmle, plab and fmge patt...
Cystic fibrosis sample questions based on neet pg , usmle, plab and fmge patt...Cystic fibrosis sample questions based on neet pg , usmle, plab and fmge patt...
Cystic fibrosis sample questions based on neet pg , usmle, plab and fmge patt...
Medico Apps
 
Phrenic nerve sample questions based on neet pg , usmle, plab and fmge patter...
Phrenic nerve sample questions based on neet pg , usmle, plab and fmge patter...Phrenic nerve sample questions based on neet pg , usmle, plab and fmge patter...
Phrenic nerve sample questions based on neet pg , usmle, plab and fmge patter...
Medico Apps
 
Epilepsy questions for neet pg, usmle, plab , fmge (mci screening)
Epilepsy questions for neet pg, usmle, plab , fmge (mci screening)Epilepsy questions for neet pg, usmle, plab , fmge (mci screening)
Epilepsy questions for neet pg, usmle, plab , fmge (mci screening)
Medico Apps
 
Bartholin's glands sample mcq for neet pg, usmle , plab and fmge or mci scree...
Bartholin's glands sample mcq for neet pg, usmle , plab and fmge or mci scree...Bartholin's glands sample mcq for neet pg, usmle , plab and fmge or mci scree...
Bartholin's glands sample mcq for neet pg, usmle , plab and fmge or mci scree...
Medico Apps
 

More from Medico Apps (20)

Previous year question on hepatomegaly based on neet pg, usmle, plab and fmge...
Previous year question on hepatomegaly based on neet pg, usmle, plab and fmge...Previous year question on hepatomegaly based on neet pg, usmle, plab and fmge...
Previous year question on hepatomegaly based on neet pg, usmle, plab and fmge...
 
Previous year question on rabies based on neet pg, usmle, plab and fmge or mc...
Previous year question on rabies based on neet pg, usmle, plab and fmge or mc...Previous year question on rabies based on neet pg, usmle, plab and fmge or mc...
Previous year question on rabies based on neet pg, usmle, plab and fmge or mc...
 
Previous year question on medulloblastoma based on neet pg, usmle, plab and f...
Previous year question on medulloblastoma based on neet pg, usmle, plab and f...Previous year question on medulloblastoma based on neet pg, usmle, plab and f...
Previous year question on medulloblastoma based on neet pg, usmle, plab and f...
 
Previous year question on pharyngeal arches embryology based on neet pg, usml...
Previous year question on pharyngeal arches embryology based on neet pg, usml...Previous year question on pharyngeal arches embryology based on neet pg, usml...
Previous year question on pharyngeal arches embryology based on neet pg, usml...
 
Previous year question on pemphigus vulgaris based on neet pg, usmle, plab an...
Previous year question on pemphigus vulgaris based on neet pg, usmle, plab an...Previous year question on pemphigus vulgaris based on neet pg, usmle, plab an...
Previous year question on pemphigus vulgaris based on neet pg, usmle, plab an...
 
Previous year question on leptospirosis based on neet pg, usmle, plab and fmg...
Previous year question on leptospirosis based on neet pg, usmle, plab and fmg...Previous year question on leptospirosis based on neet pg, usmle, plab and fmg...
Previous year question on leptospirosis based on neet pg, usmle, plab and fmg...
 
Previous year question on glycolysis based on neet pg, usmle, plab and fmge o...
Previous year question on glycolysis based on neet pg, usmle, plab and fmge o...Previous year question on glycolysis based on neet pg, usmle, plab and fmge o...
Previous year question on glycolysis based on neet pg, usmle, plab and fmge o...
 
Previous year question on glycolysis based on neet pg, usmle, plab and fmge o...
Previous year question on glycolysis based on neet pg, usmle, plab and fmge o...Previous year question on glycolysis based on neet pg, usmle, plab and fmge o...
Previous year question on glycolysis based on neet pg, usmle, plab and fmge o...
 
Previous year question on bone cyst based on neet pg, usmle, plab and fmge or...
Previous year question on bone cyst based on neet pg, usmle, plab and fmge or...Previous year question on bone cyst based on neet pg, usmle, plab and fmge or...
Previous year question on bone cyst based on neet pg, usmle, plab and fmge or...
 
Previous year question on ketamine based on neet pg, usmle, plab and fmge or ...
Previous year question on ketamine based on neet pg, usmle, plab and fmge or ...Previous year question on ketamine based on neet pg, usmle, plab and fmge or ...
Previous year question on ketamine based on neet pg, usmle, plab and fmge or ...
 
Ketamine (anaesthesia )sample questions based on neet pg , usmle, plab and fm...
Ketamine (anaesthesia )sample questions based on neet pg , usmle, plab and fm...Ketamine (anaesthesia )sample questions based on neet pg , usmle, plab and fm...
Ketamine (anaesthesia )sample questions based on neet pg , usmle, plab and fm...
 
Radiology most important signs sample questions based on neet pg , usmle, pla...
Radiology most important signs sample questions based on neet pg , usmle, pla...Radiology most important signs sample questions based on neet pg , usmle, pla...
Radiology most important signs sample questions based on neet pg , usmle, pla...
 
Hiv aids sample questions based on neet pg , usmle, plab and fmge pattern (mc...
Hiv aids sample questions based on neet pg , usmle, plab and fmge pattern (mc...Hiv aids sample questions based on neet pg , usmle, plab and fmge pattern (mc...
Hiv aids sample questions based on neet pg , usmle, plab and fmge pattern (mc...
 
Meconium stained baby sample questions based on neet pg , usmle, plab and fmg...
Meconium stained baby sample questions based on neet pg , usmle, plab and fmg...Meconium stained baby sample questions based on neet pg , usmle, plab and fmg...
Meconium stained baby sample questions based on neet pg , usmle, plab and fmg...
 
Pancreatitis sample questions based on neet pg , usmle, plab and fmge pattern...
Pancreatitis sample questions based on neet pg , usmle, plab and fmge pattern...Pancreatitis sample questions based on neet pg , usmle, plab and fmge pattern...
Pancreatitis sample questions based on neet pg , usmle, plab and fmge pattern...
 
Alzheimers disease sample questions based on neet pg , usmle, plab and fmge p...
Alzheimers disease sample questions based on neet pg , usmle, plab and fmge p...Alzheimers disease sample questions based on neet pg , usmle, plab and fmge p...
Alzheimers disease sample questions based on neet pg , usmle, plab and fmge p...
 
Cystic fibrosis sample questions based on neet pg , usmle, plab and fmge patt...
Cystic fibrosis sample questions based on neet pg , usmle, plab and fmge patt...Cystic fibrosis sample questions based on neet pg , usmle, plab and fmge patt...
Cystic fibrosis sample questions based on neet pg , usmle, plab and fmge patt...
 
Phrenic nerve sample questions based on neet pg , usmle, plab and fmge patter...
Phrenic nerve sample questions based on neet pg , usmle, plab and fmge patter...Phrenic nerve sample questions based on neet pg , usmle, plab and fmge patter...
Phrenic nerve sample questions based on neet pg , usmle, plab and fmge patter...
 
Epilepsy questions for neet pg, usmle, plab , fmge (mci screening)
Epilepsy questions for neet pg, usmle, plab , fmge (mci screening)Epilepsy questions for neet pg, usmle, plab , fmge (mci screening)
Epilepsy questions for neet pg, usmle, plab , fmge (mci screening)
 
Bartholin's glands sample mcq for neet pg, usmle , plab and fmge or mci scree...
Bartholin's glands sample mcq for neet pg, usmle , plab and fmge or mci scree...Bartholin's glands sample mcq for neet pg, usmle , plab and fmge or mci scree...
Bartholin's glands sample mcq for neet pg, usmle , plab and fmge or mci scree...
 

Recently uploaded

Physiology of Special Chemical Sensation of Taste
Physiology of Special Chemical Sensation of TastePhysiology of Special Chemical Sensation of Taste
Physiology of Special Chemical Sensation of Taste
MedicoseAcademics
 
POST OPERATIVE OLIGURIA and its management
POST OPERATIVE OLIGURIA and its managementPOST OPERATIVE OLIGURIA and its management
POST OPERATIVE OLIGURIA and its management
touseefaziz1
 
The Normal Electrocardiogram - Part I of II
The Normal Electrocardiogram - Part I of IIThe Normal Electrocardiogram - Part I of II
The Normal Electrocardiogram - Part I of II
MedicoseAcademics
 
New Directions in Targeted Therapeutic Approaches for Older Adults With Mantl...
New Directions in Targeted Therapeutic Approaches for Older Adults With Mantl...New Directions in Targeted Therapeutic Approaches for Older Adults With Mantl...
New Directions in Targeted Therapeutic Approaches for Older Adults With Mantl...
i3 Health
 
Novas diretrizes da OMS para os cuidados perinatais de mais qualidade
Novas diretrizes da OMS para os cuidados perinatais de mais qualidadeNovas diretrizes da OMS para os cuidados perinatais de mais qualidade
Novas diretrizes da OMS para os cuidados perinatais de mais qualidade
Prof. Marcus Renato de Carvalho
 
Surat @ℂall @Girls ꧁❤8527049040❤꧂@ℂall @Girls Service Vip Top Model Safe
Surat @ℂall @Girls ꧁❤8527049040❤꧂@ℂall @Girls Service Vip Top Model SafeSurat @ℂall @Girls ꧁❤8527049040❤꧂@ℂall @Girls Service Vip Top Model Safe
Surat @ℂall @Girls ꧁❤8527049040❤꧂@ℂall @Girls Service Vip Top Model Safe
Savita Shen $i11
 
Tom Selleck Health: A Comprehensive Look at the Iconic Actor’s Wellness Journey
Tom Selleck Health: A Comprehensive Look at the Iconic Actor’s Wellness JourneyTom Selleck Health: A Comprehensive Look at the Iconic Actor’s Wellness Journey
Tom Selleck Health: A Comprehensive Look at the Iconic Actor’s Wellness Journey
greendigital
 
Couples presenting to the infertility clinic- Do they really have infertility...
Couples presenting to the infertility clinic- Do they really have infertility...Couples presenting to the infertility clinic- Do they really have infertility...
Couples presenting to the infertility clinic- Do they really have infertility...
Sujoy Dasgupta
 
Maxilla, Mandible & Hyoid Bone & Clinical Correlations by Dr. RIG.pptx
Maxilla, Mandible & Hyoid Bone & Clinical Correlations by Dr. RIG.pptxMaxilla, Mandible & Hyoid Bone & Clinical Correlations by Dr. RIG.pptx
Maxilla, Mandible & Hyoid Bone & Clinical Correlations by Dr. RIG.pptx
Dr. Rabia Inam Gandapore
 
Cervical & Brachial Plexus By Dr. RIG.pptx
Cervical & Brachial Plexus By Dr. RIG.pptxCervical & Brachial Plexus By Dr. RIG.pptx
Cervical & Brachial Plexus By Dr. RIG.pptx
Dr. Rabia Inam Gandapore
 
Pulmonary Thromboembolism - etilogy, types, medical- Surgical and nursing man...
Pulmonary Thromboembolism - etilogy, types, medical- Surgical and nursing man...Pulmonary Thromboembolism - etilogy, types, medical- Surgical and nursing man...
Pulmonary Thromboembolism - etilogy, types, medical- Surgical and nursing man...
VarunMahajani
 
BENIGN PROSTATIC HYPERPLASIA.BPH. BPHpdf
BENIGN PROSTATIC HYPERPLASIA.BPH. BPHpdfBENIGN PROSTATIC HYPERPLASIA.BPH. BPHpdf
BENIGN PROSTATIC HYPERPLASIA.BPH. BPHpdf
DR SETH JOTHAM
 
Ocular injury ppt Upendra pal optometrist upums saifai etawah
Ocular injury  ppt  Upendra pal  optometrist upums saifai etawahOcular injury  ppt  Upendra pal  optometrist upums saifai etawah
Ocular injury ppt Upendra pal optometrist upums saifai etawah
pal078100
 
Prix Galien International 2024 Forum Program
Prix Galien International 2024 Forum ProgramPrix Galien International 2024 Forum Program
Prix Galien International 2024 Forum Program
Levi Shapiro
 
ACUTE SCROTUM.....pdf. ACUTE SCROTAL CONDITIOND
ACUTE SCROTUM.....pdf. ACUTE SCROTAL CONDITIONDACUTE SCROTUM.....pdf. ACUTE SCROTAL CONDITIOND
ACUTE SCROTUM.....pdf. ACUTE SCROTAL CONDITIOND
DR SETH JOTHAM
 
Phone Us ❤85270-49040❤ #ℂall #gIRLS In Surat By Surat @ℂall @Girls Hotel With...
Phone Us ❤85270-49040❤ #ℂall #gIRLS In Surat By Surat @ℂall @Girls Hotel With...Phone Us ❤85270-49040❤ #ℂall #gIRLS In Surat By Surat @ℂall @Girls Hotel With...
Phone Us ❤85270-49040❤ #ℂall #gIRLS In Surat By Surat @ℂall @Girls Hotel With...
Savita Shen $i11
 
Triangles of Neck and Clinical Correlation by Dr. RIG.pptx
Triangles of Neck and Clinical Correlation by Dr. RIG.pptxTriangles of Neck and Clinical Correlation by Dr. RIG.pptx
Triangles of Neck and Clinical Correlation by Dr. RIG.pptx
Dr. Rabia Inam Gandapore
 
Physiology of Chemical Sensation of smell.pdf
Physiology of Chemical Sensation of smell.pdfPhysiology of Chemical Sensation of smell.pdf
Physiology of Chemical Sensation of smell.pdf
MedicoseAcademics
 
heat stroke and heat exhaustion in children
heat stroke and heat exhaustion in childrenheat stroke and heat exhaustion in children
heat stroke and heat exhaustion in children
SumeraAhmad5
 
ARTIFICIAL INTELLIGENCE IN HEALTHCARE.pdf
ARTIFICIAL INTELLIGENCE IN  HEALTHCARE.pdfARTIFICIAL INTELLIGENCE IN  HEALTHCARE.pdf
ARTIFICIAL INTELLIGENCE IN HEALTHCARE.pdf
Anujkumaranit
 

Recently uploaded (20)

Physiology of Special Chemical Sensation of Taste
Physiology of Special Chemical Sensation of TastePhysiology of Special Chemical Sensation of Taste
Physiology of Special Chemical Sensation of Taste
 
POST OPERATIVE OLIGURIA and its management
POST OPERATIVE OLIGURIA and its managementPOST OPERATIVE OLIGURIA and its management
POST OPERATIVE OLIGURIA and its management
 
The Normal Electrocardiogram - Part I of II
The Normal Electrocardiogram - Part I of IIThe Normal Electrocardiogram - Part I of II
The Normal Electrocardiogram - Part I of II
 
New Directions in Targeted Therapeutic Approaches for Older Adults With Mantl...
New Directions in Targeted Therapeutic Approaches for Older Adults With Mantl...New Directions in Targeted Therapeutic Approaches for Older Adults With Mantl...
New Directions in Targeted Therapeutic Approaches for Older Adults With Mantl...
 
Novas diretrizes da OMS para os cuidados perinatais de mais qualidade
Novas diretrizes da OMS para os cuidados perinatais de mais qualidadeNovas diretrizes da OMS para os cuidados perinatais de mais qualidade
Novas diretrizes da OMS para os cuidados perinatais de mais qualidade
 
Surat @ℂall @Girls ꧁❤8527049040❤꧂@ℂall @Girls Service Vip Top Model Safe
Surat @ℂall @Girls ꧁❤8527049040❤꧂@ℂall @Girls Service Vip Top Model SafeSurat @ℂall @Girls ꧁❤8527049040❤꧂@ℂall @Girls Service Vip Top Model Safe
Surat @ℂall @Girls ꧁❤8527049040❤꧂@ℂall @Girls Service Vip Top Model Safe
 
Tom Selleck Health: A Comprehensive Look at the Iconic Actor’s Wellness Journey
Tom Selleck Health: A Comprehensive Look at the Iconic Actor’s Wellness JourneyTom Selleck Health: A Comprehensive Look at the Iconic Actor’s Wellness Journey
Tom Selleck Health: A Comprehensive Look at the Iconic Actor’s Wellness Journey
 
Couples presenting to the infertility clinic- Do they really have infertility...
Couples presenting to the infertility clinic- Do they really have infertility...Couples presenting to the infertility clinic- Do they really have infertility...
Couples presenting to the infertility clinic- Do they really have infertility...
 
Maxilla, Mandible & Hyoid Bone & Clinical Correlations by Dr. RIG.pptx
Maxilla, Mandible & Hyoid Bone & Clinical Correlations by Dr. RIG.pptxMaxilla, Mandible & Hyoid Bone & Clinical Correlations by Dr. RIG.pptx
Maxilla, Mandible & Hyoid Bone & Clinical Correlations by Dr. RIG.pptx
 
Cervical & Brachial Plexus By Dr. RIG.pptx
Cervical & Brachial Plexus By Dr. RIG.pptxCervical & Brachial Plexus By Dr. RIG.pptx
Cervical & Brachial Plexus By Dr. RIG.pptx
 
Pulmonary Thromboembolism - etilogy, types, medical- Surgical and nursing man...
Pulmonary Thromboembolism - etilogy, types, medical- Surgical and nursing man...Pulmonary Thromboembolism - etilogy, types, medical- Surgical and nursing man...
Pulmonary Thromboembolism - etilogy, types, medical- Surgical and nursing man...
 
BENIGN PROSTATIC HYPERPLASIA.BPH. BPHpdf
BENIGN PROSTATIC HYPERPLASIA.BPH. BPHpdfBENIGN PROSTATIC HYPERPLASIA.BPH. BPHpdf
BENIGN PROSTATIC HYPERPLASIA.BPH. BPHpdf
 
Ocular injury ppt Upendra pal optometrist upums saifai etawah
Ocular injury  ppt  Upendra pal  optometrist upums saifai etawahOcular injury  ppt  Upendra pal  optometrist upums saifai etawah
Ocular injury ppt Upendra pal optometrist upums saifai etawah
 
Prix Galien International 2024 Forum Program
Prix Galien International 2024 Forum ProgramPrix Galien International 2024 Forum Program
Prix Galien International 2024 Forum Program
 
ACUTE SCROTUM.....pdf. ACUTE SCROTAL CONDITIOND
ACUTE SCROTUM.....pdf. ACUTE SCROTAL CONDITIONDACUTE SCROTUM.....pdf. ACUTE SCROTAL CONDITIOND
ACUTE SCROTUM.....pdf. ACUTE SCROTAL CONDITIOND
 
Phone Us ❤85270-49040❤ #ℂall #gIRLS In Surat By Surat @ℂall @Girls Hotel With...
Phone Us ❤85270-49040❤ #ℂall #gIRLS In Surat By Surat @ℂall @Girls Hotel With...Phone Us ❤85270-49040❤ #ℂall #gIRLS In Surat By Surat @ℂall @Girls Hotel With...
Phone Us ❤85270-49040❤ #ℂall #gIRLS In Surat By Surat @ℂall @Girls Hotel With...
 
Triangles of Neck and Clinical Correlation by Dr. RIG.pptx
Triangles of Neck and Clinical Correlation by Dr. RIG.pptxTriangles of Neck and Clinical Correlation by Dr. RIG.pptx
Triangles of Neck and Clinical Correlation by Dr. RIG.pptx
 
Physiology of Chemical Sensation of smell.pdf
Physiology of Chemical Sensation of smell.pdfPhysiology of Chemical Sensation of smell.pdf
Physiology of Chemical Sensation of smell.pdf
 
heat stroke and heat exhaustion in children
heat stroke and heat exhaustion in childrenheat stroke and heat exhaustion in children
heat stroke and heat exhaustion in children
 
ARTIFICIAL INTELLIGENCE IN HEALTHCARE.pdf
ARTIFICIAL INTELLIGENCE IN  HEALTHCARE.pdfARTIFICIAL INTELLIGENCE IN  HEALTHCARE.pdf
ARTIFICIAL INTELLIGENCE IN HEALTHCARE.pdf
 

Hiv aids sample questions based on neet pg , usmle, plab and fmge pattern (mci screening) final

  • 1. A 25 year old female, known to be infected with HIV and a CD4 count 90 presented with complaints of altered sensorium following a partial seizure. Gadolinium enhanced MRI scan showed multiple ring enhancing lesions in both hemispheres, thalamus, pons, basal ganglia and cerebellum. The most probable diagnosis in this scenario is? A: Mycobacterium tuberculosis B: Multiple brain metastases C: Cryptococcal meningitis D: Pneumocystis infiltration of brain Correct Ans:A Explanation HIV patients with low CD 4 counts if presents with multiple ring enhancing lesions is suggestive of Tuberculoma, Primary CNS lymphoma and Toxoplasmosis. In this scenario only TB is the given choice and hence the answer. Sample Previous Year Question on HIV based on previous Year Questions of NEET PG, USMLE,PLAB,FMGE (MCI Screening). Please visit www.medicoapps.org for more such Quizzes A preterm baby developed bullous lesion on the skin soon after birth. The X- ray shows periostitis. What should be the next investigation? A: VDRL for mother & baby B: ELISA for HIV C: PCR for TB D: Hepatitis surface antigen for mother Correct Ans:A Explanation The baby described in the question stem is most likely suffering from the early manifestations of congenital syphilis. As per the choices given VDRL is the test done to diagnose syphilis.
  • 2. Ref: Harrison’s Principles of Internal Medicine, 16th Edition, Page 981; Skin Diseases & Sexually Transmitted Infections, Uday Khopkar – 6th Edition, Page 257; Nelson 17th ,COGDT 10th Edition, Page 667;CPDT 18th Edition, Page 56. Sample Previous Year Question on HIV based on previous Year Questions of NEET PG, USMLE,PLAB,FMGE (MCI Screening). Please visit www.medicoapps.org for more such Quizzes Thalidomide can be used in all of the following conditions, EXCEPT: A: Behcet's syndrome B: HIV associated peripheral neuropathy C: HIV associated mouth ulcers D: Erythema nodosum leprosum Correct Ans:B Explanation Thalidomide has been used for many years in the treatment of some manifestations of leprosy and has been used for erythema nodosum leprosum; it is also useful in management of the skin manifestations of lupus erythematosus. The most important toxicity is teratogenesis. Because of this effect, thalidomide prescription and use are closely regulated by the manufacturer. Other adverse effects of thalidomide include peripheral neuropathy, constipation, rash, fatigue, hypothyroidism, and increased risk of deep vein thrombosis. In more serious cases of Behcet syndrome, thalidomide (100 mg/d) is effective. Ref: Lake D.F., Briggs A.D., Akporiaye E.T. (2012). Chapter 55. Immunopharmacology. In B.G. Katzung, S.B. Masters, A.J. Trevor (Eds), Basic & Clinical Pharmacology, 12e. Sample Previous Year Question on HIV based on previous Year Questions of NEET PG, USMLE,PLAB,FMGE (MCI Screening). Please visit www.medicoapps.org for more such Quizzes All are drugs used against HIV, EXCEPT: A: Elvucitabine B: Bevirima t C: Maraviroc D: Tegaserod
  • 3. Correct Ans:D Explanation New therapies are continually being sought for HIV that exploit new viral targets, have activity against resistant viral strains, have a lower incidence of adverse effects, and offer convenient dosing. New agents of existing classes that are currently in advanced stages of clinical development include the NRTI agent elvucitabine, the NNRTI agents TMC-278 and IDX899, the PI agent bracanavir, entry inhibitors such as the CCR5 receptor antagonists vicriviroc and PRO 140, and integrase inhibitors such as elvitegravir. In addition, new drug classes such as maturation inhibitors (bevirimat) and the CD4 receptor inhibitor TNX-355 are under investigation. Ref: Katzung 11th edition Chapter 49. Sample Previous Year Question on HIV based on previous Year Questions of NEET PG, USMLE,PLAB,FMGE (MCI Screening). Please visit www.medicoapps.org for more such Quizzes Maraviroc is used in treatment of: A: CMV B: HS V C: HIV D: Measles Correct Ans:C Explanation Maraviroc binds specifically and selectively to CCR5, one of two coreceptors necessary for entrance of HIV into CD4+ cells, thus blocking entry of CCR5-tropic HIV into these cells. Ref: Katzung 11th edition Chapter 49. Sample Previous Year Question on HIV based on previous Year Questions of NEET PG, USMLE,PLAB,FMGE (MCI Screening). Please visit www.medicoapps.org for more such Quizzes Lamivudine acts against which of the following viruses? A: HIV
  • 4. B: HBV C: Both of the above D: None of the above Correct Ans:C Explanation Lamivudine inhibits HBV DNA polymerase and HIV reverse transcriptase by competing with deoxycytidine triphosphate for incorporation into the viral DNA, resulting in chain termination. Lamivudine achieves 3–4 log decreases in viral replication in most patients and suppression of HBV DNA to undetectable levels in about 44% of patients. Ref: Katzung 11th edition Chapter 49. Sample Previous Year Question on HIV based on previous Year Questions of NEET PG, USMLE,PLAB,FMGE (MCI Screening). Please visit www.medicoapps.org for more such Quizzes A 15 yr female with HIV infection on anti retroviral therapy presents with nausea vomiting and flank pain. Urine microscopy showed rectangular plate and needle shaped crystals. Serum creatine was raised. What will be the likely cause? A: Indinavi r B: Ritonavir C: Nevirapine D: Zidovudine Correct Ans:A Explanation Crystal- induced nephropathy: Indinavir has been commonly associated with crystal- induced nephropathy. Crystallization of the HIV protease inhibitor indinavir within the renal tubules is associated with a range of urinary tract manifestations, including nephrolithiasis, asymptomatic crystalluria, crystalluria with dysuria (or renal colic), and crystal nephropathy, is influenced by alkaline urine. Other drus that may cause crystal nephropathy are acyclovir, atazanavir and sulfazadine methotrexate Ref: CURRENT Diagnosis & Treatment Nephrology & Hypertension - Page 125. Sample Previous Year Question on HIV based on previous Year Questions of NEET PG, USMLE,PLAB,FMGE (MCI Screening). Please visit www.medicoapps.org for more such Quizzes
  • 5. Metformin is used as a antihyperglycemic agent in DM, but there are some other uses for this drug. All the following are such uses of the drug, except: A: PCOD B: Non-alcoholic fatty liver disease C: Alcoholic fatty liver disease D: Metabolic abnormalities associated with HIV disease Correct Ans:C Explanation Other uses of metformin: • PCOD • Impaired glucose tolerance • Obesity • Metabolic abnormalities associated with HIV disease • Non alcoholic fatty liver disease Ref: Harrison, 18th Edition, Page 2995 Sample Previous Year Question on HIV based on previous Year Questions of NEET PG, USMLE,PLAB,FMGE (MCI Screening). Please visit www.medicoapps.org for more such Quizzes Thalidomide is used in all, EXCEPT: A: Erythema nodosum leprosum B: HIV neuropathy C: HIV associated ulcer D: Multiple myeloma Correct Ans:B Explanation Thalidomide is indicated for the treatment of patients with erythema nodosum leprosum and multiple myeloma. In addition, it has orphan drug status for mycobacterial infections, Crohn's disease, HIV-associated wasting, Kaposi sarcoma, lupus, myelofibrosis, brain malignancies, leprosy, graft-versus-host disease, and aphthous ulcers. Also Know: Thalidomide should never be taken by women who are pregnant or who could become pregnant while taking the drug.It is best known for the severe, life-threatening birth defects it caused when administered to pregnant women.
  • 6. Sample Previous Year Question on HIV based on previous Year Questions of NEET PG, USMLE,PLAB,FMGE (MCI Screening). Please visit www.medicoapps.org for more such Quizzes The important coreceptors for HIV to bind with CD4 receptors are: A: CCR4 and CXCR3 B: CCR5 and CXCR4 C: CCR4 and CXCR5 D: CCR4 and CXCR2 Correct Ans:B Explanation The primary cellular receptor for HIV is CD4 (Helper subset of T cells). HIV has two major coreceptors: CCR5 and CXCR4 (for fusion and entry) ALSO KNOW: Kaposi's sarcoma cannot be explained completely by the immunodeficiency caused by HIV infection. Ref: Harrison, Edition-18, Page-1519. Sample Previous Year Question on HIV based on previous Year Questions of NEET PG, USMLE,PLAB,FMGE (MCI Screening). Please visit www.medicoapps.org for more such Quizzes Neurologic abnormalities have been noted in about one-third of patients with AIDS. Which of the following is NOT seen in HIV involvement of CNS? A: Perivascular giant cell B: Vacuolar degeneration of post column C: Microglial nodule formation D: Inclusion bodies
  • 7. Correct Ans:D Explanation The main cell types that are infected in the brain in HIV infection are the perivascular macrophages and the microglial cells. A diffuse and multifocal rarefaction of the cerebral white matter accompanied by scanty perivascular infiltrates of lymphocytes and clusters of a few foamy macrophages, microglial nodules, and multinucleated giant cells. Vacuolar degeneration of posterior column is associated with the AIDS dementia complex. Ref: Ropper A.H., Samuels M.A. (2009). Chapter 33. Viral Infections of the Nervous System, Chronic Meningitis, and Prion Diseases. In A.H. Ropper, M.A. Samuels (Eds), Adams and Victor's Principles of Neurology, 9e. Sample Previous Year Question on HIV based on previous Year Questions of NEET PG, USMLE,PLAB,FMGE (MCI Screening). Please visit www.medicoapps.org for more such Quizzes Which of the following disease cause modification of finger prints? A: Leprosy B: HI V C: Hypertension D: Acromegaly Correct Ans:A Explanation Leprosy can modify finger prints. Electrical injury and radiation can also modify finger prints. In criminal cases impression of all 10 fingers are taken but in civil cases left thumb impression is taken in case of males and right thumb impression in case of females. Dactylography or finger print system consist of taking the impression of the pulp of the fingers and thumb with printers ink on an unglazed white paper. The fingerprints of even identical twins are different. The chances of two persons having identical finger prints is about one in thirty times the population of the world. Ref: Concise Textbook Of Forensic Medicine & Toxicology By Sharma, page 19.
  • 8. Sample Previous Year Question on HIV based on previous Year Questions of NEET PG, USMLE,PLAB,FMGE (MCI Screening). Please visit www.medicoapps.org for more such Quizzes Identify the CORRECT description of Molotov cocktail: A: Petrol bomb B: Mixture of alcohol and Cocaine C: Combination of drugs used for HIV treatment D: Combination of drugs used by Cocaine addicts Correct Ans:A Explanation A Molotov cocktail is also known as a petrol bomb or alcohol bomb. It is a simple type of improvised incendiary device. The simplest form consists of a stoppered bottle filled with a combustible liquid, such as gasoline or high-proof alcohol, with a fuel-soaked rag stuffed in the neck of the bottle. Sample Previous Year Question on HIV based on previous Year Questions of NEET PG, USMLE,PLAB,FMGE (MCI Screening). Please visit www.medicoapps.org for more such Quizzes Sentinel laboratories are mainly involved in: A: External Quality Assessment of laboratories B: Diagnosis of HIV infection C: Control of Bioterrorism D: Diagnosis of Tuberculosis Correct Ans:C Explanation Because communication between clinical and public health laboratories is so critical in the post-2001 era, Centre for Disease Control (CDC) created an organizational structure whereby isolates and lines of communication flow freely among numbers. The Laboratory Response Network (LRN) was originally designed as a four-tier system. Clinical laboratories (Level A) were at the bottom of a triangle, sending specimens to a public health laboratory (level B) or Level C
  • 9. (state public health laboratory) for confirmation. Isolates were ultimately sent to CDC or United States Army Medical Research Institute for Infectious Diseases (USMRIID)-Level D, for archiving ang sophisticated molecular testing. This original system has now been changed to a three-tier system in which Level A laboratories are now called Sentinel laboratories; Level B & C laboratories are called Reference laboratories & Level D are called National laboratories. The main role of Sentinel Microbiology laboratories is to raise suspicion when a targeted agent is suspected in a human specimen. Detection of a possible bioterrorism event will depend on: • A laboratory having an active microbial surveillance and monitoring program • Vigilant technologists looking for a disease that does not occur naturally in a particular geographic region (eg: Plague in New York city); is transmitted by an aerosol route of infection; is a single case of disease caused by an unusual agent (eg: Burkholderia mallei usually only seen in the far east); Good communication with infection control practitioners, infectious disease Physicians and local or regional public health laboratories. Ref: Bailey & Scott’s Diagnostic Microbiology; 12th edition; Page: 954-955 Sample Previous Year Question on HIV based on previous Year Questions of NEET PG, USMLE,PLAB,FMGE (MCI Screening). Please visit www.medicoapps.org for more such Quizzes One of the most common causes of diarrhea in HIV infected patients is cryptosporidiosis. Which of the following is the method used for diagnosis of cryptosporidiosis in fecal matter? A: Ziehl-Neelsen stain of the stool B: Gram stain of the stool C: Normal saline suspension of stool D: Iodine suspension of stool Correct Ans:A Explanation
  • 10. Cryptosporidiosis is a protozoan which is most commonly involved in the pathogenesis of diarrhea in HIV infected patients. The cysts for cryptosporidiosis are best demonstrated by Ziehl-Neelsen stain of fecal mater. Other important clinical laboratory tests for stool are summarized as follows. Analysis of Stool: 1. Physical examination for color, frank blood, undigested food, worms etc. 2. Chemical examination of Reducing substance for lactose intolerance, of Fecal fat for malabsorption syndrome, of Protein content for Protein losing enteropathy. Occult blood test. 3. Direct light microscopy for demonstration of RBC, WBC,Crystals and Ova of different types of helminths. Demonstration of protozoan like E. histolytica and Giardia. 4. Culture of stool for Salmonella, Shigella, Campylobacter, etc. 5. Assay for demonstration toxins in stool in cases of patients suffering from Pseudomembranous colitis (toxin of Cl. difficile). 6. ELISA- This test it is able to find viral antigens (Rotavirus) as a causative agent for infantile diarrhea. 7. Electron microscopy for demonstration of virus (Rotavirus). 8. Gram/Z-N stain. Z-N staining of stool smear is highly diagnostic for demonstration of Cryptosporidiosis (protozoa) responsible for diarrhea in an immunocompromised host (AIDS). Ref: Brooks G.F. (2013). Chapter 46. Medical Parasitology. In G.F. Brooks (Ed),Jawetz, Melnick, & Adelberg's Medical Microbiology, 26e. Sample Previous Year Question on HIV based on previous Year Questions of NEET PG, USMLE,PLAB,FMGE (MCI Screening). Please visit www.medicoapps.org for more such Quizzes Which of the following causes highest risk of nosocomial infection to a patient? A: Patient admitted for elective surgery B: HIV patient coming in follow up OPD C: Patient undergoing endoscopy D: Patient admitted for normal delivery Correct Ans:A Explanation
  • 11. The fact that at least 25–50% of nosocomial infections are due to the combined effect of the patient's own flora and invasive devices highlights the importance of improvements in the use and design of such devices. Urinary tract infections (UTIs) account for ~34% of nosocomial infections. Pneumonia accounts for ~13% of nosocomial infections. Wound infections account for ~17% of nosocomial infections UTI is the most common cause of nosocomial infection. Other common cause is pneumonia and surgical procedures. Contaminated surgical equipments and from health care workers can be the cause of infection in surgical wound. Also Know: The most common pathogens in postoperative wound infections are S. aureus, coagulase-negative staphylococci, and enteric and anaerobic bacteria. In rapidly progressing postoperative infections, which manifest within 24–48 h of a surgical procedure, the level of suspicion regarding group A streptococcal or clostridial infection should be high. Ref: Weinstein R.A. (2012). Chapter 131. Health Care–Associated Infections. In D.L. Longo, A.S. Fauci, D.L. Kasper, S.L. Hauser, J.L. Jameson, J. Loscalzo (Eds), Harrison's Principles of Internal Medicine, 18e. Sample Previous Year Question on HIV based on previous Year Questions of NEET PG, USMLE,PLAB,FMGE (MCI Screening). Please visit www.medicoapps.org for more such Quizzes FALSE statement ragarding HIV-associated TB is: A: Extrapulmonary TB is common among HIV-infected patients B: The diagnosis of TB in HIV-infected patients may be difficult C: Immune reconstitution inflammatory syndrome (IRIS) is more common among patients with advanced immunosuppression and extra pulmonary TB D: Patients with both HIV infection and TB are more infectious than persons without HIV co- infection Correct Ans:D Explanation The most infectious patients have cavitary pulmonary disease. Patients with sputum smear–negative and those with culture-negative pulmonary TB and extrapulmonary TB are essentially noninfectious. Because persons with both HIV
  • 12. infection and TB are less likely to have cavitations, and they are less likely to be smear positive they may be less infectious than persons without HIV co-infection. Ref: Harrisons principles of internal medicine, 18th edition, Page: 1342. Sample Previous Year Question on HIV based on previous Year Questions of NEET PG, USMLE,PLAB,FMGE (MCI Screening). Please visit www.medicoapps.org for more such Quizzes Regarding HIV infection, not true is: A: p24 is used for early diagnosis B: Lysis of infected CD4 cells is seen C: Dendritic cells do not support replication D: Macrophage is a reservoir for the virus Correct Ans:C Explanation Follicular dendritic cells from tonsils, can be infected with HIV even without the involvement of CD4. Ref: Text Book of Microbilogy By Ananthanarayan, 6th Edition, Page 546 Sample Previous Year Question on HIV based on previous Year Questions of NEET PG, USMLE,PLAB,FMGE (MCI Screening). Please visit www.medicoapps.org for more such Quizzes 'H5 Nl' may be best described as a: A: Bird flu virus B: Vaccine for HIV C: Agent for Japanese encephalitis D: New strain of Plasmodium falciparum Correct Ans:A Explanation
  • 13. H5N1 is the Avian Flue a type of Influenza A virus, also called the Bird Flu Virus. Ref: Molecular Medicine: An Introductory Text By Trent, 3rd Edition, Pages 209-10; Clinical Infectious Disease By Schlossberg, 2008, Page 1290 Sample Previous Year Question on HIV based on previous Year Questions of NEET PG, USMLE,PLAB,FMGE (MCI Screening). Please visit www.medicoapps.org for more such Quizzes The genome of HIV virus contains which of the following? A: Single stranded DNA B: Single stranded RNA C: Double stranded DNA D: Double stranded RNA Correct Ans:B Explanation HIV is an enveloped icosahedral sphere. It has 2 identical, non complementary strands of RNA and 3 enzymes (reverse transcriptase, integrase and protease) packed in a cone shaped protein core. This core is surrounded by a protein coat called capsid. This capsid along with the enclosed nucleic acid is called nucleocapsid. Capsid acts as a protective shell around the nucleic acid core and helps to introduce the viral genome into host cell by adsorbing readily to host cell surfaces. Most RNA viruses are single stranded except reoviridae which is double stranded. Most DNA virus are double stranded except poxviridae which is single stranded. Ref: HIV and AIDS: Basic Elements and Priorities By S. Karthikeyan, Pages 41- 3
  • 14. Sample Previous Year Question on HIV based on previous Year Questions of NEET PG, USMLE,PLAB,FMGE (MCI Screening). Please visit www.medicoapps.org for more such Quizzes Regarding HIV which of the following statement is not TRUE? A: It is a DNA retrovirus B: Contains Reverse Transcriptase C: May infect host CD4 cells other than T-lymphocytes D: Causes a reduction in host CD4 cells at late stage of disease Correct Ans:A Explanation The genome of HIV is diploid, composed of 2 identical single stranded positive sense RNA copies. In association with viral RNA is the reverse transcriptase enzyme which is the characteristic feature of retroviruses. Pathogenesis: When virus infects a cell RNA, viral RNA is transcribed by the enzyme, first to single stranded DNA and then to double stranded DNA which is integrated into the host cell chromosome. The provirus initiate viral replication by directing synthesis of viral RNA and other components. When naked virus buds outs through host cell surface membrane, it acquires a lipoprotein envelope which consist of lipid derived from the host cell membrane and glycoprotein which are virus coded. The spikes which constitute the major surface component of the virus, binds to CD4 receptors on susceptible host cells. The transmembrane pedicle cause cell fusion. Ref: Ananthanarayan and Panicker’s Textbook of Microbiology, 8th Edition, Page 395 Sample Previous Year Question on HIV based on previous Year Questions of NEET PG, USMLE,PLAB,FMGE (MCI Screening). Please visit www.medicoapps.org for more such Quizzes
  • 15. CMV retinitis in HIV occurs when the CD4 counts fall below which of the following levels? A: 50 B: 100 C: 200 D: 150 Correct Ans:A Explanation CMV Retinitis is the most common cause of loss of vision in AIDS patients. Majority of cases of CMV retinitis occur in patients with a CD4+ T cell count <50/micro L. Therefore, patients at high risk of CMV retinitis (CD4+ T cell count <100/micro L) should undergo an ophthalmologic examination every 3–6 months. CMV retinitis usually presents as a painless, progressive loss of vision. Patients may also complain of blurred vision, floaters and scintillations. It is usually bilateral. The characteristic retinal appearance is presence of perivascular hemorrhage and exudate. Treatment consists of oral valacyclovir, ganciclovir, IV ganciclovir, or IV foscarnet, with cidofovir as an alternative. Ref: Textbook of Microbiology By Ananthanarayan and Panicker, 8th Edition, Page 573 ; Harrison’s Principles of Internal Medicine, 18th Edition, Chapter 189 Sample Previous Year Question on HIV based on previous Year Questions of NEET PG, USMLE,PLAB,FMGE (MCI Screening). Please visit www.medicoapps.org for more such Quizzes Most common genital lesion in HIV patient is: A: Chlamydia B: Herpe s C: Syphilis D: Candida Correct Ans:B Explanation
  • 16. Herpes simplex virus is the is the most common opportunistic infection causing genital lesions in HIV patients. Herpes simplex virus type I and Type II cause recurrent oral, labial, genital and perianal herpes simplex lesions in HIV infected individuals. Herpes simplex virus type II (HSV2) is associated with genital lesions. Ref: Diagnosis and Management of HIV/AIDS : A Clinician's Perspective By Usha K. Baveja, B. B. Rewari, Page 93; Centers for Disease Control and Prevention. 2006 Sexually Transmitted Diseases Treatment Guidelines. MMWR Recomm Rep 2006; 55(RR-11), Pages 1 - 94. Sample Previous Year Question on HIV based on previous Year Questions of NEET PG, USMLE,PLAB,FMGE (MCI Screening). Please visit www.medicoapps.org for more such Quizzes Which of the following has the highest chance of transmission of HIV? A: Anal intercourse B: Transfusion of blood products C: Kidney transplant D: Transplacental transmission Correct Ans:B Explanation The approximate chance of infection/exposure is given below of various scenario a) Sexual intercourse i.e oral, anal or vaginal : 0.1 – 1.0 % b) Blood and blood product transfusion : >90% c) Tissue and organ donation : 5 to 90 % d) Sharing needles by drug addicts : 0.5 – 1.0 % e) Mother to baby : 30 % Ref: Textbook of microbiology By Ananthanarayanan, 6th Edition, Page 551 Sample Previous Year Question on HIV based on previous Year Questions of NEET PG, USMLE,PLAB,FMGE (MCI Screening). Please visit www.medicoapps.org for more such Quizzes
  • 17. A 28-year-old HIV-positive male complains of pain on swallowing. Physical examination is remarkable for white plaque-like material on his tongue and buccal mucosa, which is scraped and sent to the laboratory. Based on these findings, and on the laboratory results, the man is diagnosed with acquired immunodeficiency syndrome (AIDS). With which of the following agents is the man most likely infected? A: Candida albicans B: Cytomegaloviru s C: Herpes simplex I D: Human herpesvirus 8 Correct Ans:A Explanation Candida albicans produces oral thrush, an AIDS-defining lesion, which is common in acute HIV disease, and becomes increasing common as the CD4 + cell count falls. The lesions are usually painless. Diagnosis is by demonstration of pseudohyphae using a wet smear with confirmation by culture. Although cytomegalovirus is associated with numerous clinical scenarios in the AIDS population, including odynophagia (painful swallowing), it would not produce white plaques on the oral mucosa. Herpes Simplex Iproduces vesicular lesions occurring in clusters in the oral cavity. There is an increased risk of herpes infections in the AIDS group, but the lesions do not resemble those described in the question. Human herpesvirus 8 is the causative agent of Kaposi's sarcoma, a malignancy arising from endothelial cells that appears as hemorrhagic nodules in different organ systems. It is the most common cancer in the HIV infected population. Sample Previous Year Question on HIV based on previous Year Questions of NEET PG, USMLE,PLAB,FMGE (MCI Screening). Please visit www.medicoapps.org for more such Quizzes
  • 18. Which of the following is the MOST common cause of sporadic viral encephalitis in adults? A: Herpes virus B: Enteroviruses C: Ebstein-Barr virus D: HIV Correct Ans:A Explanation The most commonly identified viruses causing sporadic cases of acute encephalitis in adults are herpesviruses (HSV, VZV, EBV). Note Frequently Asked: Enteroviruses (coxsackieviruses, echoviruses, and human enteroviruses 68–71) are the most commonly identified viruses causing sporadic cases of acute meningitis. Ref: Roos K.L., Tyler K.L. (2012). Chapter 381. Meningitis, Encephalitis, Brain Abscess, and Empyema. In D.L. Longo, A.S. Fauci, D.L. Kasper, S.L. Hauser, J.L. Jameson, J. Loscalzo (Eds), Harrison's Principles of Internal Medicine, 18e Sample Previous Year Question on HIV based on previous Year Questions of NEET PG, USMLE,PLAB,FMGE (MCI Screening). Please visit www.medicoapps.org for more such Quizzes HIV virus has a particularly high affinity for which of the following? A: CD4 T Lymphocytes and B cells B: CD4 T Lymphocytes and monocytes C: CD8 T Lymphocytes and B cells D: CD8 T Lymphocytes and CD4 T Lymphocytes Correct Ans:B Explanation HIV has a particular tendency to bind to CD4 T lymphocytes (helper T lymphocytes) and monocytes and then become internalized. Most other cells, include CD8 T lymphocytes (suppressor T cells) and B cells, are not particularly susceptible to the HIV virus.
  • 19. HIV preferentially infects and kills helper (CD4) T lymphocytes, resulting in the loss of cell-mediated immunity and a high probability that the host will developopportunistic infections. Other cells (e.g., macrophages and monocytes) that have CD4 proteins on their surfaces can be infected also. Also know: The genome of HIV consists of two identical molecules of single-stranded, positive-polarity RNA and is said to be diploid. In addition to the three typical retroviral genes gag, pol, and env, which encode the structural proteins, the genome RNA has six regulatory genes. Two of these regulatory genes, tat and rev, are required for replication, and the other four, nef, vif, vpr, and vpu, are not required for replication and are termed “accessory" genes. Ref: Levinson W. (2012). Chapter 45. Human Immunodeficiency Virus. In W. Levinson (Ed), Review of Medical Microbiology & Immunology, 12e. Sample Previous Year Question on HIV based on previous Year Questions of NEET PG, USMLE,PLAB,FMGE (MCI Screening). Please visit www.medicoapps.org for more such Quizzes What is the chance of HIV infection after needle prick injury? A: 1/100 B: 1/30 0 C: 1/10000 D: 1 in 1 Lakh Correct Ans:B Explanation Needle prick injury: HIV transmission following skin puncture from a contaminated needle is 0.3 % and a mucous membrane exposure is 0.09%. The seroconversion rate for HIV is 1:300, or 0.3% from a hollow-bore needle stick injury. Antiretroviral drugs as postexposure prophylaxis decreases the risk of infection. Must know:
  • 20. In Hepatitis B, the seroconversion ranges from 23 to 62%, with higher rates associated with patients who are HBeAg positive, as compared to HBeAG negative. The seroconversion rate for Hepatitis C is 1.8%.For Hepatitis C no prophylactic treatment is available. HBV post exposure prophlylaxis with hepatitis B immunoglobulin and initiation of Hep B vaccine is 90%effective. Ref: Harrison’s Principles of internal medicine, 16th edition, page 1081. Sample Previous Year Question on HIV based on previous Year Questions of NEET PG, USMLE,PLAB,FMGE (MCI Screening). Please visit www.medicoapps.org for more such Quizzes A 24-year-old man is infected with HIV during a sexual encounter with a prostitute. He does not notice any symptoms in the period immediately after the encounter. During the asymptomatic latent phase of his infection, the virus is actively proliferating, and can be found in association with which of the following? A: B lymphocytes B: Follicular dendritic cells in lymph nodes C: Ganglion cells D: Oligodendrocytes Correct Ans:B Explanation Follicular dendritic cells in the germinal centers of lymph nodes are important reservoirs of HIV. Although some follicular dendritic cells are infected with HIV, most viral particles are found on the surface of their dendritic processes. Follicular dendritic cells have receptors to the Fc portion of immunoglobulins that serve to trap HIV virions coated with anti-HIV antibodies. These coated HIV particles retain the ability to infect CD4+ T cells as they traverse the dendritic cells. B lymphocytes have a surface marker (CD21 protein-a complement receptor) to which an Epstein-Barr envelope glycoprotein can bind. The virus associates with the host cell genome, producing a latent infection. These B cells undergo polyclonal activation and proliferation.
  • 21. Ganglion cells, particularly the satellite cells around the ganglion cells in the dorsal root ganglia, can be infected by varicella-zoster. Herpes type I and II infect neurons that innervate skin and mucous membranes. Oligodendrocytes are directly infected by two viruses, JC virus (a polyomavirus) and measles virus. JC virus causes progressive multifocal leukoencephalopathy (PML), and measles virus produces a latent syndrome called subacute sclerosing panencephalitis (SSPE). Ref: Levinson W. (2012). Chapter 58. Cellular Basis of the Immune Response. In W. Levinson (Ed), Review of Medical Microbiology & Immunology, 12e. Sample Previous Year Question on HIV based on previous Year Questions of NEET PG, USMLE,PLAB,FMGE (MCI Screening). Please visit www.medicoapps.org for more such Quizzes During pre-operative investigations, a man found to have positive serological results for HIV. Which of the following is highly specific test for HIV antibodies in this patient? A: ELISA B: Southern blot C: Northern blot D: Western blot Correct Ans:D Explanation The Western blot test is used as a measure of specific HIV-1 antibodies to confirm a positive ELISA result. The criterion for a positive test is any two bands corresponding to p24, gp41, and gp120/160. The absence of bands is a negative result, whereas the presence of bands that do not meet the criterion for a positive test is an indeterminate result. False-positive and false-negative results are relatively uncommon. HIV-2-infected patients may give an unusual pattern on an HIV-1 Western blot; therefore, if HIV-2 is suspected, a separate HIV-2 Western blot is required. Ref: Brooks G.F., Carroll K.C., Butel J.S., Morse S.A., Mietzner T.A. (2013). Chapter 47. Principles of Diagnostic Medical Microbiology. In G.F. Brooks, K.C.
  • 22. Carroll, J.S. Butel, S.A. Morse, T.A. Mietzner (Eds), Jawetz, Melnick, & Adelberg's Medical Microbiology, 26e. Sample Previous Year Question on HIV based on previous Year Questions of NEET PG, USMLE,PLAB,FMGE (MCI Screening). Please visit www.medicoapps.org for more such Quizzes A 28-year-old HIV-positive male complains of pain on swallowing. Physical examination is remarkable for white plaque-like material on his tongue and buccal mucosa, which is scraped and sent to the laboratory. Based on these findings, and on the laboratory results, the man is diagnosed with acquired immunodeficiency syndrome (AIDS). With which of the following agents is the man most likely infected? A: Candida albicans B: Cytomegaloviru s C: Herpes simplex I D: Human herpesvirus 8 Correct Ans:A Explanation Candida albicans produces oral thrush, an AIDS-defining lesion, which is common in acute HIV disease, and becomes increasing common as the CD4 + cell count falls. The lesions are usually painless. Diagnosis is by demonstration of pseudohyphae using a wet smear with confirmation by culture. Although cytomegalovirus is associated with numerous clinical scenarios in the AIDS population, including odynophagia (painful swallowing), it would not produce white plaques on the oral mucosa. Herpes Simplex Iproduces vesicular lesions occurring in clusters in the oral cavity. There is an increased risk of herpes infections in the AIDS group, but the lesions do not resemble those described in the question. Human herpesvirus 8 is the causative agent of Kaposi's sarcoma, a malignancy arising from endothelial cells that appears as hemorrhagic nodules in different organ systems. It is the most common cancer in the HIV infected population.
  • 23. Sample Previous Year Question on HIV based on previous Year Questions of NEET PG, USMLE,PLAB,FMGE (MCI Screening). Please visit www.medicoapps.org for more such Quizzes With needle stick injury, there is risk of transmitting all of the following infections, EXCEPT: A: HIV B: HBV C: HCV D: HDV Correct Ans:D Explanation Most common infections spread through needle stick injury are HIV, HBV and HCV. HDV infection occur only in the presence of HBV. It is transmitted through parenteral exposure or intimate contact. Vertical HDV transmission is rare. • HBV is the most infectious of the 3 viruses. • There is about 2% risk of acquiring HCV infection after needle stick injury. The is no vaccine against HCV and no treatment can prevent infection after exposure to the virus. • The risk of acquiring HIV is less than 0.5% after a single needle stick injury involving a patient known to have HIV/AIDS in a healthcare setting. • The risk of HIV transmission from an infected patients blood or other body fluid splashed into a caregivers eyes or mouth is much lower than for needle stick injuries. Ref: Levin M.J., Weinberg A. (2012). Chapter 40. Infections: Viral & Rickettsial. In W.W. Hay, Jr., M.J. Levin, R.R. Deterding, J.J. Ross, J.M. Sondheimer (Eds),CURRENT Diagnosis & Treatment: Pediatrics, 21e, Sokol R.J., Narkewicz M.R. (2012). Chapter 22. Liver & Pancreas. In W.W. Hay, Jr., M.J. Levin, R.R. Deterding, J.J. Ross, J.M. Sondheimer (Eds),CURRENT Diagnosis & Treatment: Pediatrics, 21e. Sample Previous Year Question on HIV based on previous Year Questions of NEET PG, USMLE,PLAB,FMGE (MCI Screening). Please visit www.medicoapps.org for more such Quizzes
  • 24. What are the chances of contracting HIV after needle prick injury? A: 1/100 B: 1/30 0 C: 1/10000 D: I in 1 Lac Correct Ans:B Explanation The average risk of HIV infection following needle prick injury is 1/300. After mucocutaneous exposure risk is approximately 0.09%. Post exposure prophylaxis should be initiated within 1-2 hrs. According CDC guidelines: • A basic regimen consists of two-drug therapy, often consisting of azidothymidine and lamivudine. • In an expanded regimen: indinavir or nelfinavir is added to it. • HBV is the most infectious of the 3 viruses. • There is about 2% risk of acquiring HCV infection after needle stick injury. The is no vaccine against HCV and no treatment can prevent infection after exposure to the virus. • The risk of HIV transmission from an infected patients blood or other body fluid splashed into a caregivers eyes or mouth is much lower than for needle stick injuries. Ref: Rothman R.E., Marco C.A., Yang S. (2011). Chapter 149. Human Immunodeficiency Virus Infection and Acquired Immunodeficiency Syndrome. In R.K. Cydulka, G.D. Meckler (Eds), Tintinalli's Emergency Medicine: A Comprehensive Study Guide, 7e. Sample Previous Year Question on HIV based on previous Year Questions of NEET PG, USMLE,PLAB,FMGE (MCI Screening). Please visit www.medicoapps.org for more such Quizzes Primary Receptor for Human Immunodeficiency Virus (HIV) is: A: CD4 B: CD
  • 25. 8 C: CD3 D: CD56 Correct Ans:A Explanation HIV targets cells that express CD4, and can infect macrophages, dendritic cells (both groups express CD4 at low levels) and CD4+ T cells. Ref: Andrew E. Williams, “Immunology of Urogenital Tract and Conjunctiva”, In the book, “Immunology: Mucosal and Body Surface Defences”, USA, Wiley Blackwell Publications, Chapter 9.1; Harrison’s Principles of Internal Medicine, 17th Edition, Pages 1139, 1157; Textbook of Microbiology By Ananthnaraynan, 7th Edition, Page 586 Sample Previous Year Question on HIV based on previous Year Questions of NEET PG, USMLE,PLAB,FMGE (MCI Screening). Please visit www.medicoapps.org for more such Quizzes Most common genital lesion in HIV patient is: A: Chlamydia B: Herpe s C: Syphilis D: Candida Correct Ans:B Explanation Herpes simplex virus is the is the most common opportunistic infection causing genital lesions in HIV patients. Herpes simplex virus type I and Type II cause recurrent oral, labial, genital and perianal herpes simplex lesions in HIV infected individuals. Herpes simplex virus type II (HSV2) is associated with genital lesions. Ref: Diagnosis and Management of HIV/AIDS : A Clinician's Perspective By Usha K. Baveja, B. B. Rewari, Page 93; Centers for Disease Control and Prevention. 2006 Sexually Transmitted Diseases Treatment Guidelines. MMWR Recomm Rep 2006; 55(RR-11), Pages 1 - 94.
  • 26. Sample Previous Year Question on HIV based on previous Year Questions of NEET PG, USMLE,PLAB,FMGE (MCI Screening). Please visit www.medicoapps.org for more such Quizzes A 38-year-old man has been HIV-positive since the age of 26 but has refused antiretroviral prophylaxis. His CD4 cell count, however, is still within normal limits, and the virus is undetectable in the blood. Genetic investigations clarify this apparent puzzle, demonstrating that this subject is homozygous for a mutation affecting a chemokine receptor necessary for HIV entry into the cell. Which of the following receptors is most likely involved? A: CCR5 B: CD 4 C: CD8 D: Gp120 Correct Ans:A Explanation HIV infection does not always progress to overt AIDS. In a few fortunate individuals (called non-progressors), the infection remains asymptomatic, with a stable CD4 cell count and low or undetectable viremia. It is not entirely clear whether these patients will eventually develop AIDS. Some are resistant to HIV infection because they carry two defective copies of the gene encoding the CCR5 coreceptor. There are 15 known coreceptors, of which CCR5 and CXCR4 are the best characterized. CCR5 is a receptor for β-chemokines and serves as a coreceptor for HIV internalization. Approximately one percent of Caucasians in the U.S. is homozygous for such a protective CCR5 variant, and 20% are heterozygotes. HIV infection follows a slower course in heterozygotes. Homozygotes for this protective CCR5 variant are not present among Black and Asian populations. Recently, a variant haplotype of CXCR1 (a leukocyte receptor for fractalkine) has been linked to rapid progression of HIV infection in homozygous individuals. Thus, investigations on HIV coreceptors are beginning to explain the variability in progression and severity of HIV infection among individuals. CD4 (2nd Choice) is an important receptor of helper T lymphocytes,
  • 27. macrophages, and Langerhans cells. It acts as the primary receptor for HIV binding. Binding to CD4, however, is not sufficient for the entry of HIV into the cells. The viral envelope gp120 protein (4th Choice) first binds CD4. This results in a conformational change in gp120 that allows this protein to bind coreceptors. Such coreceptors are present on lymphocytes and macrophages and normally serve as binding sites for chemokines. Once gp120 binds to CD4 and a coreceptor, a second viral protein, gp41, facilitates fusion of the viral envelope with the plasma membrane, allowing "injection" of the viral core into the cell. Neither CD8 (5th Choice) nor the receptor for TNF-α plays a role in HIV binding to cells. Sample Previous Year Question on HIV based on previous Year Questions of NEET PG, USMLE,PLAB,FMGE (MCI Screening). Please visit www.medicoapps.org for more such Quizzes A 24-year-old sex worker from Mumbai develops chronic abdominal pain, low- grade fever, diarrhea, and malabsorption. He saw later found to be positive for HIV. Oocysts are demonstrated in the stool. Which of the following organisms is most likely to be the cause of the patient's diarrhea? A: Diphyllobothrium latum B: Entamoeba histolytica C: Giardia lamblia D: Isospora belli Correct Ans:D Explanation All of the organisms listed are protozoa. There are two intestinal protozoa specifically associated with AIDS that can cause transient diarrhea in immunocompetent individuals but can cause debilitating, and potentially life- threatening chronic diarrhea in AIDS patients. These organisms are Isospora belli, treated with trimethoprim-sulfamethoxazole (or other folate antagonists) and Cryptosporidium parvum. Diphyllobothrium latum is the fish tapeworm and occasionally causes diarrhea.
  • 28. Entamoeba histolytica and Giardia lamblia are both causes of diarrhea, but they are not specifically associated with AIDS. Sample Previous Year Question on HIV based on previous Year Questions of NEET PG, USMLE,PLAB,FMGE (MCI Screening). Please visit www.medicoapps.org for more such Quizzes HIV patient is admitted with malabsorption, fever, chronic diarrhoea and acid fast positive organism. What is the likely causative agent? A: Giardia B: Microsporidia C: Isospora D: E. histolytica Correct Ans:C Explanation The coccidian parasite Isospora belli causes human intestinal disease. Infection is acquired by the consumption of oocysts, after which the parasite invades intestinal epithelial cells and undergoes both sexual and asexual cycles of development. Oocysts excreted in stool are not immediately infectious but must undergo further maturation. In patients who have AIDS, infections often are not self-limited but rather resemble cryptosporidiosis, with chronic, profuse watery diarrhea. Eosinophilia, which is not found in other enteric protozoan infections, may be detectable. The diagnosis is usually made by detection of the large (25-m) oocysts in stool by modified acid-fast staining. Oocyst excretion may be low-level and intermittent; if repeated stool examinations are unrevealing, sampling of duodenal contents by aspiration or small-bowel biopsy (often with electron-microscopic examination) may be necessary. Ref: Weller P.F. (2012). Chapter 215. Protozoal Intestinal Infections and Trichomoniasis. In D.L. Longo, A.S. Fauci, D.L. Kasper, S.L. Hauser, J.L. Jameson, J. Loscalzo (Eds), Harrison's Principles of Internal Medicine, 18e
  • 29. Sample Previous Year Question on HIV based on previous Year Questions of NEET PG, USMLE,PLAB,FMGE (MCI Screening). Please visit www.medicoapps.org for more such Quizzes A 31 year old HIV-positive man develops a severe pneumonia. Lower respiratory tract secretions obtained by fiberoptic bronchoscopy with bronchoalveolar lavage and stained with methenamine silver stain demonstrate cup-shaped cysts with sharply outlined walls. Which of the following organisms is the most likely pathogen in this case? A: Candida albicans B: Giardia lamblia C: Haemophilus influenzae D: Pneumocystis carinii Correct Ans:D Explanation The organism described is Pneumocystis carinii, which is an opportunistic parasite that appears to be more closely related to fungi than to protozoa. Its cyst form, when stained with silver stains, has the distinctive appearance described in the question stem, and is typically found in frothy material that occupies the lumen of alveoli. The trophozoites are smaller and much harder to recognize. Bronchoalveolar lavage is considered much more reliable than induced sputum as a diagnostic specimen. Pneumocystis pneumonia is a common infection among AIDS patients, and is very uncommon in other clinical settings. Formerly, many AIDS patients died with Pneumocystis pneumonia, but the combination of early drug treatment (with trimethoprim/sulfamethoxazole or pentamidine) and prophylaxis (usually with trimethoprim/sulfamethoxazole) has decreased the number of fatal infections. In severe cases, Pneumocystis infection can sometimes be demonstrated in extrapulmonary sites. Candida albicans(choice A) can infect the lung and stain with methenamine silver, but the description of the lavage material would probably include the terms fungal hyphae and yeast forms. Giardia lamblia (choice B) causes diarrhea, rather than pneumonia.
  • 30. Haemophilus influenzae(choice C) and Streptococcus pneumoniae(choice E) are bacteria and would not stain with silver stains. Ref: Levinson W. (2012). Chapter 52. Blood & Tissue Protozoa. In W. Levinson (Ed),Review of Medical Microbiology & Immunology, 12e. Sample Previous Year Question on HIV based on previous Year Questions of NEET PG, USMLE,PLAB,FMGE (MCI Screening). Please visit www.medicoapps.org for more such Quizzes A terminally ill HIV infected patient develops focal neurologic signs, dementia, and coma. Amoebic parasites are demonstrated in CSF. Which of the following organisms is most likely to be the causative agent? A: Acanthamoeba sp B: Entamoeba histolytica C: Giardia lamblia D: Naegleria fowleri Correct Ans:A Explanation Two types of free-living amoeba can infect the brain and meninges: Naegleria fowleri and Acanthamoeba species. The former affects healthy adolescent or adult divers, while the latter causes infection in patients with immunosuppression because of diabetes, alcoholism, cancer, or HIV infection. The brain infection characteristically has a prominent perivascular character, which causes a multifocal hemorrhagic necrotizing meningoencephalitis. Skin ulcers, nasal infection, or pneumonia may also be present. It is thought that the organisms may release a toxin causing host tissue necrosis. Systemic antifungal agents (e.g., amphotericin) have some activity against this organism, but most cases are fatal. Entamoeba histolytica causes amoebic dysentery and liver abscess. Giardia lamblia is a flagellate, rather than an amoeba, and causes diarrhea. Naegleria fowleri is an amoebic cause of meningoencephalitis in previously healthy swimmers and divers. Ref: Ray C.G., Ryan K.J. (2010). Chapter 51. Rhizopods. In C.G. Ray, K.J. Ryan (Eds), Sherris Medical Microbiology, 5e.
  • 31. Sample Previous Year Question on HIV based on previous Year Questions of NEET PG, USMLE,PLAB,FMGE (MCI Screening). Please visit www.medicoapps.org for more such Quizzes All are caused by RNA viruses, EXCEPT: A: HIV B: Dengue C: Herpangina D: Fifth disease Correct Ans:D Explanation Fifth disease also known as erythema infectiosum is caused by erythovirus (Parvovirs) B19. Parvoviruses are linear, non segmented single stranded DNA virus. The various manifestations of human parvovirus B19 infection is; • Erythema infectiosum (fifth disease) • Polyarthritis • Aplastic crisis • Chronic anemia • Congenital infection (anemia or hydrops fetalis) Ref: Oski's Pediatrics: Principles And Practice, edited by Julia A. MacMillan, Ralph David Feigin, 4 th Edition, Page 1230 Sample Previous Year Question on HIV based on previous Year Questions of NEET PG, USMLE,PLAB,FMGE (MCI Screening). Please visit www.medicoapps.org for more such Quizzes A 24-year-old man presents with fever, rash, a mild headache, and a sore throat. He denies HIV risk factors, although he is sexually active. On examination, his temperature is 100.8°F and his pulse is 90/min. There is a diffuse, erythematous, maculopapular rash over most of his body. Generalized adenopathy is appreciated, and photophobia is noted when funduscopic examination is
  • 32. attempted. If this man is not treated, which of the following changes in his serologic status will most likely occur? A: The FTA-ABS titer would fall B: The FTA-ABS titer would rise C: The VDRL titer would fall D: The VDRL titer would rise Correct Ans:C Explanation This is a case of syphilis, which is diagnosed serologically, using either treponemal and nontreponemal tests. The FTA-ABS (fluorescent treponemal antibody-absorbed test) is the most widely used of the specific tests and depends on fluorescent labeling of the organisms with anti-treponemal antibody. The specific tests tend to rise early in titer and stay elevated throughout the lifetime of the host, or at least until well after drug therapy has been completed. The VDRL (Venereal Disease Research Laboratory) is a nontreponemal test that detects antibodies that cross-react with mammalian cardiolipin called reaginic antibodies. This test will become positive after the specific test, and its titer will fall late in infection, with or without drug therapy. The FTA-ABS titer would fall is incorrect because this titer will remain high throughout the life of the host, falling off only very slowly after drug cure. The FTA-ABS titer would rise is incorrect because the specific antibody test will reach a high positive titer and remain at that level in untreated late stage syphilis. The VDRL titer would rise is incorrect because reaginic antibody levels will fall in late stage syphilis, with or without drug treatment. Ref: Lukehart S.A. (2012). Chapter 169. Syphilis. In D.L. Longo, A.S. Fauci, D.L. Kasper, S.L. Hauser, J.L. Jameson, J. Loscalzo (Eds), Harrison's Principles of Internal Medicine, 18e Sample Previous Year Question on HIV based on previous Year Questions of NEET PG, USMLE,PLAB,FMGE (MCI Screening). Please visit www.medicoapps.org for more such Quizzes
  • 33. A man presents to a dermatologist because of a severe mucocutaneous rash that involves most of his body,including his palms and soles. Questioning reveals that he is a merchant marine who several months previously had an encounter with a prostitute in Southeast Asia. Which of the following is the most likely causative agent of this rash? A: Herpes simplex I B: Herpes simplex II C: HIV D: Treponema pallidum Correct Ans:D Explanation The rash described is that of secondary syphilis, caused by Treponema pallidum. Involvement of palms and soles by a rash is unusual, and secondary syphilis should come to mind. Not all patients with secondary syphilis have a severe form of the rash, and consequently some cases are missed. Primary syphilis takes the form of a painless, button-like mass called chancres. Tertiary syphilis, which is now rare, has a propensity for involving the aorta and central nervous system and can also cause "gummas" (granulomatous-like lesions) in many sites, notably including liver and bone. Herpes simplex I usually causes perioral Herpes simplex II usually causes genital vesicular lesions. HIV does not itself cause a rash, although coinfection with other organisms can result in a rash. Ref: Ray C.G., Ryan K.J. (2010). Chapter 37. Spirochetes. In C.G. Ray, K.J. Ryan (Eds), Sherris Medical Microbiology, 5e. Sample Previous Year Question on HIV based on previous Year Questions of NEET PG, USMLE,PLAB,FMGE (MCI Screening). Please visit www.medicoapps.org for more such Quizzes A 23-year-old, ill-appearing woman comes to the emergency room with a fever. She notes the recent development of nausea, diarrhea, and a rash. Her last menstrual period began 3 days ago.Physical examination is remarkable for blood pressure of 90/45 mm Hg and heart rate of 120 beats per minute. A diffuse
  • 34. erythematous rash with areas of desquamation over the hands and feet is noted. Infection with which of the following agents is the most likely cause of these signs and symptoms? A: Clostridium perfringens B: HIV-1 C: Shigella dysenteriae D: Staphylococcus aureus Correct Ans:D Explanation This patient has toxic shock syndrome (TSS), a multisystem syndrome caused by a toxin (TSST-1) formed by certain strains of S. aureus. TSS usually affects several organ systems (gastrointestinal, renal, hepatic, hematopoietic, musculoskeletal, pulmonary) and can result in death. TSS has historically been associated with the use of tampons in young women, but can also occur in other patient populations. Fever, hypotension, diarrhea, and diffuse rash with desquamation of the hands and feet are common symptoms. Management of shock, renal failure, and adult respiratory distress syndrome (ARDS) are a priority if these conditions are present, in addition to appropriate antibacterial treatment. C. perfringens causes gas gangrene, with necrosis of soft tissues, usually after a traumatic wound. It is also a cause of food poisoning. Although HIV-1 can cause many diverse findings and should never be immediately ruled out, the findings in this patient are most specific for TSS. Shigella dysenteriae is a cause of dysentery characterized by fever, abdominal cramps, and bloody diarrhea. Ref: Ray C.G., Ryan K.J. (2010). Chapter 24. Staphylococci. In C.G. Ray, K.J. Ryan (Eds), Sherris Medical Microbiology, 5e. Sample Previous Year Question on HIV based on previous Year Questions of NEET PG, USMLE,PLAB,FMGE (MCI Screening). Please visit www.medicoapps.org for more such Quizzes
  • 35. A 33 year old HIV-positive male complains of headache and blurred vision. Physical exam reveals papilledema and ataxia. Head CT is normal but CSF obtained by lumbar puncture reveals encapsulated organisms observable with India ink. Which of the following is true concerning this organism? A: It can also be identified with methenamine silver stain B: It consists of branching septate hyphae C: It exists as a mycelial form at room temperature and as yeast at 37° C D: It is an encapsulated non dimorphic yeast found worldwide Correct Ans:D Explanation This patient has cryptococcal meningitis, as evidenced by the "encapsulated organisms observable with India ink" in the CSF (a classic clue). Cryptococcus is a non dimorphic yeast, meaning that it exists only in the yeast form. It is encapsulated (that's why the India ink stain works so well) and it reproduces by budding. It is found worldwide in bird droppings (think pigeons). It can also cause transient pulmonary illness in otherwise healthy individuals. The methenamine silver stain is used primarily to demonstrate Pneumocystis carinii in tissues. Branching septate hyphae are characteristic of Aspergillus fumigatus, among other fungi. Mycoses that exist in mycelial and yeast forms (dimorphism; diphasic;) are Histoplasma capsulatum, Coccidioides immitis, Blastomyces dermatidis, and Sporothrix schenckii. Ref: Ray C.G., Ryan K.J. (2010). Chapter 46. Cryptococcus, Histoplasma, Coccidioides, and Other Systemic Fungal Pathogens. In C.G. Ray, K.J. Ryan (Eds), Sherris Medical Microbiology, 5e. Sample Previous Year Question on HIV based on previous Year Questions of NEET PG, USMLE,PLAB,FMGE (MCI Screening). Please visit www.medicoapps.org for more such Quizzes All are indicators of goal 6 of Millennium Development Goal, EXCEPT: A: HIV prevalence among women aged 15-49 years
  • 36. B: Number of children orphaned by HIV/AIDS C: Prevalence and death rates of tuberculosis D: Prevalence and death rates of malaria Correct Ans:A Explanation Goal: 6. Combat HIV/AIDS, malaria and other diseases 18. HIV prevalence among young people aged 15 to 24 years 19. Condom use rate of the contraceptive prevalence rate 20. Number of children orphaned by HIV/AIDS 21. Prevalence and death rates associated with malaria 22. Proportion of population in malaria-risk areas using effective malaria prevention and treatment measures 23. Prevalence and death rates associated with tuberculosis 24. Proportion of tuberculosis cases detected and cured under Directly Observed Treatment, Short-course (DOTS) Ref: Park’s textbook of Preventive and Social Medicine, 21st edition, page-27. Sample Previous Year Question on HIV based on previous Year Questions of NEET PG, USMLE,PLAB,FMGE (MCI Screening). Please visit www.medicoapps.org for more such Quizzes Targeted intervention programmes for HIV are done among the following group of people, EXCEPT: A: Commercial sex worker B: Migrant laborers C: Street children D: Industrial worker
  • 37. Correct Ans:D Explanation The basic purpose of the targeted interventions for HIV is to reduce the rate of transmission of HIV among the most vulnerable population such as, • Sex workers • Intravenous drug users • Homosexual men • Truckers • Migrant laborers • Street children The main activities under these intervention projects are behaviour change, communication, treatment for STDs and creating an enabling environment that will facilitate behaviour change. Ref: Park’s Textbook of Preventive and Social Medicine, 19th edition, Page 359. Sample Previous Year Question on HIV based on previous Year Questions of NEET PG, USMLE,PLAB,FMGE (MCI Screening). Please visit www.medicoapps.org for more such Quizzes As per WHO 2010 guidelines, when should an HIV infected adult be started on anti retroviral therapy? A: CD4 count less than 600 irrespective of staging B: CD4 count less than 350 irrespective of staging C: CD4 count less than 500 irrespective of staging D: CD4 count less than 450 irrespective of staging Correct Ans:B Explanation Target population 2010 ART guideline 2006 ART guideline HIV+ asymptomatic ARV-naive individuals CD4 ≤350 cells/mm3 CD4 ≤200 cells/mm3 HIV+ symptomatic ARV-naive individuals WHO clinical stage 2 if CD4 ≤350 cells/mm3 OR WHO clinical stage 3 or 4 irrespective WHO stage 2 or 3 and CD4 ≤200 cells/mm3 WHO stage 3 if CD4 not available
  • 38. of CD4 cell count WHO stage 4 irrespective of CD4 cell count Consider treatment for WHO clinical stage 3 and CD4 cell count between 200 and 350 cells/mm3 HIV+ pregnant women CD4 ≤350 cells/mm3 irrespective of clinical symptoms OR WHO clinical stage 3 or 4 irrespective of CD4 cell count WHO stage 1 or 2 and CD4 ≤200 cells/mm3 WHO stage 3 and CD4 ≤350 cells/ mm3 WHO stage 4 irrespective of CD4 count Ref: Epidemiology of communicable diseases Park’s textbook of Preventive and Social Medicine, 21stedition, page-327. Sample Previous Year Question on HIV based on previous Year Questions of NEET PG, USMLE,PLAB,FMGE (MCI Screening). Please visit www.medicoapps.org for more such Quizzes You are asked to help consult on a study which was recently completed in coastal African country. In this study, a total of 274 soldiers stationed in three different camps were examined and tested for the presence of bacterial sexually transmitted diseases (STD) and human immunodeficiency virus (HIV) positivity. In addition to the information from the clinical exam and laboratory specimens, interviews were conducted to gather information on age, years of military service, ethnicity, and region of origin. The data are now to be analyzed to learn which factors were associated with the presence of STDs, including HIV. What is the most accurate description of this study design? A: A case-control study B: A cohort study C: A clinical trial D: A cross-sectional study Correct Ans:D Explanation
  • 39. In a cross-sectional study, a variety of independent variables can be studied for their association with the outcome variables. In this example, the independent variables would include age, years of military service, ethnicity, and region of origin. The outcome variables include the presence of bacterial sexually transmitted diseases (STDs) and HIV positivity. This study does not meet the criteria for being a case-control, cohort, or case-cohort study, nor is it a clinical trial. The cross-sectional study is limited in its power, and valid inferences may be especially difficult to make from cross-sectional studies, even when statistically significant associations are observed. Sample Previous Year Question on HIV based on previous Year Questions of NEET PG, USMLE,PLAB,FMGE (MCI Screening). Please visit www.medicoapps.org for more such Quizzes HIV sentinel surveillance is used to calculate: A: Detection of high risk group B: Prevalence C: Monitoring disease trend D: Detection of incidence Correct Ans:C Explanation HIV sentinel surveillance used to calculate Trend finding HIV sentinel surveillance (HSS) in India • HIV sentinel surveillance (HSS) in India, since its inception in 1998, has evolved into a credible and robust system for HIV epidemic monitoring and acclaimed as one of the best in the world. • Sentinel surveillance provides essential information to understand the trends and dynamics of HIV epidemic among different risk groups in the country • It aids in refinement of strategies and prioritization of focus for prevention, care and treatment intervention under the National AIDS Control Programme (NACP) • HIV estimates of prevalence, incidence and mortality developed based on findings from HIV Sentinel Surveillance enable the programme in assessing the impacts the impacts at a macro level.
  • 40. Ref: Park 22nd ed/ 400. Sample Previous Year Question on HIV based on previous Year Questions of NEET PG, USMLE,PLAB,FMGE (MCI Screening). Please visit www.medicoapps.org for more such Quizzes KAP studies in India were first used to study: A: HIV B: Malaria C: Family planning D: Cancer cervix Correct Ans:C Explanation KAP studies were used to do family planning studies. It gives a description as to what people think and know about fertility and family planning. It also measures as to how effective is family planning programme. HIV, malaria, and cervical cancer were also studied using KAP. But family planning were the earliest studied by applying the principles of KAP. Ref: Studies in Family Planning - India By Kamala Gopal Rao, Pages 840-42 Sample Previous Year Question on HIV based on previous Year Questions of NEET PG, USMLE,PLAB,FMGE (MCI Screening). Please visit www.medicoapps.org for more such Quizzes All of the following are zoonosis, except: A: Plague
  • 41. B: Japanese Encephalitis (JE) C: HIV D: Tuberculosis (TB) Correct Ans:C Explanation Zoonosis is defined by the Joint FAO/WHO Expert Committee as 'those diseases and infections which are naturally transmitted between vertebrate animals and man’. There are more than 150 diseases and infections having reservoir as animals and is communicable to man. These zoonotic diseases may be caused by viruses, bacteriae, rickettisiae, protozoa, helminths, anthropods, fungi or insects. HIV is not a zoonotic disease while the other three infections are zoonotic diseases. The human immunodeficiency virus has only human as reservoirs. Ref: Park Textbook of Preventive and Social Medicine, 19th Edition, Pages 642, 688 Sample Previous Year Question on HIV based on previous Year Questions of NEET PG, USMLE,PLAB,FMGE (MCI Screening). Please visit www.medicoapps.org for more such Quizzes Which of the following is the number of national helpline for HIV/AIDS? A: 1081 B: 109 1 C: 1097 D: 1100 Correct Ans:C Explanation A toll free national AIDS telephone help line has been set up to provide access to information and counselling. Ref: Park 21st edition, page 399.
  • 42. Sample Previous Year Question on HIV based on previous Year Questions of NEET PG, USMLE,PLAB,FMGE (MCI Screening). Please visit www.medicoapps.org for more such Quizzes A new drug with in vitro activity against HIV is tested on a population of patients with Western-blot confirmed HIV infections, in Mumbai. Out of the 200 individuals in the patient population, 100 individuals are chosen by a lot to receive the drug. The drug, which is tasteless, is administered in a cup of orange juice; the other patients receive pure orange juice. Neither the nurses, doctors, nor patients know which patients receive the drug. At the end of the study period, the number of CD4+ T cells is determined for all of the subjects. What is the research method used in this study? A: Case-control study B: Case report C: Cohort study D: Double-blind randomized clinical trial Correct Ans:D Explanation This study conducted in Mumbai is an example of a placebo-controlled, double-blind randomized clinical trial. In this type of study, subjects with a particular disorder are randomized (the lottery) to receive either the treatment in question or a placebo. The information about whether the patient receives the treatment or the placebo is not known to either the subject or the investigator (double-blind). This type of study may be used to infer causality, i.e., if the patients taking the medication have more CD4+ T cells than the other group, it is due to the drug in question. A case-control study (Choice : Case-control study) is a retrospective study that pairs known cases of a disease with matched controls. This type of study gives information about the importance of risk factors. The lack of a prospective design makes inferences about causality difficult. A case report (Choice : Case report) is a published report of a single incidence of a disease in a subject. This is usually done for rare diseases, or for rare associations of a disease with a particular risk factor. A cohort study (Choice : Cohort study) is an observational (non-interventional) study that tracks subjects prospectively. This type of study gives a measure of incidence of a disease. Sample Previous Year Question on HIV based on previous Year Questions of NEET PG, USMLE,PLAB,FMGE (MCI Screening). Please visit www.medicoapps.org for more such Quizzes In a city with a population of 1,000,000, 10,000 individuals have HIV disease. There are 1000 new cases of HIV disease and 200 deaths each year from the disease. There are 2500
  • 43. deaths per year from all causes. Assuming no net emigration from or immigration to the city, the incidence of HIV disease in this city is given by which of the following? A: 200/1,000,000 B: 800/1,000,00 0 C: 1000/1,000,000 D: 2500/1,000,000 Correct Ans:C Explanation The incidence of a disease is given by the number of new cases in a given period divided by the total population. In this case, this is equal to 1000/1,000,000. The disease-specific mortality rate is the number of deaths per year from a specific disease divided by the population; in this case, 200/1,000,000. The rate of increase of a disease is given by the number of new cases per year, minus the number of deaths (or cures) per year, divided by the total population. Since there is yet no cure for HIV disease, the number of cures is 0. In this case, the rate of increase is (1000-200)/1,000,000 = 800/1,000,000. The crude mortality rate is given by the number of deaths from all causes, divided by the population; in this case, 2500/1,000,000. Sample Previous Year Question on HIV based on previous Year Questions of NEET PG, USMLE,PLAB,FMGE (MCI Screening). Please visit www.medicoapps.org for more such Quizzes All of the following are zoonosis, except: A: Plague B: Japanese Encephalitis (JE) C: HIV D: Tuberculosis (TB) Correct Ans:C Explanation Zoonosis is defined by the Joint FAO/WHO Expert Committee as 'those diseases and infections which are naturally transmitted between vertebrate animals and man’. There are more than 150 diseases and infections having reservoir as animals and is communicable to man. These zoonotic diseases may be caused by viruses, bacteriae, rickettisiae, protozoa, helminths, anthropods, fungi or insects.
  • 44. HIV is not a zoonotic disease while the other three infections are zoonotic diseases. The human immunodeficiency virus has only human as reservoirs. Ref: Park Textbook of Preventive and Social Medicine, 19th Edition, Pages 642, 688 Sample Previous Year Question on HIV based on previous Year Questions of NEET PG, USMLE,PLAB,FMGE (MCI Screening). Please visit www.medicoapps.org for more such Quizzes Which of the following exposure carries the maximum risk of transmission of HIV? A: Needle Prick B: Trans-Placental C: Blood Transfusion D: Sexual Intercourse Correct Ans:C Explanation It is estimated that 90% of individuals who were exposed to HIV - contaminated products became infected. So, blood transfusion carries maximum risk of HIV transmission. 75-90% of individuals who received contaminated factor VIII acquire HIV. Ref: Harrison’s Principles of Internal Medicine, 17th Edition, Page 1144; Sexual and Reproductive Health: A Public Health Perspective By Paul Van Look, Pages 117-9 Sample Previous Year Question on HIV based on previous Year Questions of NEET PG, USMLE,PLAB,FMGE (MCI Screening). Please visit www.medicoapps.org for more such Quizzes One of your staff nurse had a deep prick from the needle used to inject an HIV positive individual. What is the treatment regime that should be started in her/him? Note that drug resistance was suspected in the HIV patient. A: 2 NRTIs for 3 months B: 1 NRTI and 1 NtRTI for 28 days C: 2 NRTIs + 1 protease inhibitor for 28 days D: 2 protease inhibitor + 2 NtRTIs for 3 months
  • 45. Correct Ans:C Explanation In this scenario ARVs should be initiated as soon as possible and no later than 72 hours. It should include 2 NRTI drugs. If drug resistance is suspected a protease inhibitor should also be added. The duration of treatment is 28 days. If the blood sample drawn from the exposed person is HIV negative at the time of exposure, the test should be repeated at 3 and 6 months. Ref: Park, Edition 21, Page - 328 Sample Previous Year Question on HIV based on previous Year Questions of NEET PG, USMLE,PLAB,FMGE (MCI Screening). Please visit www.medicoapps.org for more such Quizzes WHO stage IV HIV includes all, EXCEPT: A: Toxoplasmosis B: Pneumocystis carinii C: HIV wasting syndrome D: Oral thrush Correct Ans:D Explanation According to WHO clinical staging system for HIV infection, oral thrush is seen in stage III disease. • Stage IV includes, • HIV wasting syndrome • Pneumocystis carinii pneumonia • Toxoplasmosis of the brain • Cryptosporidiosis • Cryptococcosis • Cytomegalovirus disease • Herpes virus infection • Progressive multifocal leukoencephalopathy • Disseminated fungal infections • Candidiasis of the esophagus, trachea, bronchi, or lungs • Atypical mycobacteriosis • Non-typhoid salmonella septicemia • Extrapulmonary TB • Lymphoma • Kaposi sarcoma • HIV encephalopathy Ref: Park’s Textbook of Preventive and Social Medicine, 19th edition, Page 292.
  • 46. Sample Previous Year Question on HIV based on previous Year Questions of NEET PG, USMLE,PLAB,FMGE (MCI Screening). Please visit www.medicoapps.org for more such Quizzes According to CDC recommendations, HIV screening of pregnant women is: A: Opt in testing B: Opt out testing C: Compulsory D: Symptomatic Correct Ans:B Explanation The Center for Disease Control and Prevention, the American College of Obstetricians and Gynecologists, the American Academy of Pediatrics, and the United States Preventive Services Task Force recommend prenatal screening using an opt-out approach. This means that the woman is notified that HIV testing is included in a comprehensive set of antenatal tests, but that testing may be declined. Women are given information regarding HIV but are not required to sign a specific consent. Through the use of such opt-out strategies, HIV testing rates have increased. Each provider should be aware of specific state laws concerning screening. Ref: Cunningham F.G., Leveno K.J., Bloom S.L., Hauth J.C., Rouse D.J., Spong C.Y. (2010). Chapter 59. Sexually Transmitted Diseases. In F.G. Cunningham, K.J. Leveno, S.L. Bloom, J.C. Hauth, D.J. Rouse, C.Y. Spong (Eds), Williams Obstetrics, 23e. Sample Previous Year Question on HIV based on previous Year Questions of NEET PG, USMLE,PLAB,FMGE (MCI Screening). Please visit www.medicoapps.org for more such Quizzes Cotrimoxazole therapy is to be given in HIV infected patients irrespective of presence of symptoms if CD4 count is less than: A: 100 B: 15 0 C: 200 D: 350 Correct Ans:C Explanation Routine prophylaxis with co-trimoxazole (CPT) is provided under the national programme. Under the national programme, CPT may be initiated in the following scenarios: If CD4 is not available (or result pending): WHO clinical stage 3 and 4
  • 47. If CD4 is available: • HIV infected adults with CD4 <200 cells/mm • or CD4 <350 cells/mm if if patient is symptomatic • or WHO clinical stage 3 or 4 irrespective of CD4. Ref: http://naco.gov.in/upload/Policies%20&%20Guidelines/1.%20Antiretroviral%20The rapy%20Guidelines%20for%20HIV- Infected%20Adults%20and%20Adolescents%20Including%20Post-exposure.pdf Sample Previous Year Question on HIV based on previous Year Questions of NEET PG, USMLE,PLAB,FMGE (MCI Screening). Please visit www.medicoapps.org for more such Quizzes Anti retroviral therapy is to be given in HIV infected patients irrespective of presence of symptoms if CD4 count is less than: A: 100 B: 15 0 C: 200 D: 350 Correct Ans:C Explanation • Offer ART to symptomatic patients if the CD4 count is 200–350 cells/mm3 • Consider ART for asymptomatic patients with CD4 count between 200-350 cells/mm3 and monitor closely for new symptoms. • If the CD4 count is 200–250 cells/mm3, physicians can consider repeating the CD4 test in 4 weeks in asymptomatic patients. This is to rule out the possibility of a 20% margin of error in laboratory results. • Patients should start ART before the CD4 count drops below 200 cells/mm3 Ref: http://naco.gov.in/upload/Policies%20&%20Guidelines/1.%20Antiretroviral%20The rapy%20Guidelines%20for%20HIV- infected%20Adults%20and%20Adolescents%20Including%20Post-exposure.pdf Sample Previous Year Question on HIV based on previous Year Questions of NEET PG, USMLE,PLAB,FMGE (MCI Screening). Please visit www.medicoapps.org for more such Quizzes
  • 48. Anti retroviral therapy is to be given in HIV infected patients irrespective of presence of symptoms if CD4 count is less than: A: 100 B: 15 0 C: 200 D: 350 Correct Ans:C Explanation • Offer ART to symptomatic patients if the CD4 count is 200–350 cells/mm3 • Consider ART for asymptomatic patients with CD4 count between 200-350 cells/mm3 and monitor closely for new symptoms. • If the CD4 count is 200–250 cells/mm3, physicians can consider repeating the CD4 test in 4 weeks in asymptomatic patients. This is to rule out the possibility of a 20% margin of error in laboratory results. • Patients should start ART before the CD4 count drops below 200 cells/mm3 Ref: http://naco.gov.in/upload/Policies%20&%20Guidelines/1.%20Antiretroviral%20The rapy%20Guidelines%20for%20HIV- infected%20Adults%20and%20Adolescents%20Including%20Post-exposure.pdf Sample Previous Year Question on HIV based on previous Year Questions of NEET PG, USMLE,PLAB,FMGE (MCI Screening). Please visit www.medicoapps.org for more such Quizzes You are asked to help consult on a study which was recently completed in coastal African country. In this study, a total of 274 soldiers stationed in three different camps were examined and tested for the presence of bacterial sexually transmitted diseases (STD) and human immunodeficiency virus (HIV) positivity. In addition to the information from the clinical exam and laboratory specimens, interviews were conducted to gather information on age, years of military service, ethnicity, and region of origin. The data are now to be analyzed to learn which factors were associated with the presence of STDs, including HIV. What is the most accurate description of this study design? A: A case-control study B: A cohort study C: A clinical trial D: A cross-sectional study Correct Ans:D
  • 49. Explanation In a cross-sectional study, a variety of independent variables can be studied for their association with the outcome variables. In this example, the independent variables would include age, years of military service, ethnicity, and region of origin. The outcome variables include the presence of bacterial sexually transmitted diseases (STDs) and HIV positivity. This study does not meet the criteria for being a case-control, cohort, or case-cohort study, nor is it a clinical trial. The cross-sectional study is limited in its power, and valid inferences may be especially difficult to make from cross-sectional studies, even when statistically significant associations are observed. Sample Previous Year Question on HIV based on previous Year Questions of NEET PG, USMLE,PLAB,FMGE (MCI Screening). Please visit www.medicoapps.org for more such Quizzes A HIV infected patient develops multibacillary leprosy. What is the recommended treatment? A: No anti leprosy drug should be given B: Rifampicin, dapsone and clofazimine C: Rifampicin and dapsone D: Rifampicin only Correct Ans:B Explanation Multi drug therapy for leprosy is not contraindicated in HIV infection. WHO recommends the following drugs for treatment of multibacillary leprosy • Rifampicin: 600 mg once monthly • Dapsone: 100 mg daily • Clofazimine: 300 mg once monthly supervised; and 50 mg daily, self administered. Ref: Park 21st edition, page 297. Sample Previous Year Question on HIV based on previous Year Questions of NEET PG, USMLE,PLAB,FMGE (MCI Screening). Please visit www.medicoapps.org for more such Quizzes Tonsillar carcinoma is associated with infection of:
  • 50. A: HI V B: HPV C: HSV D: None of the above Correct Ans:B Explanation HPV is an epitheliotropic virus that has been detected to varying degrees within samples of oral cavity squamous cell carcinoma. Infection alone is not considered sufficient for malignant conversion; however, results of multiple studies suggest a role of HPV in a subset of head and neck squamous cell carcinoma. Approximately 40% of tonsillar carcinomas demonstrate evidence of HPV types 16 and 18. Ref: Schwartz’s principle of surgery 9th edition, chapter 18. Sample Previous Year Question on HIV based on previous Year Questions of NEET PG, USMLE,PLAB,FMGE (MCI Screening). Please visit www.medicoapps.org for more such Quizzes Which of the following is an absolute contraindication for renal transplant? A: Untreated malignancy B: Age >65 years C: HIV infection D: Chronic hepatitis B or C Correct Ans:A Explanation Absolute contraindications to kidney transplantation Untreated malignancy Active infection Untreated HIV infection or AIDS Any condition where life expectancy is under two years Relative contraindications to kidney transplantation Comorbid condition Age >65 years Untreated coronary artery disease Obesity HIV infection Previous malignancy Chronic hepatitis B or C
  • 51. Ref: CLINICAL REVIEW, Renal transplantation, BMJ 2011;343:d7300 ; The Renal Association. Assessment of the potential kidney transplant recipient. 2011. Sample Previous Year Question on HIV based on previous Year Questions of NEET PG, USMLE,PLAB,FMGE (MCI Screening). Please visit www.medicoapps.org for more such Quizzes Salmonella osteomyelitis is common in which of the following? A: Sickle cell disease B: HIV C: IV drug abusers D: Pregnancy Correct Ans:A Explanation Increased incidence of salmonella (may spread from gallbladder infection) is seen in sickle cell anemia, but staph aureus is still the most common. Staph aureus is the most common cause of osteomyelits in all children, including those with sickle cell disease. Salmonella osteomyelitis occurs most commonly in children with sickle cell disease, but is still less common than Staph aureus in these patients. Osteomyelitis most often affects diaphysis. Sample Previous Year Question on HIV based on previous Year Questions of NEET PG, USMLE,PLAB,FMGE (MCI Screening). Please visit www.medicoapps.org for more such Quizzes Which of the following congenital infections is associated with minimal teratogenic risk to the fetus: A: HIV B: Rubella C: Varicella D: CMV Correct Ans:A Explanation HIV infection in pregnant women has no proven teratogenic effects on the fetus. While the other three are teratogenic infections. Ref: Dutta textbook of Obstetrics 6th edition, page 299
  • 52. Sample Previous Year Question on HIV based on previous Year Questions of NEET PG, USMLE,PLAB,FMGE (MCI Screening). Please visit www.medicoapps.org for more such Quizzes All of the following interventions are recommended to prevent mother to child transmission of HIV, except: A: Avoid Ergometrine in third stage of labour B: Highly Active Antiretroviral Therapy (HAART) C: Elective Caesarean section D: Intrapartum Zidovudine Correct Ans:A Explanation Ergometrine is give the third stage of labour to reduce post partum haemorrhage, it has no role in the prevention of HIV transmission. Ref: Williams Obstetrics, 23rd Edition, Page 1251,1252, 1253; Textbook of Obstetrics By DC Dutta, 6th Edition, Page 301; Disease Control Priorities in Developing Countries By Dean T. Jamison, World Bank, Disease Control Priorities Project, Page 346 Sample Previous Year Question on HIV based on previous Year Questions of NEET PG, USMLE,PLAB,FMGE (MCI Screening). Please visit www.medicoapps.org for more such Quizzes A patient presented with intense vulvar pruritus, a white curd like, cheesy vaginal discharge and vulvar erythema. Candidiasis was diagnosed. Which among the following is NOT a risk factor for vaginal candidiasis? A: Diabetes mellitus B: Hypertensio n C: Human Immunodeficiency Virus (HIV) infection D: Pregnancy Correct Ans:B Explanation Risk factors of candida infection include: • Diabetes mellitus • Human immunodeficiency virus [HIV]
  • 53. • Obesity • Pregnancy • Medication (antibiotics, corticosteroids, oral contraceptives) • Chronic debilitation Ref: Mazdisnian F. (2007). Chapter 37. Benign Disorders of the Vulva & Vagina. In A.H. DeCherney, L. Nathan (Eds), CURRENT Diagnosis & Treatment Obstetrics & Gynecology, 10e. Sample Previous Year Question on HIV based on previous Year Questions of NEET PG, USMLE,PLAB,FMGE (MCI Screening). Please visit www.medicoapps.org for more such Quizzes A 24 year old female with multiple sex partners presented with vaginal discharge which is thin and white with fishy odor. Whiff test is positive which indicates the infection with which of the following organism? A: TORCH B: HI V C: Bacterial vaginosis D: Birth anoxia Correct Ans:C Explanation Whiff test is positive for bacterial vaginosis. It involves the production of a fishy odor when mixing vaginal fluid with 10% potassium hydroxide. Amstel criteria for diagnosing BV: Three of the following four must be present. • Thin, homogenous vaginal discharge • A positive whiff test • Vaginal fluid pH greater than 4.5 • Presence of clue cells (epithelial cells covered with bacteria) on microscopic examination Ref: Rosen T. (2012). Chapter 205. Gonorrhea, Mycoplasma, and Vaginosis. In L.A. Goldsmith, S.I. Katz, B.A. Gilchrest, A.S. Paller, D.J. Leffell, N.A. Dallas (Eds), Fitzpatrick's Dermatology in General Medicine, 8e.
  • 54. Sample Previous Year Question on HIV based on previous Year Questions of NEET PG, USMLE,PLAB,FMGE (MCI Screening). Please visit www.medicoapps.org for more such Quizzes Virus associated with cancer cervix is: A: HPV B: HI V C: EBV D: HTLV Correct Ans:A Explanation HPV has a established strong association in the preponderance of cervical cancer. Persistent HPV infection along with metaplasia resulting from any other etiology results in various degrees of dysplasia. HPV infection is diagnosed by demonstrating koilocytes(large cells with vacuolated cytoplasm and large bland nucleus) and dyskaryocytes (small clusters of cells with orangophilic cytoplasm and dark small nuclei) in cytology; it can also be confirmed with PCR - DNA. Ref: Recent Advances in Adolescent Health By Roza Olyai, Page 32 ; Cancer Associated Viruses By Erle Robertson, Page 509 ; Human Oncogenic Viruses By Jing-Hsiung James Ou, T. S. Benedict Yen, Pages 5-11 Sample Previous Year Question on HIV based on previous Year Questions of NEET PG, USMLE,PLAB,FMGE (MCI Screening). Please visit www.medicoapps.org for more such Quizzes Patient diagnosed as squamous cell intraepithelial lesion. Which of the following has the highest risk for progression to carcinoma? A: Low grade squamous intraepithelial neoplasia B: High grade squamous intraepithelial neoplasia C: Squamous intraepithelial neoplasia associated with HPV 16 D: Squamous intraepithelial neoplasia associated with HIV
  • 55. Correct Ans:B Explanation Cervical intraepithelial neoplasia (CIN grades I–III) and squamous intraepithelial lesions (SIL) are dysplastic changes in the cervical epithelium that are a precursor to malignancy (i.e., squamous cell carcinoma of the cervix) and are the result of infection with HPV. High-grade SIL and CIN II–III lesions are caused by HPV types 16, 18, 31, 33, 35, 39, 45, and 52, and have a high risk for progression to an invasive squamous cell carcinoma. Low-grade SIL and CIN I lesions are caused by HPV types 6, 11, 42, and 44, and have a low risk for progression to squamous cell carcinoma. Ref: Kemp W.L., Burns D.K., Brown T.G. (2008). Chapter 17. Pathology of the Male and Female Reproductive Tract and Breast. In W.L. Kemp, D.K. Burns, T.G. Brown (Eds), Pathology: The Big Picture Sample Previous Year Question on HIV based on previous Year Questions of NEET PG, USMLE,PLAB,FMGE (MCI Screening). Please visit www.medicoapps.org for more such Quizzes All the following mechanisms occur in a neonate for heat production, EXCEPT: A: Shivering B: Breakdown of brown fat with adrenaline secretion C: Universal flexion like a fetus D: Cutaneous vasoconstriction Correct Ans:A Explanation Non shivering thermogenesis or breaking of brown fat is the main method of heat production in the newborn. Shivering which is a major mechanism of heat production in adults is rarely seen in newborn. In newborns, brown fat is located in the upper back and neck, mediastinum and around the kidneys. They contain high store of triglycerides which are broken down during metabolic process to produce glycerol and fatty acids which in turn releases heat.
  • 56. Mechanisms of heat production in newborn are: • Newborn’s mechanism of heat production is primarily through increased metabolic process. Cold stress increases the metabolism and thus requiring an increase in oxygen and calorie consumption. • Vasoconstriction of blood vessels. • Increase in metabolic rate and muscular activity. • Non shivering thermogenesis Ref: Universal Newborn Hearing Screening edited by Lynn G. Spivak page 121, Examination of the Newborn: An Evidence Based Guide By Anne Lomax page 56. Sample Previous Year Question on HIV based on previous Year Questions of NEET PG, USMLE,PLAB,FMGE (MCI Screening). Please visit www.medicoapps.org for more such Quizzes A preterm baby developed bullous lesion on the skin soon after birth. The X- ray shows periostitis. What should be the next investigation? A: VDRL for mother & baby B: ELISA for HIV C: PCR for TB D: Hepatitis surface antigen for mother Correct Ans:A Explanation The baby described in the question stem is most likely suffering from the early manifestations of congenital syphilis. As per the choices given VDRL is the test done to diagnose syphilis. Ref: Harrison’s Principles of Internal Medicine, 16th Edition, Page 981; Skin Diseases & Sexually Transmitted Infections, Uday Khopkar – 6th Edition, Page 257; Nelson 17th ,COGDT 10th Edition, Page 667;CPDT 18th Edition, Page 56.
  • 57. Sample Previous Year Question on HIV based on previous Year Questions of NEET PG, USMLE,PLAB,FMGE (MCI Screening). Please visit www.medicoapps.org for more such Quizzes Which response is not seen in newborns as a measure of thermogenesis? A: Shivering B: Breakdown of brown fat C: Universal flexion D: Cutaneous vasoconstrictions Correct Ans:A Explanation Newborns capacity to generate heat by shivering is limited. Primary mechanism of heat production in the neonate is through metabolism of brown fat or through non shivering thermogenesis. Major location of brown fat storage in neonates are upper back and neck, mediastinum and around the kidneys. Brown fat contains hish stores of triglycerides which is broken down into glycerol and fatty acids. Oxidation of fatty acids result in the generation of heat. Sample Previous Year Question on HIV based on previous Year Questions of NEET PG, USMLE,PLAB,FMGE (MCI Screening). Please visit www.medicoapps.org for more such Quizzes A preterm baby developed bullous lesion on the skin soon after birth. The X- ray shows periostitis. What should be the next investigation? A: VDRL for mother & baby B: ELISA for HIV C: PCR for TB D: Hepatitis surface antigen for mother Correct Ans:A Explanation The baby described in the question stem is most likely suffering from the early manifestations of congenital syphilis. As per the choices given VDRL is the test done to diagnose syphilis.
  • 58. Ref: Harrison’s Principles of Internal Medicine, 16th Edition, Page 981; Skin Diseases & Sexually Transmitted Infections, Uday Khopkar – 6th Edition, Page 257; Nelson 17th ,COGDT 10th Edition, Page 667;CPDT 18th Edition, Page 56. Sample Previous Year Question on HIV based on previous Year Questions of NEET PG, USMLE,PLAB,FMGE (MCI Screening). Please visit www.medicoapps.org for more such Quizzes Which of the following investigation is the BEST method of diagnosing HIV in childhood? A: CD4 cell counts B: P24 antigen C: ELISA D: Anti HIV antibody Correct Ans:B Explanation The two tests used for diagnosing HIV infection in in children less than 18 months are detection of HIV nucleic acid (DNA or RNA) AND detection of HIV p24 antigen. Definitive diagnosis requires presence of reactive tests on two separate determinations using one of the above tests after 4 or 6 weeks and confirmed by the second test. Diagnostic testing should be done 6 weeks or longer after complete cessation of breast feeding. Laboratory criteria for diagnosing HIV infection in children more than 18 months: • Serologic testing • Detection of HIV nucleic acid(DNA or RNA) • Detection of HIV p24 antigen Ref: Advances in Pediatrics by Dutta By Jaypee Brothers, page 256.
  • 59. Sample Previous Year Question on HIV based on previous Year Questions of NEET PG, USMLE,PLAB,FMGE (MCI Screening). Please visit www.medicoapps.org for more such Quizzes Which of the following investigation is the BEST method of diagnosis of HIV in childhood? A: CD4 cell counts B: P24 antigen C: ELISA D: Anti HIV antibody Correct Ans:B Explanation The two tests used for diagnosing HIV infection in in children less than 18 months are detection of HIV nucleic acid (DNA or RNA) AND detection of HIV p24 antigen.Definitive diagnosis requires presence of reactive tests on two separate determinations using one of the above tests after 4 or 6 weeks and confirmed by the second test. Diagnostic testing should be done 6 weeks or longer after complete cessation of breast feeding. Infants born to HIV infected mothers will have HIV antibody until 6-18 months regardless of infection status due to transplacental passage of maternal antibody. The preferred test for infant diagnosis is detection of HIV DNA or RNA in blood, collectively referred to as nucleic acid amplification testing (NAT). Laboratory criteria for diagnosing HIV infection in children more than 18 months: • Serologic testing • Detection of HIV nucleic acid(DNA or RNA) • Detection of HIV p24 antigen
  • 60. Ref: Advances in Pediatrics by Dutta By Jaypee Brothers, page 256. McFarland E.J. (2012). Chapter 41. Human Immunodeficiency Virus Infection. In W.W. Hay, Jr., M.J. Levin, R.R. Deterding, J.J. Ross, J.M. Sondheimer (Eds), CURRENT Diagnosis & Treatment: Pediatrics, 21e. Sample Previous Year Question on HIV based on previous Year Questions of NEET PG, USMLE,PLAB,FMGE (MCI Screening). Please visit www.medicoapps.org for more such Quizzes A patient with HIV is on therapy with protease inhibitors. He presents with limitation of abduction and internal rotation of the hip. The most probable diagnosis is: A: Tuberculosis of hip joint B: Avascular necrosis (AVN) of femoral head C: Secondary osteoarthritis of hip D: Septic arthritis of hip Correct Ans:B Explanation Limitation of abduction and internal rotation of hip is often seen in avascular necrosis of femoral head or Legg-Calve-Perthes disease. Hypertriglyceridemia is a well known risk factor of avascular necrosis of femoral head (AVN). It has been suggested that treatment with protease inhibitors increase triglyceride levels, thereby increasing risk of avascular necrosis of femoral head. Ref: Current Diagnosis and Treatment of Pain (CDTP), 2006, Page 305; Antiretrovial Therapy By Clercq, 2001, Page 167; Apley’s System of Orthopaedics and Fractures By Louis Solomon, 9th Edition, Chapter 19, The Hip, Pages 511-15 Sample Previous Year Question on HIV based on previous Year Questions of NEET PG, USMLE,PLAB,FMGE (MCI Screening). Please visit www.medicoapps.org for more such Quizzes Which one of the following viruses is least likely to cross placenta? A: Rubella B: Herpes Simplex Virus
  • 61. C: HIV D: HBV Correct Ans:D Explanation Considering all the above given options, the hepatitis B virus is least likely to cross the placenta due to its larger molecular size (virion of 42nm in diameter) which gives it less access through the placenta. It is not teratogenic. Ref: Dutta Textbook of obstetrics 6th edition, Pages 292, 299-300; Medical disorders in pregnancy: an update By Hiralal Konar, Pralhad Kushtagi, Pages 67-74. Sample Previous Year Question on HIV based on previous Year Questions of NEET PG, USMLE,PLAB,FMGE (MCI Screening). Please visit www.medicoapps.org for more such Quizzes All of the following interventions are recommended to prevent mother to child transmission of HIV, except: A: Avoid Ergometrine in third stage of labour B: Highly Active Antiretroviral Therapy (HAART) C: Elective Caesarean section D: Intrapartum Zidovudine Correct Ans:A Explanation Ergometrine is give the third stage of labour to reduce post partum haemorrhage, it has no role in the prevention of HIV transmission. Ref: Williams Obstetrics, 23rd Edition, Page 1251,1252, 1253; Textbook of Obstetrics By DC Dutta, 6th Edition, Page 301; Disease Control Priorities in Developing Countries By Dean T. Jamison, World Bank, Disease Control Priorities Project, Page 346 Sample Previous Year Question on HIV based on previous Year Questions of NEET PG, USMLE,PLAB,FMGE (MCI Screening). Please visit www.medicoapps.org for more such Quizzes All of the following interventions are recommended to prevent mother to child transmission of HIV, except: A: Avoid breast feeding B: Highly Active Antiretroviral Therapy (HAART) C: Vaginal delivery
  • 62. D: Intrapartum Zidovudine Correct Ans:C Explanation For HIV-infected mothers, a scheduled cesarean delivery at 38 weeks can reduce the risk of mother - to - child transmission of the virus. If, before her scheduled cesarean delivery, rupture of membranes or she goes into labor, a cesarean delivery may not reduce the risk of mother to-child transmission of HIV. Ref: Recommendations for Use of Antiretroviral Drugs in Pregnant HIV-1-Infected Women By Michael A. Stoto, Donna A. Almario, Institute of Medicine (U.S.). Committee on Perinatal Transmission of HIV, National Research Council (U.S.). Board on Children, Youth, and Families, Pages 358-360. Sample Previous Year Question on HIV based on previous Year Questions of NEET PG, USMLE,PLAB,FMGE (MCI Screening). Please visit www.medicoapps.org for more such Quizzes Non Immune Hydrops Fetalis is caused by: A: CMV B: Parvovirus C: HSV D: HI V Correct Ans:B Explanation Non immune hydrops can be caused by a variety of causes. Among the infectious cause, parvovirus can lead to non immune hydrops. Hydrops fetalis refers to excessive accumulation of fluid in at least two fetal body cavities. It can be of two types, immune and non immune. Immune type result from hemolysis of fetal RBC by maternal IgG antibodies to fetal antigen. Non immune type, is the result of any non antibody mediated process that lead to hydrops. Ref: Fetal Medicine : Basic Science and Clinical Practice, 2nd Edition, Page 522; Ian Donald's Practical Obstetric Problems, 6th Edition, Page 218; William's Obstetrics, 23rd Edition, Page 627; Moffet's Pediatric Infectious Diseases : A Problem Oriented Approach, 4th Edition, Pages 377, 643. Sample Previous Year Question on HIV based on previous Year Questions of NEET PG, USMLE,PLAB,FMGE (MCI Screening). Please visit www.medicoapps.org for more such Quizzes
  • 63. Premature baby of 34 weeks was delivered. Baby had bullous lesion on the body. X ray shows periostitis. What is the next investigation? A: VDRL for mother & baby B: ELISA for HIV C: PCR for T.B D: Hepatitis surface antigen for mother Correct Ans:A Explanation This child is suffering from congenital syphilis. Here, this mother and infant require quantitative nontreponemal (VDRL, RPR) syphilis testing. The earliest manifestations of congenital syphilis (appearing 2–6 weeks after birth) include rhinitis, or “snuffles” (23%); mucocutaneous lesions (35–41%); bone changes (61%), including osteochondritis, osteitis, and periostitis detectable by x-ray examination of long bones; hepatosplenomegaly (50%); lymphadenopathy (32%); anemia (34%); jaundice (30%); thrombocytopenia; and leukocytosis. Ref: Ogle J.W., Anderson M.S. (2012). Chapter 42. Infections: Bacterial & Spirochetal.In Hay W.W., Jr, Levin M.J., Deterding R.R., Abzug M.J., Sondheimer J.M. (Eds),CURRENT Diagnosis & Treatment: Pediatrics, 21e. Sample Previous Year Question on HIV based on previous Year Questions of NEET PG, USMLE,PLAB,FMGE (MCI Screening). Please visit www.medicoapps.org for more such Quizzes All of the following interventions are recommended to prevent mother to child transmission of HIV, except: A: Avoid Ergometrine in third stage of labour B: Highly Active Antiretroviral Therapy (HAART) C: Elective Caesarean section D: Intrapartum Zidovudine Correct Ans:A Explanation Ergometrine is give the third stage of labour to reduce post partum haemorrhage, it has no role in the prevention of HIV transmission.